Você está na página 1de 122

LINGUAGENS, CDIGOS

e suas tecnologias

Estudo da lngua inglesa no vestibular


BETWEEN ENGLISH AND PORTUGUESE
Alexandre Bacci e Eric Duarte

hexag
SISTEMA DE ENSINO
1 edio
So Paulo
2016

hexag
SISTEMA DE ENSINO

Hexag Editora, 2016


Direitos desta edio: Hexag Editora Ltda. So Paulo, 2016
Todos os direitos reservados.
Autor
Alexandre Bacci
Diretor geral
Herlan Fellini
Coordenador geral
Raphael de Souza Motta
Responsabilidade editorial
Hexag Editora
Diretor editorial
Pedro Tadeu Batista
Editor
Eric Duarte
Revisor
Eric Duarte
Pesquisa iconogrfica
Camila Dalafina Coelho
Programao visual
Hexag Editora
Editorao eletrnica
Arthur Tahan Miguel Torres
Bruno Alves Oliveira Cruz
Camila Dalafina Coelho
Eder Carlos Bastos de Lima
Raphael de Souza Motta
Capa
Hexag Editora
Fotos da capa (de cima para baixo)
http://www.fcm.unicamp.br
Acervo digital da USP (verso beta)
http://www.baia-turismo.com
Impresso e acabamento
Imagem Digital
ISBN: 978-85-68999-04-2
Todas as citaes de textos contidas neste livro didtico esto de acordo com a legislao, tendo por fim nico e exclusivo o
ensino. Caso exista algum texto, a respeito do qual seja necessria a incluso de informao adicional, ficamos disposio
para o contato pertinente. Do mesmo modo, fizemos todos os esforos para identificar e localizar os titulares dos direitos sobre
as imagens publicadas e estamos disposio para suprir eventual omisso de crdito em futuras edies.
O material de publicidade e propaganda reproduzido nesta obra est sendo usado apenas para fins didticos, no representando qualquer tipo de recomendao de produtos ou empresas por parte do(s) autor(es) e da editora.

2016
Todos os direitos reservados por Hexag Editora Ltda.
Rua da Consolao, 954 Higienpolis So Paulo SP
CEP: 01302-000
Telefone: (11) 3259-5005
www.hexag.com.br
contato@hexag.com.br

CARO ALUNO,
O Hexag Medicina referncia em preparao pr-vestibular de candidatos carreira de Medicina. Desde 2010,
so centenas de aprovaes nos principais vestibulares de Medicina no Estado de So Paulo e em todo Brasil.
Ao atualizar sua coleo de livros para 2016, o Hexag considerou o principal diferencial em relao aos
concorrentes: a sua exclusiva metodologia fundamentada em trs pontos perodo integral, estudo orientado
(E.O.) e salas reduzidas.
O material didtico foi, mais uma vez, aperfeioado e seu contedo enriquecido, inclusive com questes
recentes dos principais vestibulares 2016.
Esteticamente, houve uma melhora em seu layout, na definio das imagens e tambm na utilizao de cores.
No total, so 80 livros, distribudos da seguinte forma:
21 livros de Cincias da Natureza e suas tecnologias (Biologia, Fsica e Qumica);
14 livros de Cincias Humanas e suas tecnologias (Histria e Geografia);
07 livros de Linguagens, Cdigos e suas tecnologias (Gramtica, Literatura e Ingls);
07 livros de Matemtica e suas tecnologias;
04 livros de Sociologia e Filosofia;
04 livros Entre Aspas (Obras Literrias da Fuvest e Unicamp);
02 livros Entre Frases (Estudo da Escrita Redao);
06 livros Entre Textos (Interpretao de Texto).
03 livros "Between English and Portuguese" (Ingls).
12 livros de Reviso (U.T.I. "Unidade Tcnica de Imerso").
O contedo dos livros foi organizado por aulas. Cada assunto contm uma rica teoria, que contempla de
forma objetiva o que o aluno realmente necessita assimilar para o seu xito nos principais vestibulares e Enem,
dispensando qualquer tipo de material alternativo complementar.
Os captulos foram finalizados com cinco categorias de exerccios, trabalhadas nas sesses de Estudo Orientado (E.O.), como segue:
E.O. Teste I: exerccios introdutrios de mltipla escolha, para iniciar o processo de fixao da matria
estudada em aula;
E.O. Teste II: exerccios de mltipla escolha, que apresentam grau mdio de dificuldade, buscando a consolidao do aprendizado;
E.O. Teste III: exerccios de mltipla escolha com alto grau de dificuldade;
E.O. Dissertativo: exerccios dissertativos nos moldes da segunda fase da Fuvest, Unifesp, Unicamp e
outros importantes vestibulares;
E.O. Enem: exerccios que abordam a aplicao de conhecimentos em situaes do cotidiano, preparando
o aluno para esse tipo de exame.
A edio 2016 foi elaborada com muito empenho e dedicao, oferecendo ao aluno um material moderno e
completo, um grande aliado para o seu sucesso nos vestibulares mais concorridos de Medicina.
Herlan Fellini

BETWEEN ENGLISH
AND PORTUGUESE
Class 1: Overview and Analysis 6
Class 2: Who is who? 20
Class 3: The here and now? 32
Class 4: Focus on the past 46
Class 5: Looking forward! 70
Class 6: Text (Do you understand? Text interpretations)

82

Class 7: Should we continue? 92


Class 8: In this chaoter 122

Mihai Simonia/Shutterstock

Class 1
Overview and Analysis

Introduo
A misso desse material no nada modesta: ser um instrumento especfico e eficaz para a preparao do estudante do Hexag para as provas brasileiras de vestibular para medicina. No se trata de uma misso modesta
devido grande variedade de vestibulares que existem no Brasil, alguns muito conhecidos (USP, UNICAMP, UNESP,
UNIFESP, ENEM) e outros muito menos conhecidos, porm to importantes quanto os mais famosos.
Por ser um instrumento especfico de preparao, muito dos contedos que os estudantes brasileiros normalmente aprendem relacionados lngua inglesa no sero contemplados nesse material. Habilidades como
listening (compreenso auditiva) e speaking (comunicao oral) e at mesmo writing (escrita) no so essenciais
para seu processo de aprendizagem e por isso no sero estudados aqui. Mesmo a habilidade de leitura e compreenso de textos (reading) estar adaptada para o ambiente especfico que composto pelas provas de lngua
inglesa dos vestibulares brasileiros.
Sendo assim... lets get ready for them!

Tipos de provas
Existem dois tipos muito difundidos de provas vestibulares no Brasil: as provas compostas por questes de mltipla
escolha (a grande maioria) e as provas compostas por questes abertas (dissertativas). Alm desses dois tipos, no
to incomum encontrar provas de somatria e provas com questes do tipo certo/errado.

Provas de mltipla escolha


A grande maioria dos processos vestibulares brasileiros apresenta questes de mltipla escolha, sendo que elas
podem exigir habilidades de leitura e compreenso de texto, vocabulrio ou gramtica. Alm disso, os enunciados
e alternativas dessas questes podem ser apresentados em portugus ou ingls.
D uma olhada nos dois exemplos abaixo.
1. (FUVEST) English is a colonial language that continued to be the official language after independence in
virtually all African countries that were under British rule. In some cases it was retained to avoid ethnic tensions. But in all cases it was retained because of its prestige and association with power.
Which of these statements is true according to the text?
a) As compared to English, African languages are inferior, poor and underdeveloped.
b) English has a greater number of rules than most African languages.
c) In former British colonies in Africa, the English language was adopted because of its prestige and power.
d) Using vernacular languages in Africa was a way of maintaining peace among different ethnic groups.
e) Adopting English as an official language in some African countries might result in a stimulus for the
development of vernacular languages.
2. (UNIFESP) WHO established the external Child Health Epidemiology Reference Group (CHERG) in 2001 to
develop estimates of the proportion of deaths in children younger than age 5 years attributable to pneumonia, diarrhoea, malaria, measles, and the major causes of death in the first 28 days of life.
O grupo Cherg:
a) trabalhou de 2001 a 2005 para estabelecer o papel da desnutrio como a principal causa da mortalidade infantil.
b) desenvolveu estimativas a respeito das principais causas de mortalidade em crianas nos primeiros 28
dias de vida.
c) concluiu que a desnutrio pode ser uma das causas de mortalidade infantil at os cinco anos de idade.
7

d) foi formado por integrantes da Organizao Mundial de Sade em 2001 e trabalhou at 2003.
e) descobriu que as principais causas de mortalidade em crianas de at 28 dias so pneumonia, diarreia,
malria e sarampo.
Cada vez menos frequentes, mas ainda existentes, so as questes de vocabulrio e de gramtica. Estude os dois exemplos abaixo.
3. (UNESP) Assinale a alternativa na qual todas as palavras so formas verbais relativas ao passado.
a) Adopted, become, decided, recognized, ruled.
b) Adopted, allow, become, recognized, ruled.
c) Approved, became, been, decided, ruled.
d) Allow, approved, became, decided, may.
e) Can, debated, entitled, made, offered.

Provas Dissertativas
As trs principais instituies de ensino estaduais paulistas (USP, UNICAMP, UNESP) exigem de seus candidatos a
resoluo de questes dissertativas. Esse tipo de questo tambm pode ser encontrado em vestibulares em outros
estados da federao. No caso paulista, as respostas devem ser elaboradas em PORTUGUS.
Observe o exemplo a seguir:
(UNICAMP)
Leia o texto abaixo e responda questo 1
My name is Joe. That is what my colleague, Milton Davidson. He is a programmer and I am a computer. I am
Miltons experimental model. His Joe. Milton has never married, though he is nearly 40 years old. He has
never found the right woman, he told me. One day he said, Ill find her yet, Joe. Im going to find the best.
Im going to have true love and youre going to help me. Im tired of improving you in order to solve the
problems of the world. Solve my problem.
Find me true love.
1.
a) Do que Milton Davidson est cansado?
b) Por que Milton Davidson no se casou e o que ele espera que Joe faa por ele?
Resoluo:
a) De acordo com as informaes apresentadas pelo texto, Milton Davidson est cansado de melhorar seu
modelo experimental de computador, cujo nome Joe, para resolver os problemas do mundo.
b) Segundo o texto, Milton Davidson no se casou porque ele ainda no encontrou a melhor parceira possvel e ele espera que seu computador (Joe) ajude-o a encontrar essa mulher ideal e que significa para
ele o amor verdadeiro.

Provas de somatria
(UFSC) Read the text and answer.
Introducing Cordel

Brazil's "literatura de cordel" is a kind of folk-popular poetry ______ involves both the oral and written traditions and is very popular in northeastern Brazil. After a hiatus of ______ years when its production fell ______
because of economic and social change in Brazil, it is ______ a revival due primarily to the personal computer and
printer which allow poets to ______ the high cost of typographies and printing shops. In addition, there is a large
______ of "cordel" type poetry on the internet.
Adapted from: <http://www.currancordelconnection.com/en/what-is-cordel>. Accessed on August 17th, 2012.

1. Choose the CORRECT proposition(s) to complete the text above.


01) what many chiefly transforming refuse occurrence
02) which some significantly experiencing avoid presence
04) there various largely renovating decline attendance
08) who few extensively increasing change existence
16) that several considerably undergoing escape incidence

Provas do tipo Certo/Errado


(UNB) Jan or Johannes Vermeer van Delft (16321675), a Dutch genre painter who lived and worked in Delft all his
life, created some of the most exquisite paintings in Western art.
His works are rare. Of the 35 or 36 paintings generally attributed to him, most portray figures in interiors. All his
works are admired for the sensitivity with which he rendered effects of light and color and for the poetic quality of
his images. He produced meticulously constructed interiors with just one or two figures usually women. These
are intimate genre paintings in which the principal figure is invariably engaged in some everyday activity. Often
the light enters Vermeers paintings from a window. He was a master at describe the way light illuminates objects.
1. Judge the items that follow according to the text above. (T/F)
( ) 1 It took around two centuries for Vermeers paintings to be attributed to him again.
( ) 2 Some of Vermeers paintings are considered strange.
( ) 3 Even though there were just a few of them, Vermeers paintings proved to be very influential in the
history of Dutch painting.
9

E.O. Teste I
TEXTO PARA A PRXIMA QUESTO
Social Network for Good
By Matt Shaw on April 26, 2011 in Blog

On Saturday my wife Lana was out for coffee


with a friend. She saw an older lady holding
a please help sign. After talking with her
for a few minutes and finding out some of
her needs, Lana called me. This sweet lady,
who not long ago lost her job and then her
apartment, wanted some personal hygiene
items and a tent.
I posted a request for the tent on Twitter
and Facebook. Immediately it was retweeted
by others on Twitter. On Facebook, it was reposted by others, including Charlotte 24-7.
One of their followers spotted the post, contacted 24-7, who contacted me, and I called
the donor. He not only had a tent to give, but
wanted to drive 50 miles to deliver it. Giving
someone a tent is not an answer or solution
to homelessness. Its a band-aid. However,
it meets a very basic and immediate need.
Have you ever camped in a rainstorm? (I
have.) Now imagine doing it without a tent
nightly. We cant disparage the importance
of this kind of help. The lady who received
it was ecstatic.
Helping like this doesnt have to be painful
its you and I using our connections and
sharing the needs. I cant help every person,
neither can you, but when we speak up, it
gives others a chance to get involved.
Available at: <http://www.speakupmag.
org/2011/04/ social-network-forgood/>.

1. (UPF) A alternativa que melhor resume o


texto :
a) Por meio de um fato acontecido com sua esposa, o autor explica como as redes sociais
podem auxiliar a encontrar outras pessoas.
b) Por meio do relato de um fato incomum
do dia a dia, o autor ilustra como as redes
sociais podem ser usadas para ajudar outras pessoas.
c) O autor relata como sua mulher ajudou outras pessoas por meio das redes sociais.
d) O autor apresenta argumentos para convencer os leitores de seu blog a no usar as redes sociais para serem solidrios, pois pode
ser perigoso.
e) O autor e sua esposa comentam como pessoas necessitadas podem ser ajudadas por
meio das redes sociais.
10

TEXTO PARA A PRXIMA QUESTO


Tomorrows world
This text is going to be a little different.
Its about predictions. What is going to happen tomorrow? What will the future bring?
Things like that. Optimists and pessimists
have their own answers. We are going to
know what they are. Then you are going
to decide which group (optimists or pessimists) made each prediction. You are also
going to give your own opinion about each
prediction (whether you consider it possible
or impossible to happen).
The City of the Future will have a roof - a
huge geodesic dome that will cover the buildings and population.
Man will invent a kind of machine that
will be able to think.
Well be able to go to the Moon and to the
planets. Scientists will live and work in space colonies.
The population of the world will exceed 10
billion people before the beginning of the
next century.
There will be wars in every part of the
world.
Medical science will find a cure for several
different diseases before the year 2000.
Well have a lot of free time because computers will do much of our work.
Nuclear energy will be safe. There wont be
any danger of accidents.
Noise and air pollution will belong to the
past.
Big cities will continue to grow and there wont be enough food for everybody. How
many of these predictions do you consider
possible? Are you an optimist or a pessimist?
2. (Faap) De acordo com o texto, na cidade
do Futuro:
a) haver um telhado um enorme domo geodsico que cobrir os prdios e a populao.
b) haver somente prdios, todos em cores padronizadas.
c) toda a cidade ser coberta por uma lona sinttica para proteg-la do mau tempo.
d) cada prdio ter uma cobertura especial.
e) existir uma capa protetora sobre os edifcios comerciais.
TEXTO PARA A PRXIMA QUESTO
Your Time Their Future
Frank could have been the average American teenager. But with profound family,
drug and behavioral problems, and almost
completely without resources, Frank was
anything but average. Hed drifted into an
alternative school for troubled youths, where,
despite special classes, Franks principal still

considered him the worst kid of the whole


lot. Clearly, Frank was in a downward spiral
with little hope of reversing the direction.
However, Elizabeth Mller, program coordinator at the Freeport Youth Outreach Center, refused to give up on the teenager. She matched
him with a mentor, Dr. Lawrence Brennan, who
found Frank an after-school job. More
importantly, Dr. Brennan promised to meet
with Frank for a few hours every week to talk
and relax, and eventually to become a friend
and confidant.
The results were powerful. In one semester,
Frank switched from the alternative school to
a regular junior high school and became an
honor roll student. Since then he has never
missed a class, mentor meeting or day of work.
All he needed was to be given some purpose
and direction, Brennan says.
Encarte do Readers Digest, October 1998.

3. (FEI) FRANK, segundo o texto:


a) nunca teve maiores problemas psicolgicos.
b) era de famlia sem recursos.
c) era um garoto americano tpico.
d) pertencia classe mdia.
e) no precisou frequentar escolas especiais.
4. (Uern adaptada) Leia o anncio abaixo.

5. (PUC adaptada) Leia a citao abaixo:


I find television very educating. Every time
somebody turns on the set, I go into the
other room and read a book.
Groucho Marx (1890-1977)

O comediante estadunidense Groucho Marx


fez um comentrio que enfatiza que a TV:
a) ensina as pessoas a ler livros de forma silenciosa.
b) leva-o a ler um livro toda vez que ela ligada.
c) exibe muitos comerciais de livros e outros
produtos culturais.
d) exibe alguns programas que encorajam hbitos de leitura.
e) exibe muitos programas com contedo educativo.
TEXTO PARA AS PRXIMAS 3 QUESTES
Japanese researchers have spent five years
developing a humanoid robot system that
can understand and respond to simultaneous speakers. They posit a restaurant scenario in which the robot is a waiter. When
three people stand before the robot and
simultaneously order pork cutlet meals or
French dinners, the robot understands at
about 70 percent comprehension, responding by repeating each order and giving
the total price. This process takes less than
two seconds and, crucially, requires no prior
voice training. Such auditory powers mark
a fundamental challenge in artificial intelligence how to teach machines to pick out
significant sounds amid the hubbub. This is
known as the cocktail party effect, and most
machines do not do any better than humans
who have had a few too many martinis.
Scientific American. August 2007.

Anncios publicitrios so utilizados para


promover ideias, conceitos e produtos. tpico da linguagem publicitria a utilizao
de variados recursos para atingir seus objetivos. De acordo com informao contida no
anncio acima, possvel:
a) aprender ingls sem custos.
b) aprender ingls on-line.
c) aprender ingls facilmente.
d) aprender ingls em casa.
e) aprender qualquer lngua.
11

6. (Fuvest) De acordo com o texto, pesquisadores japoneses desenvolveram um rob que:


a) responde a perguntas e d conselhos s pessoas, em poucos segundos.
b) recebe treinamento para produzir fala inteligvel.
c) consegue adivinhar os desejos das pessoas.
d) tem capacidade de interagir com interlocutores diferentes ao mesmo tempo.
e) interage prontamente com qualquer pessoa,
em vrias lnguas.
7. (Fuvest) Segundo o texto, num cenrio de
restaurante, o rob:
a) serve a mesa e escolhe os pratos para
os clientes.
b) anota pedidos e prepara a comida.
c) organiza o funcionamento do restaurante.
d) l o cardpio em voz alta e d sugesto
de pratos.
e) repete os pedidos feitos e informa o valor
da refeio.
8. (Fuvest) O texto refere-se a the cocktail
party effect, que metfora para descrever
a dificuldade de:
a) selecionar sons relevantes, a que prestar
ateno, em locais com muita gente.
b) escolher pessoas importantes com quem
conversar em festas.
c) circular e se aproximar de pessoas em coquetis.
d) beber s um pouco para conseguir interagir
com outros convidados.
e) conversar casualmente sobre diversos temas.

9. (Fuvest) O estudo mencionado no texto:


a) avaliou a influncia de nveis de hostilidade
em indivduos deprimidos.
b) baseou-se em diferentes tipos de exames
mdicos e respostas a questionrios.
c) foi importante para controlar os nveis da
protena C3 no sangue de soldados com presso arterial elevada.
d) foi desenvolvido para testar a relao entre
presso arterial e diabetes.
e) concentrou-se na anlise de manifestaes
depressivas em veteranos do Vietn.
10. (Fuvest) Os resultados do estudo mencionado no texto sugerem que:
a) os nveis da protena C3 diminuem em homens que passaram por experincias traumticas de guerra.
b) nveis altos de hostilidade so potencial
ameaa sade mental.
c) doenas cardiovasculares podem estar relacionadas aos nveis de hostilidade, raiva e
depresso.
d) baixa ingesto de protenas pode causar raiva e depresso, estimulando comportamentos hostis.
e) os veteranos do Vietn analisados estavam
menos deprimidos que o esperado.

E.O. Teste II
Instruo: Leia a histria em quadrinhos
de Archie, What goes up, para responder s
questes de nmeros 1 e 2.

TEXTO PARA AS PRXIMAS 2 QUESTES


Researchers studying 313 healthy Vietnam
veterans have found that anger, depression
and hostility may increase the risk for cardiovascular disease, diabetes and high blood
pressure. Over a period of ten years, the
men had regular physical examinations involving a wide variety of medical tests. They
also underwent psychological examinations
using well-established questionnaires to determine their levels of hostility, anger and
depression. The researchers measured blood
levels of a protein called C3, a marker for the
inflammation that is a risk factor for cardiovascular illnesses. After controlling for other
variables, the scientists found that those in
the highest one-quarter in hostility, anger
and depression showed a steady and significant increase in C3 levels, while those in the
lowest one-quarter had no increase.
<www.nytimes.com>. August 14, 2007. Adaptado.

12

Tales fromRiverdale. Archie Comic Publications. Dez. 2006.

1. (Unesp) Qual a alternativa correta?


a) O pai de Archie est nervoso porque recebeu
uma longa carta de Bill Going contendo notcias desagradveis sobre Archie.
b) O pai de Archie, que motorista de txi,
est furioso porque o conserto de seu carro
custou muito caro.
c) O pai de Archie est nervoso porque as contas que ele tem que pagar esto ficando muito caras.
d) O pai de Archie pediu para ver as notas escolares do filho e descobriu que Archie obteve
notas baixas.
e) O pai de Archie gostaria de no tomar conhecimento sobre uma reportagem que
Archie escreveu.
2. (Unesp) Qual o significado, na histria, dos
termos cable bill, a good time e report card,
respectivamente?
a) Cabo eltrico, uma boa hora e carto de reprter.
b) Conta da televiso a cabo, uma boa hora e
carto de reprter.
c) Cabo eltrico, um bom tempo e carto de
reprter.
d) Conta da televiso a cabo, uma boa hora e
boletim escolar.
e) Cabo eltrico, um bom tempo e boletim
escolar.
Instruo: Leia o texto a seguir para responder as questes de nmeros 3 a 4.
Two of the greatest obstacles that comics have
in reaching readers are exposure and cost.
Fortunately, the internet has provided remedies for both. Many comic book creators and
publishers have put their comics online, available as full issues and at absolutely no cost
to the reader. And unlike torrents or scanned
files, these comics are completely legal.
Here I have endeavored to collect as many
of these as possible, now totalling over 300
full issues and stories, in one place. Whether you have been meaning to try a new
title, or if youve never read a comic in your
life, theres still something here for everyone. Follow a link or two or three. Some comics that I especially recommend carry an
asterisk, but I havent come close to reading
everything here. Maybe youll find something you enjoy.
Adaptado de: <www.lorencollins.net/freecomic>.

3. (Unesp) Qual o melhor ttulo para o texto?


a) My costly comic book collection
b) Remedy obstacles in comic books
c) 300 recommended funny stories
d) Enjoy funny books online
e) Free online comic books

4. (Unesp) De acordo com o texto, dois obstculos ao acesso dos leitores s histrias em
quadrinhos so:
a) a divulgao e o material escaneado da
internet.
b) o custo da internet e a legalidade de material escaneado.
c) a legalidade de material escaneado e de edies completas.
d) o desconhecimento e o desinteresse por histrias em quadrinhos.
e) o custo e a divulgao das histrias ao pblico.
Instruo: Leia o texto Status of same-sex
marriage para responder s questes de nmeros 5 a 7.
Status of same-sex marriage
South America
Argentina
The Autonomous City of Buenos Aires (a federal district and capital city of the republic) allows same-sex civil unions.
The province of Rio Negro allows same-sex
civil unions, too.
Legislation to enact same-sex marriage
across all of Argentina was approved on July
15, 2010.
Brazil
A law that would allow same-sex civil unions
throughout the nation has been debated.
Until the end of the first semester of 2010
the Supremo Tribunal Federal had not decided about it.
Colombia
The Colombian Constitutional Court ruled in
February 2007 that same-sex couples are entitled to the same inheritance rights as heterosexuals in common-law marriages. This ruling
made Colombia the first South American nation
to legally recognize gay couples. Furthermore,
in January 2009, the Court ruled that same-sex
couples must be extended all of the rights offered to cohabitating heterosexual couples.
Ecuador
The Ecuadorian new constitution has made
Ecuador stand out in the region. Ecuador has
become the first country in South America
where same-sex civil union couples are legally recognized as a family and share the
same rights of married heterosexual couples.
Uruguay
Uruguay became the first country in South
America to allow civil unions (for both opposite-sex and same-sex couples) in a national
platform on January 1, 2008. Children can
be adopted by same-sex couples since 2009.
Adaptado de: <http://en.wikipedia.org>.

13

5. (Unesp) Assinale a alternativa correta.


a) Segundo o texto, os pases nos quais os direitos de casais heterossexuais e de casais
homossexuais so os mesmos so o Equador
e a Colmbia.
b) De acordo com as informaes do texto, entende-se que unies civis entre pessoas do
mesmo sexo so legais em todos os pases da
Amrica do Sul.
c) De acordo com o texto, entende-se que, dentre os pases da Amrica do Sul, somente no
Brasil ainda no se permitem unies civis
entre pessoas do mesmo sexo.
d) O pas da Amrica do Sul onde as unies civis entre pessoas do mesmo sexo demoraram
mais para ser legalizadas o Uruguai.
e) As unies civis entre pessoas do mesmo
sexo, vlidas em todo o territrio brasileiro,
foram aprovadas em 2010 pelo Supremo Tribunal Federal.
6. (Unesp) Assinale a alternativa correta.
a) A Colmbia foi a ltima nao sul-americana
a aprovar a unio civil de casais htero ou
homossexuais.
b) A Argentina foi a segunda nao sul-americana a reconhecer os direitos dos casais do
mesmo sexo.
c) O Equador foi o pas sul-americano que menos se empenhou para reconhecer os direitos dos homossexuais.
d) O Uruguai foi o primeiro pas sul-americano
a aprovar unies civis de casais htero e homossexuais.
e) O Brasil no tem demonstrado nenhum interesse no reconhecimento dos direitos dos
casais homossexuais.
7. (Unesp) Com base nas informaes do texto,
o que podemos inferir a respeito da situao
atual dos casais do mesmo sexo na Argentina?
a) As unies civis entre pessoas do mesmo sexo
so vlidas somente em Buenos Aires e na
provncia de Rio Negro.
b) Os casais do mesmo sexo provavelmente ainda no tm todos os direitos dos casais heterossexuais.
c) A provncia de Rio Negro foi a regio onde
unies civis entre pessoas do mesmo sexo
foram aprovadas mais recentemente.
d) Em Buenos Aires, as leis para unies civis
entre pessoas do mesmo sexo so diferentes
do restante do pas.
e) Os casais homossexuais podero ter exatamente os mesmos deveres dos casais heterossexuais.

14

E.O. Teste III


1. (UFMS) Considerando o texto abaixo, correto afirmar:

Nobody ever smiles around here, so nobody will ever see your teeth. Thats our dental plan.
<http://www.glasbergen.com/images/sun.gif>

Na questo a seguir, determine a soma dos


itens corretos.
Os funcionrios da empresa para a qual
01)
a jovem est se candidatando a emprego
so cegos.
02) A empresa para a qual a jovem est se candidatando a emprego no possui plano odontolgico.
04) O ambiente, na empresa para a qual a jovem
est se candidatando a emprego, no alegre.
08) A entrevistadora estimula a vinda da nova
funcionria empresa.
16) As duas mulheres da ilustrao esto conversando sobre dentistas.
TEXTO PARA A PRXIMA QUESTO
There are many misconceptions about what
most biologists understand by evolution. By
definition, describing any animal as primitive is not the same as saying that it has
not undergone the same amount of adaptive
change as everything else. Coelacanths are
certainly very much like some fossils, but
that does not mean that they have stopped
evolving. In much the same way, modern
crocodiles are very similar to fossil crocodiles. In both cases we can see that these
animals are supremely adapted to their environments, but these environments have
not changed recently and nor have the animals. Mantis shrimps, as a group of animals,
are twice as old as the dinosaurs. The dinosaurs environment changed too rapidly for
them to adapt (maybe literally overnight if
asteroid-impact theories are correct), but
the mantis shrimps have obviously been
able to cope with changes. Everything alive
today is equally modern, and when biologists describe a creature as primitive they
mean simply that it does not appear to have

changed much recently. Fossils only give information about the harder parts of animals
that existed in the past. Nothing about the
physiology or behaviour of deceased animals
is preserved in the rocks.
Adaptado de: <fishinsects.suite101.com>.

2. According to the text:


I. a primitive animal is one which has not
undergone the same e volutionary process that evolved animals went through.
II. although coelacanths and crocodiles are
very similar to their fossilized ancestors,
it is correct to say that they have evolved.
III. the evolution of animals is determined
by the changes in their respective habitats.
IV. dinosaurs are as old as mantis shrimps.
V. all living species will succeed in surviving forever.
Est correta apenas a afirmao:
a) I.
b) II.
c) III.
d) IV.
e) V.
TEXTO PARA AS PRXIMAS 4 QUESTES
Motion picture is one of the almost popular
forms of art and entertainment throughout
the world. Every week, millions of people
go to the movies. Many millions more watch movies in television. In addition, TV
networks use motion picture techniques to
film many program that appear on television
each week.
Motion pictures are a major source of information as well as of entertainment. Movies
can take us back into history. They can recreate the lives of great men and women.
Motion pictures can introduce us to new ideas and help us explore serious social issues.
Students learn from educational films in
school. Industries use movies to train employees and to advertise their products.
issues = questes
(THE WORLD BOOK ENCYCLOPEDIA. Vol.13.
Chicago, World Book Inc., 1988)

Na(s) questo(es) a seguir, escreva no espao apropriado a soma dos itens corretos.

08) in certain parts of the world


16) in every part of the world
32) in the whole world
4. (UFPR) In which of the following sentences
is the verb used correctly?
01) Does he enjoys working in films?
02) A film star is a well-known cinema actor or
actress.
04) Television competes directly with the cinema.
08) It dont take a long time to make a film.
16) Are you interested in the cinema?
32) People sometimes become emotionally involved with movie stars.
5. (UFPR) __________ of our history and the
lives of __________ of our great men and women are recreated by the movies.
Choose the alternative(s) that can complete
the sentence above correctly.
01) Much - much
02) Much - many
04) Many - much
08) A lot - a lot
16) A lot - much
32) A lot - many
6. (UFPR) Based on the text it is correct to say
that:
01) Many people go to the movies.
02) The cinema is a very popular form of art.
04) Some films deal with the lives of famous people.
08) Some TV programs are made using motio picture techniques.
16) Some industries allow their employees to go
to the movies.
32) The only objective of a film is to entertain
people.

E.O. Dissertativo
Responda s
PORTUGUS

perguntas

seguir

em

Did Charles Darwin Delay in Publishing Origins of Species?

3. (UFPR) Motion picture is one of the most


popular forms of art and entertainment
throughout the world.
Which expressions can replace THROUGHOUT
THE WORLD in the sentence above?
01) except the world
02) although the world
04) all over the world
15

Charles Robert Darwin (1809-1882) began


working on his theories of the Origins of
Species in 1837; however his works were published more than twenty years after that.
There is much speculation as to why it took
so long to publish the groundbreaking book;
some suggest that he was afraid to challenge the scientific community and upset the
Church. After first positing his ideas regarding how species underwent a natural selection and could possibly adapt over time,
the text provoked adverted reactions from
his intellectual mentors, Charles Lyell and
Sir John Herschel. Still, Darwin continued
working on his theory; if he really was affected by the potential of a negative response
to his ideas, some believe that it seems more
plausible that he would have abandoned research completely. Throughout the period
during which he was working on the Origins
of Species, he published essays revealing
his work in progress. Many now believe that
Darwin did not delay publishing, but rather,
took over two decades to complete his work.
Adaptado de: <http://www.suite101.com/greatthinkers/4>.

1. (Unicamp) Que hiptese levantada por


algumas pessoas para justificar o fato de
Darwin ter demorado mais de duas dcadas
para publicar Origins of Species?
2. (Unicamp) Indique duas evidncias que contrariam essa hiptese.
Texto
In the latest move to inflame the racially
tinged issue ahead of Novembers congressional they intend to call hearings on overturning the 14th amendment to the constitution, which grants citizenship to anyone
born in the US. Leading Republicans have
denounced the provision as outdated, saying
it encourages invasion by birth canal in
which illegal immigrants smuggle themselves into the US to have anchor babies.
The change is being pushed by the Republican whip in the Senate, John Kyl, and
senator Lindsey Graham, who said that
birthright citizenship is a mistake. The
14th amendment was adopted in 1868 after
the civil war to block laws that prevented
former slaves from becoming US citizens.
Reform must be approved by two-thirds
of both houses of Congress and ratified by
three-quarters of US states or by calling a
convention by the states.
Guardian.co.uk. 3 August 2010. Adaptado.

Baseando-se nas informaes fornecidas


pelo texto, responda s questes a seguir:
16

3. (Fuvest) O que a 14 emenda Constituio


dos Estados Unidos assegura e por que ela
foi adotada?
4. (Fuvest) Qual a questo polmica apresentada no texto com relao aos imigrantes?
TEXTO PARA A PRXIMA QUESTO
Film about de Menezes premieres in home
town
By Jan Onoszko in Rio de Janeiro Friday, 19 June 2009

The life story of the Brazilian man shot dead


by police on a London Underground train
because they believed he was a suicide bomber is celebrated in a film which premieres
in his home town this evening.
The population of Gonzaga is expected to double in size as 10,000 people pack the towns
football ground for the first screening of the
film, entitled Jean Charles. Jean Charles de
Menezes was 27 years old when Metropolitan Police officers fired seven bullets into
his head at Stockwell Tube station on 22 July
2005. The force was found guilty of endangering public safety in a subsequent inquiry into
the incident but no individual officers have
been held accountable for his death.
The Gonzaga mayor, Eugenia-Maria Magalhes, said: We wish the town could have
become known for other reasons, if it had to
be known at all. What happened still has a
profound effect on all of us. Theres a lot of
indignation, pain, sadness, and Jean Charles is greatly missed. He was an ordinary
boy who left us in search of a better life.
The BBC commissioned the film and approached Henrique Goldman to direct and write
it, but it later pulled out of the project because they didnt agree on what perspective the film should take. I dont know why
they pulled the plug, said Goldman. He managed to keep the project going when the
UK Film Council provided half the funding.
The Government which lets the police get
away with murder also allows us to make
the film, said Goldman. This schizophrenic behaviour is very British.
<www.independent.co.uk/arts-entertainment/films>

5. (Unesp) De que fonte foram efetivamente


obtidos recursos para financiar a produo
do filme? Essa fonte de carter pblico ou
privado? Qual a porcentagem desse apoio financeiro em relao ao total de gastos?

E.O. Enem

Disponvel em: http://www.chris-alexander.


co.uk/1191. Acesso em: 28 jul. 2010 (adaptado).

1. Criadas pelos pases membros da Organizao


das Naes Unidas e por organizaes internacionais, as metas de desenvolvimento do milnio envolvem oito objetivos a serem alcanados
at 2015. Apesar da diversidade cultural, esses
objetivos, mostrados na imagem, so comuns
ao mundo todo, sendo dois deles:
a) O combate AIDS e a melhoria do ensino
universitrio.
b) A reduo da mortalidade adulta e a criao
de parcerias globais.
c) A promoo da igualdade de gneros e a erradicao da pobreza.
d) A parceria global para o desenvolvimento e a
valorizao das crianas.
e) A garantia da sustentabilidade ambiental e
combate ao trabalho infantil.
2. Os cartes-postais costumam ser utilizados
por viajantes que desejam enviar notcias
dos lugares que visitam a parentes e amigos. Publicado no site do projeto ANDRILL,
o texto em formato de carto-postal tem o
propsito de:

TEXTO PARA A PRXIMA QUESTO


I used to rule the world
Seas would rise when I gave the word
Now in the morning and I sleep alone
Sweep the streets I used to own
I used to roll the dice
Feel the fear in my enemys eyes
Listen as the crowd would sing Now the old
king is dead! Long live the king!
One minute I held the key
Next the walls were closed on me
And I discovered that my castles stand
Upon pillars of salt and pillars of sand []
MARTIN, C. Viva la vida, Coldplay. In: Viva la vida
or Death and all his friends. Parlophone, 2008.

3. Letras de msicas abordam temas que, de


certa forma, podem ser reforados pela repetio de trechos ou palavras. O fragmento
da cano Viva la vida, por exemplo, permite
conhecer o relato de algum que:
a) costumava ter o mundo aos seus ps e, de
repente, se viu sem nada.
b) almeja o ttulo de rei e, por ele, tem enfrentado inmeros inimigos.
c) causa pouco temor a seus inimigos, embora
tenha muito poder.
d) limpava as ruas e, com seu esforo, tornou-se rei de seu povo.
e) tinha a chave para todos os castelos nos
quais desejava morar.
TEXTO PARA A PRXIMA QUESTO
THE WEATHER MAN
They say that the British love talking about
the weather. For other nationalities this can
be a banal and boring subject of conversation, something that people talk about when
they have nothing else to say to each other.
And yet the weather is a very important
part of our lives. That at least is the opinion of Barry Gromett, press officer for The
Met Office. This is located in Exeter, a pretty
cathedral city in the southwest of England.
Here employees and computers supply
weather forecasts for much of the world.
Speak Up. Ano XXIII, n 275.

a) comunicar o endereo da nova sede do projeto nos Estados Unidos.


b) convidar colecionadores de cartes-postais a
se reunirem em um evento.
c) anunciar uma nova coleo de selos para angariar fundos para a Antrtica.
d) divulgar s pessoas a possibilidade de receberem um carto-postal da Antrtica.
e) solicitar que as pessoas visitem o site do
mencionado projeto com maior frequncia.

4. Ao conversar sobre a previso do tempo, o


texto mostra:
a) o aborrecimento do cidado britnico ao falar sobre banalidades.
b) a falta de ter o que falar em situaes de
avaliao de lnguas.
c) a importncia de se entender sobre meteorologia para falar ingls.
d) as diferenas e as particularidades culturais
no uso de uma lngua.
e) o conflito entre diferentes ideias e opinies
ao se comunicar em ingls.
17

TEXTO PARA A PRXIMA QUESTO

E.O. Teste III


1. 02 + 04 = 06
2. B
3. 04 + 16 + 32 = 52
4. 02 + 04 + 16 + 32 = 54
5. 02 + 08 + 32 = 42
6. 01 + 02 + 04 + 08 = 15

E.O. Dissertativo

5. Ao optar por ler a reportagem completa sobre o assunto anunciado, tem-se acesso a
duas palavras que Bill Gates no quer que o
leitor conhea e que se referem:
a) aos responsveis pela divulgao desta informao na internet.
b) s marcas mais importantes de microcomputadores do mercado.
c) aos nomes dos americanos que inventaram a
suposta tecnologia.
d) aos sites da internet pelos quais o produto j
pode ser conhecido.
e) s empresas que levam vantagem para serem
suas concorrentes.

Gabarito
E.O. Teste I
1. B

2. A

3. B

4. C

5. B

6. D

7. E

8. A

9. B

10. C

E.O. Enem

E.O. Teste II

18

1. C

2. D

6. E

7. C

3. E

1.
Especula-se que o atraso para a publicao
de A origem das espcies tenha ocorrido
devido ao temor que Darwin teria de desafiar a comunidade cientfica, alm de temer
reaes adversas entre os eclesisticos.
2.
A hiptese mencionada anteriormente pode
ser contrariada pelo fato de que ele continuou publicando papers sobre a evoluo de
seu trabalho, mesmo aps sua divulgao
inicial. Se ele realmente estivesse preocupado com reaes negativas, ele teria interrompido completamente sua pesquisa, algo
que de fato nunca aconteceu.
3.
A 14 emenda constituio dos Estados
Unidos da Amrica assegura cidadania estadunidense a qualquer pessoa nascida em seu
territrio. Essa emenda foi adotada para evitar a adoo de leis que impedissem antigos
escravos de se tornarem cidados daquele
pas.
4.
A 14 emenda est sendo utilizada para burlar as leis de imigrao dos EUA. Casais entram ilegalmente no pas para terem bebs
que tero assim a cidadania estadunidense e
serviro como uma espcie de ncora que
os manteria legalmente nos EUA.
5.
A princpio, os recursos foram fornecidos
pela BBC, que porteriormente retirou-se do
projeto. Efetivamente, metade do dinheiro
para o filme foi obtido com o UK Film Council (Conselho Britnico de Cinema, em traduo livre), que uma fonte pblica, conforme evidenciado pela fala de Goldman The
government that lets the police... also allows
us to make the film. (O mesmo governo que
permite que a polcia... nos permite fazer o
filme.)

1. C
4. E

5. A

2. D

3. A

4. D

5. E

19

Class 2

Who is who?

lukeruk/Shutterstock

As unidades que trataro de gramtica foram escritas pensando nas dificuldades e facilidades mais frequentes do
vestibulando brasileiro. Apenas como sugesto, tente encarar essa abordagem como elemento facilitador do seu
aprendizado.

Personal pronouns
Subject Pronouns

Object Pronouns

ME

YOU

YOU

HE

HIM

SHE

HER

IT

IT

WE

US

YOU

YOU

THEY

THEM

Os personal pronouns da lngua inglesa podem, com algumas adaptaes, ser comparados aos pronomes pessoais
da lngua portuguesa. O que o portugus chama de pronomes pessoais do caso reto, a lngua inglesa chama de

subject pronouns, e os pronomes oblquos so anlogos aos object pronouns.


Observe alguns usos dos subject pronouns:
I am Brazilian Eu sou brasileiro
You are Brazilian Voc/vocs so brasileiros
He is Brazilian Ele brasileiro.
She is Brazilian Ela brasileira.
It is Brazilian Isto/Esta coisa brasileira
We are Brazilian Ns somos brasileiros.
They are Brazilian Elas/Eles so brasileiros(as).
De forma geral, a funo desta categoria de pronomes nas duas linguagens similar: pronomes substituem
nomes. No comum repetirmos o mesmo nome muitas vezes, mas sim substitu-lo por um pronome uma vez que
sabemos a quem tal pronome se refere.

Exemplos:
1. John loves Mary.
He loves Mary.
O exemplo acima faz a substituio do nome John pelo pronome He. Observe o exemplo a seguir
2. Mary loves John.
Mary loves him.
A diferena entre eles que no exemplo 1 o nome John tem funo de sujeito e foi substitudo por um
pronome de sujeito (He). J no exemplo 2, o nome John tem funo de objeto e foi substitudo por um pronome
de objeto (him).
21

Veja mais exemplos:


1. Brazilians agree with you. > We agree with you.
2. They agree with other Brazilians > They agree with them.
3. The prime minister will adress the issue > She/He will adress the issue.
Vamos observar alguns casos nos quais h grande semelhana entre o uso dos pronomes em lngua inglesa
e em lngua portuguesa. I, He, she e we no costumam apresentar grandes dificuldades de compreenso.
Entretanto, possvel observar problemas quando negociamos os significados dos seguintes pronomes para
o portugus.
You voc, vocs
They Eles, elas (ou eles e elas)
It refere-se a coisas, objetos, animais (sempre no singular) e fenmenos da natureza. Ex. chuva, dia, noite etc.
Vamos detalh-los.
O pronome you pode ser utilizado tanto no singular (voc) quanto no plural (vocs), e somente podem ser
diferenciados os usos dentro do contexto apresentado.

Exemplos:
1. You are here (voc est aqui) ou You are here (Vocs esto aqui)
O pronome they, assim como you, pode ter algumas interpretaes, dependendo do contexto.
2. They live here (Elas moram aqui/Eles moram aqui).
H ainda outra possibilidade: a de que o pronome they refira-se a um grupo formado por mulheres e homens.
3. They live here (Eles e elas moram aqui).
O caso do pronome it mais complexo. Ele tem a funo de substituir, quando no singular, coisas, animais,
objetos inanimados, sentimentos, planetas etc.

Exemplos:
1. The city of So Paulo is located in Brazil. > It is located in Brazil. (A cidade de So Paulo localizada no
Brasil.)
2. Pour the liquid in that jar. > Pour the liquid in it. (Coloque o lquido naquela jarra/nela.)

Notas importantes:
1. O pronome it s utilizado no singular. Quando nos referimos a, por exemplo, objetos no plural, o pronome
utilizado dever ser them.
Exemplos: I didnt really like the movie I didnt really like it (sing)
I didnt really like the movies I didnt really like them (plural)
2. O que a lngua portuguesa caracteriza como oraes sem sujeito, so expressadas em ingls pelo uso do
pronome it. Nesses casos importante observar que o pronome it no est se referindo a nada.
Exemplos: It is late ( tarde)
It rains a lot in London (Chove muito em Londres)
3. Algumas excees: Animais, quando se sabe o seu sexo, podem ser referidos como he/she. Bebs podem
tambm ter referncia pelo pronome it. Barcos (e s vezes meios de transportes grandes (nibus, trens)
podem ser referidos como it.
Exemplos: Your dog is fantastic. She/He is fantastic.
Dont be afraid and take my little baby. Dont be afraid and take it.
The USS Enterprise is approaching. She is approaching.
22

Possessive pronouns
Possessive Adjective Pronouns

Possessive Substantive Pronouns

MY

MINE

YOUR

YOURS

HIS

HIS

HER

HERS

ITS

-------

OUR

OURS

YOUR

YOURS

THEIR

THEIRS

Os pronomes desse tpico, como o prprio nome supe, introduzem uma relao de posse com um substantivo e
so bastante semelhantes ao que encontramos na lngua portuguesa. Mas aqui podemos enfrentar alguns problemas. Se por um lado o portugus apresenta marcao qudrupla de pronomes (Eu meu, minha, meus, minhas) e
o ingls apresente marcao simples para eles, este tem dois tipos de pronomes possessivos (Adjective e Substantive), enquanto aquele tem apenas um, o que pode gerar algumas dificuldades.
Veja os exemplos abaixo:
1. My cellphone is broken (Meu celular est quebrado)
2. My cellphones are broken (Meus celulares esto quebrados.)
Como podemos observar, o mesmo pronome my pode expressar singular ou plural. Observe mais exemplos.
1. My sister lives near here (Minha irm mora perto daqui)
2. My sisters live near here (Minhas irms moram perto daqui)
Assim como nos exemplos anteriores, possvel perceber que o mesmo pronome (my) tambm pode anteceder substantivos femininos singulares ou plurais, sem que seja necessrio alterar o prprio pronome.
Observe abaixo uma rodada completa dos possessive adjective pronouns.
My hope is big > Minha esperana grande.
Your hope is big > A sua/vossa esperana grande.
His hope is big. > A esperana dele grande.
Her hope is big. > A esperana dela grande.
Its hope is big.* > A esperana do Brasil grande.
Our hope is big. > A nossa esperana grande.
Your hope is big. > Vossas esperanas/A esperana de vocs grande.
Their hope is big. > A esperana deles/delas grande.
(*) Foi necessrio introduzir algum contexto neste exemplo. A esperana refere-se a um pas, o que no caso pediria o uso do pronome its.

Veja mais exemplos:


1. The school and its teachers are waiting for you. (A escola e seus professores(os professores da escola) esto
esperando por voc;
2. The clock was working. Its parts were fixed. (O relgio estava funcionando. As peas dele(as peas do relgio) foram consertadas.

23

Possessive Adjective ou Possessive Substantive?


Preste muita ateno ao que est sendo discutido agora. Em alguns momentos, pode ser pedido que o vestibulando
opte entre os dois tipos de pronomes acima mencionados. Alm de ser um tpico que gera uma dvida razovel,
ele tambm muito requisitado em questes gramaticais.
Tenha em mente que os pronomes Possessive Adjectives (my, your, his, her, its, our, their) so aqueles que
so normalmente usados. Voc s os substituir pelos Possessive Substantive (mine, yours, his, hers, ours, theirs)
em situaes especiais. So elas:
1. Quando o substantivo a que se referir o pronome estiver omitido/subentendido.
Exemplo: I cant find my phone. Can I use yours? (Eu no consigo encontrar meu telefone. Posso usar o
seu (telefone)?
2. Quando o substantivo a que se refere o pronome estiver antes do prprio pronome.
Exemplo: It belongs to a friend of mine. (Isto pertence a um amigo meu).
3. Aps preposies
Exemplo: They will come with their parents, and well come with ours. (Eles viro com os pais deles e ns,
com os nossos.

Reflexive pronouns
Reflexive Pronouns
Myself
Yourself
Himself
Herself
Itself
Ourselves
Yourselves
Themselves

A identificao dos Reflexive pronouns bastante simples: eles so aqueles terminados em self (singular)
ou selves (plural) e tem uso reflexivo(como o prprio nome sugere) ou enftico.

Uso reflexivo
Este o uso mais comum e acontece quando o sujeito e objeto em uma orao so o mesmo.
Exemplo: I consider myself a good listener. (Eu me considero um bom ouvinte.)
No exemplo anterior, o sujeito da orao (I) o mesmo do objeto (myself). Quando esta situao acontece,
necessrio o uso do correspondente reflexive pronoun.
Importante: Devido interferncia da lngua portuguesa, bastante comum a ocorrncia de erros como
I consider me a good listener.
24

Observe agora usos com os outros pronomes:


1. He considers himself a good listener.(Ele se considera um bom ouvinte)
2. She considers herself a good listener. (Ela se considera uma boa ouviente)
3. The picture speaks for itself. (A foto/imagem fala por si mesma)

Listeners (Plural)
1a pessoa pl - We consider ourselves good listener. (Ns nos consideramos bons ouvintes)
2a pessoa pl -You consider yourselves good listener. (Vocs se consideram bons ouvintes)
3a pessoa pl -They consider themselves good listener. (Elas/eles se consideram bons ouvintes)

Uso Reflexivo
Este uso especfico dos Reflexive Pronouns costuma causar certo desconforto ao leitor brasileiro. Vamos observar
alguns exemplos:
1. She herself knows how to fix it.(Ela mesma sabe como consertar isto)
2. I myself dont understand the situation. (Eu mesmo/Eu pessoalmente no entendo a situao).
Assim como no uso reflexivo, existe repetio de sujeito/objeto.

25

E.O. Teste I
Para os prximos 8 exerccios, escolha a
alternativa que faz uso correto dos pronomes.
1. George is always daydreaming. _________ is
never connected to reality.
a) Him
b) Them
c) Ourselves
d) He
e) She
2. I depend on my computer. I simply cant live
without _______.
a) Him
b) He
c) Them
d) Its
e) It
3. In our modern world, computers are
everywhere. Smart people have to learn how
to deal with __________.
a) Itself
b) Them
c) It
d) Their
e) Theirs
4. Janeth entered the room and took a quick
look at __________ in the mirror.
a) Ourselves
b) Them
c) Her
d) Himself
e) Herself

5. Parents should keep an eye on ____________


young kids at all times.
a) Their
b) Them
c) Themselves
d) Yours
e) Theirs

6. You got a loan to open your business. I will


do the same to open __________.
a) Me
b) Myself
c) Mine
d) Their
e) My
26

7. He ______________ built the car that won


the championship.
a) Him
b) Himself
c) Herself
d) His
e) Ourselves
8. Sarah likes______, but I dont like _______.
a) Me her
b) I she
c) Me hers
d) I her
e) Me herself
TEXTO PARA AS PRXIMAS 2 QUESTES
Did you know?
Where there are salmon streams there usually are bears. And Vancouver Island, with its
abundant salmon streams, seems like perfect bear habitat. The American black bear
(Ursus americanus) does inhabit the island,
but oddly there are no grizzly bears (Ursus
arctos), even though many are found just a
short distance away on the mainland of British Columbia. Why? Some scientists believe
that the two species of bear simply cannot
coexist on islands, even though they do coexist on the continent perhaps it would
mean too much competition in a confined
space. But the fact of the matter is that no
one really knows. Alice J. Dunn
Disponvel em: <http://magma.
nationalgeographic. com/ngm/0302/feature06/
index.html>. Acessado em 04 mar. 2004.

9. (Fatec) Which of the sentences below is false, according to the text?


a) Grizzly bears inhabit the mainland of British
Columbia
b) The American black bear and the grizzly
bear coexist on the continent.
c) The American black bear and the grizzly
bear coexist on islands.
d) Bears usually live near salmon streams.
e) There are a lot of salmon streams on Vancouver Island.
10. (Fatec) Its in ... with its abundant salmon
streams ... its refers to:
a) a salmon stream.
b) Vancouver Island.
c) the American black bear.
d) British Columbia.
e) a grizzly bear.

E.O. Teste II
1. (Cesgranrio) Mark the option which
completes the following sentence with the
adequate pronouns:
I. Businessmen have ........ own priorities.
II.
Everyone must feel happy with ........
working habits.
III. Working from home allows a mother to
spend more time with ........ children.
IV. If you have never tried to work at home,
you cannot discuss ........ disadvantages.
a) I his, II their, III her, IV their.
b) I their, II its, III their, IV its.
c) I their, II his, III her, IV its.
d) I its, II your, III its, IV their.
e) I his, II his, III their, IV your.
TEXTO PARA A PRXIMA QUESTO
Perhaps the only small thing native to Texas
are pecans. Nearly blind Grandma Shaw shared
her top-secret recipe with son Norman, who
now bakes the tiny pecans (which have more
natural oils and juice than the larger ones)
in a brown sugar filling and flaky crust. Just
pop IT in the oven for eight minutes at 300
degrees, add a scoop of vanilla ice cream, and
bliss! One 9-inch pecan pie is $16.95, which
includes free shipping by second-day air in
the continental United States. Contact Cryer
Creek Kitchens. Phone: 800-468-0088. Fax:
903-872-9204.
2. (Uel) In the text, it refers to:
a) oven.
b) rust.
c) scoop.
d) recipe.
e) ice cream.
TEXTO PARA A PRXIMA QUESTO
Literature is the only place in any society where, within the secrecy of our own heads, we can
hear voices talking about everything in every
possible way. The reason for ensuring that privileged arena is preserved is not that writers
want the absolute freedom to say and do whatever THEY please. It is that we, all of us, readers and writers and citizens and generals and
godmen need that little, unimportant-looking
room. We do not need to call it sacred, but we
need to remember it is necessary.
3. (Uel) In the text, THEY refers to:
a) readers.
b) citizens.
c) voices.
d) writers.
e) heads.

TEXTO PARA A PRXIMA QUESTO


The political party may well be the most important institution in a democracy, for it is the
instrument through which modern democracies govern themselves. Parties not only recruit
people to fill top government positions and
formulate the policies those officials will carry out; they also bring together under THEIR
banners disparate sectors of the electorate,
thereby providing government with a broadly
based popular legitimacy. These functions are
essential to the operation of any government.
4. (FGV) O pronome em maisculo THEIR
refere-se a:
a) policies.
b) people.
c) officials.
d) parties.
e) democracies.
5. (Unitau) Assinale a alternativa que
corresponde sequncia de pronomes que
mais adequadamente completam a sentena a
seguir:
____cat is sick because______ ate _____
spoiled food over there.
a) its; he; that
b) its; he; this
c) his; its; this
d) its; it; that
e) his; it; that
TEXTO PARA A PRXIMA QUESTO
Why is the South Pole colder than the North
Pole? Robert Bindschadler, a senior fellow and
glaciologist at NASAs Goddard Space Flight
Center, explains.
Both polar regions of the earth are cold, primarily because they receive far less solar radiation than the tropics and mid-latitudes do.
At either pole the sun never rises more than
23.5 degrees above the horizon and both locations experience six months of continuous
darkness. Moreover, most of the sunlight that
does shine on the polar regions is reflected
by the bright white surface. What makes the
South Pole so much colder than the North Pole
is that it sits on top of a very thick ice sheet,
which 1itself 4sits on a continent. The surface
of the ice sheet at the South Pole is more than
9,000 feet in elevation - more than a mile and
a half above sea level. Antarctica is by far the
highest continent on the earth. In comparison, the North Pole 2rests in the middle of the
Arctic Ocean, where the surface of floating ice
5
rides only a foot or so above the surrounding
sea. The Arctic Ocean also 3acts as an effective heat reservoir, warming the cold atmosphere in the winter and drawing heat from
the atmosphere in the summer.
<http://www.sciam.com>

27

6. (UFMG) The word itself (ref. 1) refers to:


a) a thick ice sheet.
b) a continent.
c) the South Pole.
d) the North Pole.
TEXTO PARA A PRXIMA QUESTO
Answer the questions with information from the text.

7. (PUCRS) Fill in the gaps with the suitable pronouns:


a) my she her his
b) my it our its
c) his it his her
d) his he his its
e) its he our our
TEXTO PARA AS PRXIMAS 3 QUESTES
Drinking problem
For years, athletes have been told to gulp lots of water to avoid dehydration. But a new study in
The New England Journal of Medicine shows that some long-distance runners are overdoing
it. Researchers found that 13 percent of 488 competitors at the 2002 Boston Marathon drank so
much fluid they developed hyponatremia, a life-threatening condition in which the blood salt
levels plummet. But with warmer weather on the horizon, weekend warriors still need to worry
about dehydration, which can lead to heat exhaustion and heat-stroke. According to the American
Council on Exercise (acefitness.org), drinking plain H2O is usually preferable to sports drinks. For
moderate exercise of 60 minutes or less, drink about eight ounces of water 20 to 30 minutes prior
to working out; four to eight ounces every 10 to 15 minutes during exercise and eight ounces or
more within 30 minutes of finishing.
8. (Unesp) Os pronomes it e they, em destaque no texto, referem-se respectivamente a:
a) beber muita gua e 488 competidores.
b) desidratao e pesquisadores.
c) beber muita gua e 13 por cento dos 488 competidores.
d) desidratao e 13 por cento dos 488 competidores.
e) beber muita gua e pesquisadores.
9. (Unesp)A causa para os atletas citados no texto desenvolverem hiponatremia foi:
a) o nvel de sal no sangue ter permanecido o mesmo.
b) consequncia de desidratao durante a competio.
c) consequncia da exausto devido competio.
d) o fato de serem corredores de longa distncia.
e) a ingesto de muito lquido durante a competio.
10. (Unesp) As duas ocorrncias da conjuno but no texto apresentam uma ideia de:
a) adio.
b) causa.
c) concesso.
d) contraste.
e) substituio.
28

E.O. Teste III


TEXTO PARA AS PRXIMAS 3 QUESTES
Climate change: forecast for 2100 is floods
and heat ... and its mans fault
By Nick Allen

9:04PM BST 16 Aug 2013


Climate scientists have concluded that temperatures could jump by up to 5C and sea
levels could rise by up to 82 cm by the end
of the century, according to a leaked draft
of a United Nations (UN) report.
The UN Intergovernmental Panel on
Climate Change (IPCC) also said there was a
95 per cent likelihood that global warming
is caused by human activities. That was the
highest assessment so far from the IPCC,
which put the figure at 90 per cent in a
previous report in 2007, 66 per cent in
2001, and just over 50 per cent in 1995.
Reto Knutti, a professor at the Swiss Fede-ral Institute of Technology in Zurich, said:
We have got quite a bit more certain that
climate change is largely man-made. Were
less certain than many would hope about
the local impacts. The IPCC report, the
first of three in 2013 and 2014, will face
intense scrutiny particularly after errors in
the 2007 study, which wrongly predicted
that all Himalayan glaciers could melt by
2035. Almost 200 governments have agreed to try to limit global warming to below
2C above pre-industrial times, which is
seen as a threshold for dangerous changes
including more droughts, extinctions, floods and rising seas that could swamp coastal
regions and island nations. Temperatures
have already risen by 0.8C since the Industrial Revolution.
The report will say there is a high risk global temperatures will rise by more than
2C this century. They could rise anywhere from about 0.6C to almost 5C a wider
range at both ends of the scale than predicted in the 2007 report. It will also say
evidence of rising sea levels is unequivocal. The report projects seas will rise by
between 30 cm and 82 cm by the late 21st
century. In 2007 the estimated rise was between 18 cm and 58 cm, but that did not
fully account for changes in Antarctica and
Greenland.
Scientists say it is harder to predict local
impacts. Drew Shindell, a Nasa scientist,
said: I talk to people in regional power
planning. They ask, Whats the temperature
going to be in this region in the next 20 to
30 years, because thats where our power
grid is? We cant really tell.
Adapted from <telegraph.co.uk>

1. (UFPR) Consider the following statements


concerning global warming and the leaked
draft of the IPCC report:
1. Scientists think it is 95% likely that
human activity is causing global warming.
2. Temperatures could be 5C warmer by
the end of the current century.
3. Sea levels are not likely to be higher than
today by the end of the century.
4. Scientists are surer now than in 2007
that humans are causing global warming.
5. 50% of the scientists believed humans
were the cause of climate change in 1995.
Which of the statements above are TRUE,
according to the text?
a) Only statements 1, 3 and 5.
b) Only statements 2, 3 and 4.
c) Only statements 3 and 5.
d) Only statements 1, 2 and 4.
e) Only statements 2 and 5.
2. (UFPR) Considering what the text says about
the IPCC and its predictions and conclusions
on global warming, mark true (T) or false
(F) for the following statements:
( ) The IPCC made a wrong prediction about
the Himalayas in the 2007 report.
( )
Himalayan
glaciers
will
certainly
disappear by 2035 because of global
warming.
( ) The IPCC can now be sure of how climate
change will impact different locations.
( )
IPCCs new report will be carefully
examined after the errors committed in
2007.
( ) Global warming will have a huge impact
in Swiss because of its large glaciers.
Mark the alternative which presents the correct
sequence, from top to bottom.
a) T F F T F.
b) F T T F T
c) T F T F T.
d) F F F T T.
e) T T T T F.
3. (UFPR) Mark the correct alternative, according
to the text. The word it, in boldface and
italics (paragraph 5), refers to:
a) global warming.
b) the greenhouse effect.
c) rising sea levels.
d) the 21st century.
e) the IPCC report.
TEXTO PARA AS PRXIMAS 7 QUESTES
One of the major effects of eating too much
sugar is a high incidence of tooth decay.
When we eat something with sugar in it, particularly refined sugar, enzymes in the saliva
in the mouth begin to work immediately to
29

change that sugar into a type of carbohydrate. 1As one eats, particles of the sugary food
get stuck between the teeth and around the
gums. As the food changes its chemical composition, the resultant carbohydrate produces
bacteria that begin to eat away at the enamel
on the outside of our teeth. This is 2actually
the decaying of the tooth. 3Now, if this process happens each time we eat sugar, we can
see that eating excessive amounts of sugar
causes more and more tooth decay. It is true
that some tooth decay can be avoided with
immediate brushing after eating, removing
all the particles of food trapped in the teeth. 4However, sweets are often eaten as snacks between meals and during the day, times
when people generally do not brush after
eating. 5Therefore, the dangerous process of
tooth decay is allowed to continue.
Smalley, R. L. and Hank, M.R. Refining composition
skills. 1982, Macmillan Publishing Co., Inc., p.255.

4. (Cesgranrio) Bacteria are very small organisms


which...
a) refine sugary food around the gums.
b) wear away the enamel of the teeth.
c) result in carbohydrates in the mouth.
d) change the composition of the process.
e) stick to the particles of sugary food.
5. (Cesgranrio) According to the text, whenever
we eat sugary food...
a) enzymes change it into refined sugar.
b) there is a low incidence of tooth decay.
c) the sugar in the mouth turns into saliva.
d) the teeth begin to work into the carbohydrates.
e) a type of carbohydrate is produced by the
saliva.
6. (Cesgranrio) According to the text, eating
large amounts of sugar...
a) results in accumulating weight.
b) causes growing chemical composition.
c) postpones the process of tooth decay.
d) brings about the decaying of the teeth.
e) wears away carbohydrate gain in the body.
7. (Cesgranrio) According to the text, what happens to the food we eat?
a) It gets stuck when we eat gum.
b) It is eaten away by the bacteria.
c) It attacks the enamel covering the teeth.
d) It produces particles of bacteria in the
mouth.
e) It has its chemical composition changed in
the mouth.
30

8. (Cesgranrio) In order to prevent tooth decay


one should...
a) remove the decayed tooth.
b) avoid eating particles of food.
c) eat neither between meals nor during the
day.
d) brush the teeth immediately before eating.
e) brush the teeth immediately after having
eaten.
9. (Cesgranrio) The pronoun IT in the sentence
When we eat something with sugar in
it, particularly refined sugar, enzymes
in the saliva in the mouth begin to work
immediately to change that sugar into a
type of carbohydrate. refers to the word...
a) saliva.
b) sugar.
c) mouth.
d) something.
e) refined sugar.
10. (Cesgranrio) Mark the option that contains
the appropriate pronouns to complete the
sentences below.
Animals teeth are changing ...(I)...
composition.
That animal had ...(II)... teeth in perfect
conditions.
He brushes ...(III)... teeth whenever he eats
something.
If the patient dies, we call ...(IV)... relatives.
The bacteria found ...(V)... way to the
stomach.
a) (I) their, (II) its, (III) his, (IV) his, (V) their
b) (I) its, (II) their, (III) its, (IV) his, (V) its.
c) (I) their, (II) its, (III) her, (IV) her, (V) his.
d) (I) his, (II) their, (III) his, (IV) her, (V) their.
e) (I) their, (II) his, (III) their, (IV) its, (V) her.

E.O. Dissertativo
1. Complete the text below using personal, possessive or reflexive pronouns.
Ernest Hemingway was probably the greatest American writer in the 20th century.
__________ was born in the USA in the beginning of the 20th century but traveled and
lived in many countries. All of _________
books deal mostly with the universe of the
simple men. This is exactly what happens in
his most famous book The Old Man and the
Sea. _________ is about a fisherman who
is facing many difficulties to earn a life. In
one last try to get some fish, the fisherman
goes to the sea and catches a gigantic fish,
ties __________ to his boat and takes it to
his village, so it can be eaten by people. He
thinks that the fish is so big that it can be

food to all of _________.


All Hemingways works are extremely famous around the world, especially three of
_________: For whom the bell tolls, Snows
of Kilimanjaro and Farewell to Arms. He-mingway __________ was a fisherman,
pugilist and fought in the Spanish Civil
War in the 1930s. In America, Hemingway
is still seen as a true hero and people use
__________ personal history to inspire
_________ lives. Unfortunately, Hemingway
fell into a very strong depression and killed
_________ in 1961, what couldnt be enough
to erase all his achievements.

Gabarito
E. O. Teste I
1. D

2. E

3. B

4. E

5. A

6. C

7. B

8. A

9. C

10. B

E. O. Teste II
1. C

2. B

3. D

4. D

5. E

6. A

7. B

8. C

9. C

10. D

E. O. Teste III
1. D

2. A

3. E

4. B

5. E

6. D

7. E

8. E

9. D

10. A

E. O. Dissertativo

1.
he his - it it them them himself
his - their- himself

31

Zurijeta/Shutterstock

Class 3
The here and now

Present tenses (tempos verbais de presente)


A ideia desta unidade fazer uma coletnea das formas verbais da lngua inglesa que, de uma maneira ou de
outra so compreendidas como formas verbais dos tempos presentes do portugus. Antes de qualquer coisa vital
entender que quando algum l um texto em lngua estrangeira, esta pessoa no est traduzindo o texto, mas sim
negociando o seu significado com a lngua materna desse leitor, no nosso caso, com o portugus. A traduo uma
diviso do estudo de idiomas e apesar do senso comum dizer eu no estou conseguindo traduzir este texto, no
h tempo hbil nos vestibulares para que se faa uma traduo.
Vrios tempos e formas verbais da lngua inglesa, aps uma rpida negociao de seu significado enquanto
se executa a leitura, chegam para o leitor como se estivessem no presente em portugus, independente de como
so chamadas em ingls. Portanto, a abordagem aqui proposta consiste em agrupar essas formas e tempos verbais
do ingls. So elas: simple present, to be, there to be e alguns modal verbs.

Simple present (regra geral)


A grande maioria dos verbos em ingls segue as regras apresentadas nesse item, portanto muita ateno.
Ao contrrio do portugus, que apresenta mltiplas desinncias, o ingls tem como regra geral (excees
sero vistas no final deste item) a adio da letra s s terceiras pessoas do singular (he, she, it). Observe os
exemplos a seguir:
To need = precisar, necessitar
1. I need to study better. (Eu preciso estudar melhor)
2. You need to study better. (Voc precisa estudar melhor)
3. He needs to study better. (Ele precisa estudar melhor)
4. She needs to study better. (Ela precisa estudar melhor)
5. It needs improvements. (Isto precisa de melhorias)
6. We need to study better. (Ns precisamos estudar melhor)
7. You need to study better. (Vocs precisam estudar melhor)
8. They need to study better. (Eles/elas precisam estudar melhor)
Percebe-se que enquanto a lngua inglesa acrescenta ao bare infinitive dos verbos (infinitivo sem a partcula to) a
letra s aos pronomes de terceira pessoa do singular (he, she, it), a lngua portuguesa acrescenta vrias desinncias
ao verbo (preciso, precisa, precisamos etc)

33

Casos particulares:
1. To have = ter, possuir
Exemplo: I have, you have, he/she/it has, we have you have, they have. (NUNCA use he/she/it
haves)
2. Verbos terminados em s, y(precedido por vogal), ch, sh, x e o, z, ss acrescenta-se es
ao invs de somente s.
Exemplos: I wash He washes

You watch She watches

They go It goes

I fix She fixes

3. Verbos terminados em y precedidos de consoantes. Substitua o y por ies


Exemplos: I fly it flies

They try He tries.

Formas negativas do simple present


A forma negativa dos verbos no simple present segue uma norma bem simples: acrescente do not (dont) quando
o sujeito for I, you, we, they; acrescente does not (doesnt) quando o sujeito for he, she, it.
Importante: Aps o uso dos auxiliares do ou does, utilize o base form (infinitivo sem o to).

Exemplos:
1. I dont (do not) need to study better. (Eu no preciso estudar melhor)
2. You dont (do not) need to study better. (Voc no precisa estudar melhor)
3. He doesnt (does not) need to study better. (Ele no precisa estudar melhor)
4. She doesnt (does not) need to study better. (Ela no precisa estudar melhor)
5. It doesnt (does not) need improvements. (Isto no precisa de melhorias)
6. We dont (do not) need to study better. ((Ns no precisamos estudar melhor)
7. You dont (do not) need to study better. (Vocs no precisam estudar melhor)
8. They dont (do not)need to study better. (Eles/elas no precisam estudar melhor)

Formas interrogativas
Para que se obtenham as formas interrogativas dos verbos no simple present, deve-se fazer uso dos verbos auxiliaries do (para os pronomes I, you, we, they) e does (he, she, it), colocados antes dos respectivos sujeitos.
Observe os exemplos abaixo:
1. Do I need to study more?. (Eu preciso estudar mais?)
2. Do you need to study more? (Voc precisa estudar mais?)
3. Does he need to study more? (Ele precisa estudar mais?)
4. Does she need to study more? (Ela precisa estudar mais?)
5. Does it need improvements? (Isto precisa de melhorias?)
6. Do we need to study more? ((Ns precisamos estudar mais?)
7. Do you need to study more?. (Vocs precisam estudar mais?)
8. Do they need to study melhor? (Eles/elas precisam estudar mais?)
34

To be
O verbo to be no segue as mesmas regras que orientam a maioria dos verbos em ingls. Ele o nico que apresenta trs formas de presente. So elas:
1. I am Brazilian. (Eu sou brasileiro)
2. He, she it is from the Netherlands. (Ele/ela/isto da Holanda.)
3. You, we, they are thirsty.(Voc(s), ns, eles(as) esto com sede)
Talvez a maior dificuldade ao se lidar com o verbo to be resida no fato de que ao ser vertido para o portugus, ele possa ter tanto estar quanto ser como significado, dependendo do contexto em que o verbo est
inserido.

Formas negativas
Para se conseguir as formas negativas do verbo to be, deve-se apenas acrescentar a partcula not forma afirmativa. Observe as contraes possveis (abaixo apresentadas).
1. I am not Brazilian. (Eu no sou brasileiro)
2. He, she it is not (isnt) from the Netherlands. (Ele/ela/isto no da Holanda.)
3. You, we, they are not (arent) thirsty. (Voc(s), ns, eles(as) no esto com sede.)

Formas interrogativas
Para que se obtenham as formas interrogativas do verbo to be, deve-se apenas inverter a posio do verbo e do
sujeito. Observe os exemplos:
1. Am I right? (Eu estou correto?)
2. Are you (we/they) interested? (Vocs, elas(es) ns estamos interessados?)
3. Is he (she/it) Australian? (Ele/ela/isto da Austrlia?)

There to be
A estrutura there to be normalmente entendida como o verbo haver, existir, ocorrer. Ele apresenta duas formas no
presente there is (para o singular) e there are (para o plural).

Exemplos:
1. There is a school near my home. (H/existe uma escola perto de casa)
2. There are many schools near home. (H/existem muitas escolas perto de casa)
3. Deve-se acrescentar a partcula not s formas afirmativas do there to be para que se obtenham suas respectivas formas negativas. Observe as contraes possveis (abaixo apresentadas)
There is not (isnt) a single drugstore around here.(No h/existe uma nica farmcia prximo daqui)
There are not (arent) drugstores on your street. (No h/existem farmcias na sua rua)
35

Formas interrogativas
Para que se obtenham as formas interrogativas do verbo there to be, deve-se apenas inverter a posio do verbo
e do sujeito. Observe os exemplos:
1. Is there a school near your home? (H/existe uma escola perto de sua casa?)
2. Are there many schools near your home? (H/existem muitas escolas perto de casa?)

Can/could/may/might/must/should (modal verbs)


Os verbos neste item so classificados como modal verbs e sero assim estudados em unidades seguintes. Porm,
quando traduzidos para o portugus acabam soando como verbos no presente simples, e por isso esto tambm
nesta unidade. Por serem extremamente comuns e essenciais para a compreenso dos textos, sero agora estudados.
1. They can consider your offer - Eles

considerar sua oferta. (Existe boa possibilidade de ela ser aceita)

2. They could consider your offer - Eles podem/poderiam considerar sua oferta (A possibilidade existe, mas
menor do que no exemplo anterior)
3. They may/might consider your offer Eles podem/poderiam/talvez aceitem sua oferta (Nestes casos, a possibilidade ainda menor)
4. You should consider their offer Voc deve/deveria considerar a oferta deles (como conselho ou sugesto)
5. You must consider their offer Voc deve/deveria considerar a oferta deles (Tambm como conselho, mas
mais urgente, impositivo)

Formas negativas
Deve-se acrescentar a partcula not s formas afirmativas dos modal verbs para que se obtenham suas respectivas
formas negativas. Observe as contraes possveis (abaixo apresentadas).
1. I cannot (cant) consider your offer. (Eu no posso considerar sua oferta)
2. He could not (couldnt) consider your offer. (Ele no poderia considerar sua oferta?)
3. You may not like their offer. (Voc no poderia considerar a oferta deles)
4. You might not like their offer. (Voc no poderia considerar a oferta deles)
5. They should not (shouldnt) consider your offer. (Elas no deveriam considerar sua oferta)
6. You must not (mustnt) consider their offer. (Voc no deve/deveria considerar a oferta delas)

Formas interrogativas
Para que se obtenham as formas interrogativas dos modal verbs, deve-se apenas inverter a posio do verbo e do
sujeito. Observe os exemplos:
1. Can they consider my offer? (Eles podem considerar minha oferta?).
2. Could they consider your offer? (Eles podem/poderiam considerar sua oferta)
3. May you consider their offer? (Vocs poderiam considerar minha oferta?)
4. Should I consider their offer? (Eu devo/deveria considerar a oferta deles?)
5. Must you really consider their offer? (Voc deve/deveria realmente considerar a oferta deles?)
36

Present continuous (present progressive)


O Present Continuous apresenta enorme semelhana com o tempo verbal Presente Contnuo da lngua portuguesa.
Ele um tempo verbal que serve para expressar aes que esto acontecendo simultneas fala. O Present Continuous (ou Progressive) formado pelo presente do verbo to be (concordando com cada sujeito) mais um outro
verbo no gerund (sufixo ING).
Veja os exemplos:
1. I am typing a letter right now. (Eu estou digitando uma carta neste exato momento.)
2. You are reading my letter. (Voc est lendo minha carta.)
3. He is writing a letter on his computer. (Ele est escrevendo uma carta no computador dele.)
4. She is studying in her room. (Ela est estudando no quarto dela.)
5. It is working very well. (Isto est funcionando muito bem.)
6. We are watching a very interesting documentary. (Ns estamos assistindo um documentrio muito interessante.)
7. You are wasting your time. (Vocs esto desperdiando seu tempo.)
8. They are surfing the web on their tablet. (Elas(es) esto navegando na internet em seu tablet.)
As formas interrogativas do Present Continuous so dadas pela inverso do sujeito com o to be.

Exemplos:
1. Are you paying attention to the classes? (Voc est prestando ateno s aulas?)
2. Is it working properly? (Isto est funcionando corretamente?)
As formas negativas do Present Continuous so dadas pela adio de not s formas do to be.
1. She is not (isnt) writing a letter. (Ela no est escrevendo uma carta.)
2. I am not thinking about it. (Eu no estou pensando sobre isso.)

Apndice
Uso enftico dos auxiliares do e does
Alm de serem utilizados nas formas negativas e interrogativas, os auxiliares do e does tambm podem ser
utilizados em frases afirmativas com funo enftica. Estude os exemplos a seguir:
1. She loves you (Ela te ama) She does love you (Ela realmente te ama)
2. I speak English (Eu falo ingls) I do speak English (Eu realmente falo/ Eu falo muito bem ingls)

37

E.O. Teste I
1. A forma interrogativa de Beto lives in Paris :
a) Does Beto lives in Paris?
b) Do Beto lives in Paris?
c) Is Beto live in Paris?
d) Does Beto live in Paris?
e) Does not Beto live in Paris?
2. A forma negativa de People are not afraid
of surgeries :
a) People are not afraid of surgeries.
b) People dont afraid of surgeries.
c) People doesnt afraid of surgeries.
d) People not afraid of surgeries.
e) Are not people afraid of surgeries.
3. A frase, em ingls, correspondente a Voc
no sabe que o teste de matemtica
amanh? :
a) Dont you know that the Math test is tomorrow?
b) Do you know not that the Math test is tomorrow?
c) Do not you know that the Math test is tomorrow?
d) You know not that the Math test is tomorrow?
e) Not you know that the Math test is tomorrow?
4. A frase Jimmy plays the guitar very well
tem como forma negativa:
a) Jimmy doesnt plays the guitar very well.
b) Jimmy dont play the guitar very well.
c) Jimmy doesnt play the guitar very well.
d) Jimmy plays not the guitar very well.
e) Jimmy not play the guitar very well.

8. He ____________ pudding. Why __________


him something else?
a) dont like / do you offer
b) doesnt likes / dont you offer
c) doesnt like / dont you offers
d) doesnt like / dont you offer
e) arent like / do not you offer
9. Complete with Simple Present or Present
Continuous (Progressive)
She usually _______________ against
injustice, but at this moment she
_______________ against unemployment.
a) protest / protesting
b) protests / is protesting
c) is protesting / protests
d) protest / are protesting
e) protested / are protesting
10. Complete with Simple Present or Present
Continuous (Progressive)
Today he _______________ jeans and Tshirts, but he usually _______________ a
suit at work.
a) is wearing / wears
b) wears / is wearing
c) wearing / wear
d) wear / are wearing
e) has wearing / wearing

E.O. Teste II
TEXTO PARA A PRXIMA QUESTO

5. Complete with Simple Present or Present


Continuous (Progressive)
She generally _______________ the piano,
but at present she _______________ the
guitar.
a) is playing / plays
b) is plaing / plays
c) plays / is playing
d) playing / are playing
e) plays / is plaing
6. (Unesp) He doesnt_______ anymore.
a) smoking
b) no smoking
c) smokes
d) smoked
e) smoke
7. Books ___________ very good companions,
arent they?
a) are
b) is
c) arent
d) dont be
e) isnt
38

BRAZILS NETWORK BOOM


BRAZIL IS ON THE VERGE OF A NETWORK
SURGE. BUT EXACTLY HOW THEYLL ALL IS
STILL UP IN THE AIR.
Probably the only thing that Brazils two
pay TV heavyweights, Globo and TVA, agree
___(I)___ is that the countrys multichannel
business is on the verge of a boom. The two
companies, which have fought one of the
most IMPASSIONED battles for dominance
to be found anywhere in the pay TV world,
___(II)___ the intensity of their cable
and wireless competition and extending
it to direct-to-home television this year.
And with the number of Brazilian pay TV
subscribers expected to ___(III)___ fivefold
to 5 million by the end of the decade, both
sides are FEVERISHLY putting together
new programming services to make their
packages as ALLURING as possible. (...)
by Ian Katz Multichannel News. International,

1. (Ita) Quais os verbos que devem preencher


as lacunas II a III respectivamente?
a) are rising - raise
b) are raising - rise
c) are rising - rise
d) is raising - raise
e) is rising - rise
TEXTO PARA A PRXIMA QUESTO
Why do bees fuss about so much when
they fly, instead of forming a tidy flock
like birds? Birds flying in a flock keep to
a highly ordered pattern, whereas a swarm
of bees is a cloud of chaos. This difference
has long puzzled scientists, but now a team
of Japanese researchers has come up with
a simple mathematical model to explain
it. [The researchers] began with a simple
analogy. Stars in a galaxy move under the
influence of each others gravity in a way
that can be described by Newtons laws.
Identify the influences felt by an insect or
bird, the researchers reasoned, and its flying
patterns should be just as easy to predict.
Adapted from New Scientist, 15 June 1996

2. (Fuvest) A forma correta do singular de


Why do bees fuss about so much when they
fly? :
a) Why does bee fuss about so much when it
fly?
b) Why do an bee fusses about so much when it
flies?
c) Why does a bee fuss about so much when it
flies?
d) Why does the bee fuss about so much when
it fly?
e) Why does a bee fusses about so much when
it flies?
TEXTO PARA A PRXIMA QUESTO
Tess of the DUrbervilles is the story of the
seduction, betrayal, and destruction of an
innocent girl, Tess Durbeyfield, who is led
by her foolish parents into thinking she
comes from an ancient noble family, the
DUrbervilles. Encouraged to claim Kingship
with the family, Tess is seduced by the suave, plausible Alec DUrberville, who abandons her when she bears his baby. The child
dies, and Tess finds a new love with the egotistic, self-righteous Angel Clare. When he
hears her story on their wedding night, he
too abandons her. In despair, Tess murders
Alec. She 1finds a few fleeting days of hap-piness with Clare, who returns to her before
she is captured and hanged. In the famous
last lines of the novel, which could fit any
other of Hardys works almost as well,

Justice was done, and the President of the


Immortals ... had ended his sport with views
through compassionate eyes the difference
between the fate human beings deserve and
the one that they suffer.
3. (UFRGS) The correct verbal forms of
the nouns seduction, betrayal, and
destruction are:
a) seduce - betray - destroy.
b) seduct - betray - destroy.
c) seduce - betrayal - destruct.
d) seduct - betrayal - destruct.
e) seduce - betray - destruct.
TEXTO PARA A PRXIMA QUESTO
International advertising can be a risky business. When McDonalds launched Le Big
Mac in Paris, it discovered that in local slang
this meant the big pimp. It is not just a
question of language either; national advertising styles also vary considerably. The
British like humor and irony in their ads,
whereas the Germans regard this approach
as frivolous. The French are more sexist than
the British and will use semi naked women
in almost any context. The Italians generally
like to see beautiful people wearing beautiful clothes driving beautiful cars. These
are not just national stereotypes, but based
on hard experience. Different countries also
prefer different products.

4. (UFPB) The text is PREDOMINANTLY in the


_________.
a) present tense
b) past tense
c) future tense
d) present perfect tense
e) present progressive tense
5. (Mackenzie) Indicate the alternative that
best completes the following sentence. I
________ WHEN ________ THAT I HAVE TO
STUDY.
a) dont like - she says
b) never like - she will tell me
c) cant like - she says
d) mustnt like - she speaks
e) dont like it - she tells me
6. (Mackenzie) Indicate the alternative that
best completes the following sentence. This
guide______ useful______about the city.
a) gives - information
b) have given - information
c) is giving - piece of informations
d) gave- pieces of informations
e) will give - pieces of informations
39

TEXTO PARA A PRXIMA QUESTO


NORTH KOREA
SHOP TILL YOU DROP... THE BOMB?
The Tokyo District called Akihabara is a
vast market for cool videogames, computer
gear and pet robots. But according to some
young Japanese parliamentarians, Akihabara
is to North Korean spies what U.S. atomicenergy labs are to Chinese spooks. The
North Koreans troll for gadgets to improve
Pyongyangs weapons - everything from
fish finder sonar sets to handheld Global
Positioning Systems.
Aboard one North Korean submarine captured
by South Korea last year, all the radar, GPS
and computer terminals were made in Japan,
says Ichita Yamamoto of the ruling Liberal
Democratic Party. He says Japanese technology
is also used in North Koreas Taepodong
missile, which is said to be capable of hitting
targets as far away as Alaska and Hawaii.
Last week the young lawmakers presented
legislation to block the sale of any offtheshelf item deemed useful for North
Korean weapons. But Japan has a history
of lax export controls. And the weapons
builders are finding more and more uses
for civilian technology. Which explains why
Akihabara has become North Koreas Los
Alamos.
Newsweek

7. (Fatec) Assinale a alternativa que contm,


respectivamente, os mesmos tempos verbais
dos verbos destacados na frase a seguir: He
SAYS Japanese technology IS also USED in
North Koreas....
a) rans / is run
b) lies / is layd
c) lais / is lain
d) lays / is laid
e) runs / is ran
8. (Mackenzie) Indicate the alternative that
best completes the following sentence.
Paulo knows how to drive a truck and
______.
a) Mark does neither
b) either does Mark
c) so does Mark
d) nor does Mark
e) Mark does either
TEXTO PARA A PRXIMA QUESTO

40

their families on Christmas Day, December


25th. For most British families, this is the
most important festival of the year, it combines the Christian celebration of the birth
of Christ with the traditional festivities of
winter. On Sunday before Christmas many
churches hold a carol service where special
hymns are sung. Sometimes carol-singers
can be heard on the streets as they collect
money for charity. Most families decorate
their houses with brightly-colored paper,
and they usually have a Christmas tree in
the corner of the front room, glittering with
colored lights and decorations.
There are a lot of traditions connected with
Christmas, but perhaps the most important
one is the giving of presents. Family
members wrap up their gifts and leave them
at the bottom of the Christmas tree to be
found on Christmas morning. Small children
believe that their gifts come from Santa
Claus. Their parents tell them that Santa
Claus lives in the North Pole and, on the
night before Christmas, he travels the world
in a sled pulled by reindeer. He goes down
the chimneys of houses to leave gifts only
for children who have been good. At some
time on Christmas Day, the family will sit
down to a big turkey dinner followed by
desserts specially prepared for the occasion.
Later in the afternoon, they may watch
the Queen on television as she delivers her
traditional Christmas message to the United
Kingdom. If they have room for even more
food they may enjoy a piece of Christmas
cake. December 26th is also a public
holiday, and this is the time to visit friends
and relatives or be a spectator at one of the
many sporting events.
From: SHEERIN, S.; SEATH, J.; WHITE,
G. Spotlight on Britain. Oxford: Oxford
University Press, 1995, p.13. (Adapted)

9. (UFSC) Choose the correct proposition(s) to


complete the following paragraph. Children
______ a long sock at the end of their bed on
Christmas Eve, December 24th, ______ that
Santa Claus will come down the chimney
______ night and bring them small presents,
fruit and nuts. Their expectations are usually
not ______!
01) hang - looking - before the - disapproved
02) leave - hoping - during the - disappointed
04) hide - thinking - on - motivated
08) put - expecting - at - frustrated
16) place - wishing - in - explained
32) have - feeling - after the - disagreed

CHRISTMAS

TEXTO PARA A PRXIMA QUESTO

If you try to catch a train, a bus or an airplane on December 24th, you may have difficulty in finding a seat. This is the day when
many people are traveling home to be with

Gerald Middleton was a man of 1mildly but


persistently depressive temperament. 2Such
men are not at their best at breakfast, nor

is the week before Christmas their happiest time. Both Larwood and Mrs. Larwood
had learned over the years to respect their
employers melancholy moods by remaining
silent. They did so on this morning. The house on Montpellier Square was as 3noiseless
as a tomb. Mrs. Middleton had rung up from
her house in Marlow as early as eight oclock
to inquire what arrangements her husband
had made for his annual visit to her. Would
he, she asked, arrange to bring down their
son John? Mrs. Larwood had tactfully refused to wake Professor Middleton; she would
see that he phoned Mrs. Middleton during
the morning, she said. The message was placed with the letters and newspapers beside
Geralds plate.
The prospect of speaking to his wife on the
telephone and, even more, of the family
Christmas party 4greatly heightened his depression. He decided not to open his letters
until he had read the news or to open The
Times until he had softened his spirits with
the more popular daily newspaper which always
accompanied it. It was an unwise decision:
the optimistic presentation of decidedly bad
news on the front page turned his passive
gloom into active irritation. On the middle
page was a lengthy article by his son John.
He always swore that he would not read his
sons articles, yet he always did so. Their
5
cocksure and sentimental tone at least lent
justification to his 6hearty dislike of his
younger son, particularly if he accompanied
his reading by a mental image of his wifes
cooing admiration of their sons talent.
Adapted from: WILSON, Angus. Anglo-Saxon
attitudes. Harmondsworth: Penguin, 1968. p. 11

1
0. (UFRGS) What justifies the use of verbs in
the present tense in the second sentence
(ref. 2) is the fact that that sentence
expresses a:
a) generalization.
b) systematization.
c) formalization.
d) simplification.
e) formulation.

E.O. Teste III


TEXTO PARA AS QUESTES 1 A 5
Dengue fever: Millions at risk as a new outbreak of dengue fever sweeps Latin America
Apr 19, 2007 There is no vaccine. There
is also no good way to treat it just fluids
and the hope that the fever will break. At
first it seems like a case of severe flu, but

then the fever rises, accompanied by headaches, excruciating joint pain, nausea and
rashes. In its most serious form, known as
dengue hemorrhagic fever (DHF), it involves internal and external bleeding and can
result in death. Fuelled by climate change,
dengue fever is on the rise again throughout
the developing world, particularly in Latin
America. According to the World Health Organization, dengue is now endemic in more
than 100 tropical and sub-tropical countries
around the world, affecting some 50 million
people a year, mostly in urban or semi-urban areas. A further 2.5 billion, two-fifths
of the worlds population, are considered at
risk. About 500,000 people, many of them
children, are believed each year to develop a
form of DHF serious enough to require treatment in hospital. Worldwide, 2.5% of DHF
cases die; without proper care, the proportion can exceed 20%.
Anyone who survives an infection by one
of the four viruses that cause the disease
gains lifelong immunity from that virus. But
subsequent infection by another variant increases the risk of developing DHF, which is
becoming much more common in Latin America. In Mexico, for example, just one in 50
cases was hemorrhagic six years ago, says
Jos ngel Crdoba Villalobos, Mexicos se-cretary of health. Now one in five is. Last
year just over 500,000 cases of dengue
were reported in Latin America, including
more than 14,000 hemorrhagic cases, 187
of which resulted in death. This year nearly 200,000 dengue cases have already been
reported, including 2,693 cases of DHF. At
least 37 people have died, including 11 in
Paraguay and 17 in Brazil.
The dengue viruses are transmitted to humans through the bite of a female Aedes
mosquito, which acquires the viruses while
feeding, normally on the blood of an infected person. Given that there is no known
preventive treatment or anti-viral cure, the
only practical way to prevent the viruses
spread is to eliminate the Aedes mosquitoes
by preventing them from breeding. In Mexico, the house-to-house programme mounted
by the government to get people during the
rainy season to remove rubbish
and standing water where mosquitoes breed
has been extended year-round with some
success. The number of dengue cases reported this year is well down on last year, but
the rainy season the main breeding time
for the mosquitoes has yet to come.
<www.economist.com>. Adaptado.

41

1. (Unifesp) Os sintomas iniciais da dengue:


a) so febre alta, dor na nuca e vmito intermitente.
b) causam dores nas juntas devido a pequenas
hemorragias.
c) so semelhantes aos de uma gripe forte.
d) so precursores de hemorragia interna que
leva morte.
e) devem ser tratados com ingesto de lquidos
e antitrmicos.
2. (Unifesp) Dengue fever:
a) does not have an efficient vaccine, but there
are some preventive treatments.
b) presents a higher incidence in rural and
scarcely populated areas.
c) is endemic mainly among people who live
near rivers.
d) may be transmitted through human contact
or the Aedes mosquito bite.
e) may be caused by four variants of virus.
3. (Unifesp) Segundo a OMS, a populao
mundial que corre o risco de contrair dengue
de:
a) 2,5 bilhes de pessoas.
b) 50 milhes de pessoas.
c) 50 mil pessoas.
d) 500 mil crianas.
e) 20% da populao de 100 pases.
4. (Unifesp) The DHF, the most serious form of
dengue:
a) does not develop in people who have gained
lifelong immunity from the virus.
b) develops in 20% of cases in Mexico at present.
c) caused the death of 20% of people worldwide.
d) resulted in 2,693 deaths in Latin America.
e) has affected mainly people in Brazil and Paraguay.
5. (Unifesp) A nica maneira prtica de evitar
a disseminao do vrus da dengue por
meio de:
a) tratamentos preventivos de sade pblica
intensivos.
b) programas de cura com antivirais disponveis para as populaes de risco.
c) campanhas governamentais para alertar a
populao sobre os sintomas.
d) medidas para evitar a proliferao do mosquito Aedes.
e) programas domiciliares como o mexicano,
que ocorre durante a poca de chuvas.
42

TEXTO PARA AS PRXIMAS 5 QUESTES


Brow-Raising Mona Lisa Discovery: French
Engineer Uses Technology to Uncover Secrets
of Mysterious Smiler

Oct. 18, 2007

The Mona Lisa may have a few secrets


still, but at least one of the mysteries surrounding Leonardo da Vincis 16th-century
masterpiece has been solved: the lady does
indeed have eyebrows.
Using a high-tech camera, a French engineer
has uncovered what has long been held as a
fact about the painting, that its subject was
painted without eyebrows or eyelashes. That
assumption, according to Pascal Cotte, was
wrong. Cotte, a French photographer and engineer, used a 240 million-pixel camera to
photograph the painting outside of its glass
encasing at the Louvre in Paris. The camera
used infrared technology and strong illumination to scan the painting.
With his camera, the engineer was able to
virtually peel back layers of the painting revealing how it looked when it was originally
painted. The images are currently being shown in San Francisco with a traveling exhibit
about the painter.
In the process of photographing and scanning the images, Cotte discovered that the
Mona Lisas eyebrows and eyelashes were
originally painted on the piece. They disappeared either because of a botched attempt to
clean the painting or because the paints oil
and pigment faded over time. Through the
photographs, Cotte also learned the order in
which da Vinci painted the Mona Lisa and
spotted drawings underneath the paint that
show a change in the position of her fingers.
The images also revealed the touch-up the
painting was given in 1956 to repair damage
it suffered after a rock was thrown at the
painting. The exhibit, Da Vinci: An Exhibition of Genius, runs in San Franciscos
Metre on Center through the end of the year.
Fonte: <www.abcnews.go.com/Technology/

6. (PUCSP) A obra Mona Lisa:


a) teve todos os seus segredos interpretados
pela alta tecnologia computacional, aliada
fotografia.

b) considerada a mais representativa do sculo XVI, de acordo com o fotgrafo francs


Pascal Cotte.
c) a obra-prima de Leonardo da Vinci, mas ainda no se sabe se foi ele mesmo que a pintou.
d) representa uma mulher que, segundo se
acreditava, foi pintada sem clios nem
sobrancelhas.
e) teve seu sorriso misterioso desvendado pelo
uso da tecnologia de alta definio.
7. (PUCSP) Pascal Cotte:
a) cuidadosamente descascou as camadas de
tinta retocada de Mona Lisa para chegar
representao original.
b) fotografou Mona Lisa fora de sua proteo de
vidro, com uma cmera de alta tecnologia.
c) levou Mona Lisa para uma exposio em So
Francisco, juntamente com suas fotografias.
d) descobriu que Mona Lisa foi pintada com a
ajuda de alunos de Leonardo da Vinci e que
suas mos estavam inacabadas.
e) aplicou uma forte iluminao de raios
infravermelhos para avaliar a qualidade dos
pigmentos que Leonardo da Vinci usou.
8. (PUCSP) No trecho do primeiro pargrafo
The lady does indeed have eyebrows. a expresso does indeed indica:
a) contradio.
b) nfase.
c) restrio.
d) interrogao.
e) hiptese.
9. (PUCSP) No trecho do quarto pargrafo do
texto In the process of photographing and
scanning the images, Cotte discovered that
the Mona Lisas eyebrows and eyelashes
were originally painted on the piece. a expresso the piece refere-se a:
a) eyebrows and eyelashes.
b) images.
c) oil and pigment.
d) photographing.
e) Mona Lisa.
10. (PUC-SP) Por meio das fotografias de Pascal
Cotte descobriu-se que:
a) Mona Lisa foi apedrejada em 1956 e sofreu
retoques nos danos.
b) os pigmentos usados por Leonardo da Vinci
eram oleosos e de tons esfumaados.
c) os dedos de Mona Lisa foram retocados para
recuperar a posio original.
d) a expresso caracterstica de Mona Lisa continua a ser um mistrio.
e) Mona Lisa foi pintada em uma determinada sequncia.

E.O. Dissertativo
1. Write the right form of the verbs below in
the third person singular. Follow the models.
Model: asks (to ask) answers (to answer)
a) ____________ (talk)
b) ____________ (play)
c) ____________ (stay)
d) ____________ (pass)
e) ____________ (need)
f) ____________ (take)
g) ____________ (mix)
h) ____________ (echo)
i) ____________ (watch)
j) ____________ (wash)
2. Using the words in parentheses, complete
the text below with the appropriate
tenses. You can use simple present or
present continuous. Follow the models:
Model 1: Where is Sally?
She (to watch) is watching a movie
in bed.
Model 2: What does she do for a living?
She (to write) writes travelbooks
a) Every Monday. Sarah (drive) ______________
her kids to football practice.
b) Usually, I (work) ______________ as a
secretary at IBM, but this summer I (study)
________________ French at a language
school in Lion. That is why I am in Lion.
c) Shhhhh! Be quiet! Joseph (sleep)
________________.
d) Dont forget to take your umbrella. It (rain)
________________.
e) I hate living in Seattle because it (rain,
always) ________________.
f) Im sorry I cant hear what you (say)
________________ because everybody
(talk) ________________ so loudly.
g) Laurence (write, currently) ______________
a book about his adventures in Thayland. I
hope he can find a good publisher when he
is finished.
h) John: Do you want to come over for dinner
tonight?
Sarah: Oh, Im sorry, I cant. I (go) _________
to a movie tonight with some friends.
i) The business cards (be, normally) _________
printed by a company in Cardiff. Their prices
(be) _______________ inexpensive, yet the
quality of their work is quite good.
j) This delicious chocolate (be) _____________
made by a small chocolatier in Geneve, Switzerland.
43

3. Write the interrogative form of the sentences a-l below.


a) The countries are among the best to
b) Brazil is among the best countries to
c) A good gadget relies on good software.
d) Good gadgets rely on good software.
e) There is only one device that
f) There are many devices that
g) Colombia can start a process which
h) The UK could start a process that
i) The actual CEOs might opt for a new
j) New CEOs may opt for an old approach.
k) Members should consider offers
l) Representatives must consider new proposals.
Interrogative
a) ____________________________________
b) ____________________________________
c) ____________________________________
d) ____________________________________
e) ____________________________________
f) ____________________________________
g) ____________________________________
h) ____________________________________
i) ____________________________________
j) ____________________________________
k) ____________________________________
l) ____________________________________
4. Write the negative form of the sentences a-l
above.
Negative
a) ____________________________________
b) ____________________________________
c) ____________________________________
d) ____________________________________
e) ____________________________________
f) ____________________________________
g) ____________________________________
h) ____________________________________
i) ____________________________________
j) ____________________________________
k) ____________________________________
l) ____________________________________
5. Write the interrogative-negative form of the
sentences a-l above.
Interrogative Negative
a) ____________________________________
b) ____________________________________
c) ____________________________________
d) ____________________________________
e) ____________________________________
f) ____________________________________
g) ____________________________________
h) ____________________________________
i) ____________________________________
j) ____________________________________
k) ____________________________________
l) ____________________________________
44

Gabarito
E.O. Teste I
1. D

2. A

3. A

4. C

5. C

6. E

7. A

8. D

9. B

10. A

5. E

E.O. Teste II
1. B

2. C

3. A

4. A

6. A

7. D

8. C

9. 02 + 08 = 10

10. A

E.O. Teste III


1. C

2. E

3. A

4. A

5. D

6. D

7. B

8. B

9. E

10. E

E.O. Dissertativo
1.
a) talks
b) plays
c) stays
d) passes
e) needs
f) takes
g) mixes
h) echoes
i) watches
j) washes
2.

3.

a) drives
b) work - am studying
c) is sleeping
d) is raining
e) always rains
f) are saying - is talking
g) is currently writing
h) am going
i) are normally printed - are
j) is made

Interrogative
a) Are the countries among the best to
b) Is Brazil among the best countries to
c) Does good gadget rely on good software
d) Do good gadgets rely on good softwares
e) Is there only one device that
f) Are there many devices that
g) Can Colombia start a process which
h) Could the UK start a process that

i) May the actual CEOs opt for a new


j) May new CEOs opt for an old approach.
k) Should members consider offers
l) Must representatives consider new proposals
4.

5.

Negative
a) The countries arent among the best to
b) Brazil isnt among the best countries to
c) A good gadget doesnt rely on good software.
d) Good gadgets dont rely on good softwares.
e) There isnt only one device that
f) There arent many devices that
g) Colombia cannot (cant) start a process which
h) The UK couldnt start a process that
i) The actual CEOs might not opt for a new
j) New CEOs may not opt for an old approach.
k) Members shouldnt consider offers
l) Representatives mustnt consider new proposals.

Interrogative-Negative
a) Arent the countries among the best to?
b) Isnt Brazil among the best countries to?
c) Doesnt a good gadget rely on good software?
d) Dont good gadgets rely on good softwares?
e) Isnt there only one device that?
f) Arent there many devices that?
g) Cant Colombia start a process which?
h) Couldnt the UK start a process that?
i) May the actual CEOs not opt for a new?
j) May new CEOs not opt for an old approach. ?
k) Shouldnt members consider offers?
l) Mustnt representatives consider new proposals?

45

Class 4

Focus on the past

wavebreakmedia/Shutterstock

A ideia desta unidade fazer uma coletnea das formas verbais da lngua inglesa que, de uma maneira ou de outra
so compreendidas como formas verbais dos tempos passados do portugus. Vrios tempos e formas verbais da
lngua inglesa, aps uma rpida negociao de seu significado enquanto se executa a leitura, chegam para o
leitor como se estivessem no passado em portugus, independente de como so chamados em ingls. Portanto, a
abordagem aqui proposta consiste em agrupar essas formas e tempos verbais do ingls. So elas: simple past, to
be, there to be e o modal verb can.

Simple past (regra geral)


A grande maioria dos verbos em ingls segue as regras apresentadas nesse item, portanto muita ateno. Quando
falamos em simple past, os verbos da lngua inglesa dividem-se em duas categorias: verbos regulares e verbos
irregulares.

Regular verbs (verbos regulares)


Os verbos regulares so os prediletos dos estudantes brasileiros por serem os mais simples. Para obter o passado
dos verbos irregulares, basta acrescentar o sufixo ED a sua base form (infinitivo sem a partcula to), no importando
quem seja o sujeito. Observe os exemplos a seguir:
to need (precisar, necessitar)
1. I needed to be faster. (Eu precisei ser mais rpido(a).)
2. You needed to be faster. (Voc precisou ser mais rpida(o).)
3. He needed to be faster. (Ele precisou ser mais rpido,)
4. She needed to be more skilful. (Ela precisou ser mais habilidosa.)
5. It needed to be cheaper. (Isto precisava ser mais barato.)
6. We needed to be more skilful. (Ns precisamos ser mais habilidosos (as).)
7. You needed to be more skilful. (Vocs precisaram ser mais habilidosos.)
8. They needed to be more skilful . (Elas(es) precisaram ser mais habilidosas(os).)

Irregular verbs (verbos irregulares)


Lidar com os verbos irregulares talvez seja a maior dificuldade para o estudante brasileiro quando esse lida com o
simple past da lngua inglesa. Neste ponto a lngua inglesa se assemelha portuguesa, apresentando formas verbais variadas (da o termo irregular), e assim como para o aprendizado do portugus, tambm se torna necessrio
memorizar tais formas. Porm lembre-se que h apenas uma forma de passado para cada verbo, no importando
qual seja o sujeito. Seguem-se apenas alguns exemplos. (Uma tabela de verbos irregulares est inserida no trmino
desta unidade.)
1. to take took (past)
I took the bus to school. (Eu peguei o nibus para a escola)
She took the bus to school. (Ela pegou o nibus para a escola)
We took the bus to work. (Ns tomamos o nibus para o trabalho)
2. to go went (past)
You went to college. (Voc foi para a universidade)
He went too far. (Ele foi longe demais.)
They went home together. (Eles foram juntos para casa)
47

3. to eat ate (past)


I ate the whole cake. (Eu comi o bolo inteiro)
The dog ate its food. (O cachorro comeu a comida dele)
We ate the pizza together. (Ns comemos juntos a pizza)

Formas Negativas
Para obter as formas negativas dos verbos no simple past, deve-se acrescentar, aps o sujeito e antes do verbo
principal, o verbo auxiliar did mais a partcula not, no importando se o verbo regular ou irregular. Observe os
exemplos:
1. to need
I did not (didnt) need to be more courageous than usual. (Eu no precisei ser mais corajosa do que de costume.)
He didnt (did not) need to be more skilful.(Ele no precisou ser mais habilidoso)
2. to take
I didnt take the ferry. (Eu no peguei a balsa)
They didnt take the train. (Eles(as) no pegaram o trem.
3. to go
You didnt go to work yesterday. (Vocs no foram para o trabalho ontem)
We didnt go home together. (Ns no fomos para casa juntos)

Formas interrogativas
Para se verter uma frase para a forma interrogativa quando um verbo est no simple past deve-se introduzir o verbo
auxiliar did antes do sujeito da orao. Observe que o verbo principal deve permanecer no base form (infinitivo sem
to). Veja os exemplos a seguir:

Exemplos:
1. Did you need to be more respectful? (Voc precisou ser mais respeitoso(a)?)
2. Did she take the bus to school? (Ela pegou o nibus para a escola?)
3. Did they go home early? (Elas foram cedo para casa?)

To be
O verbo to be tem duas formas de passado: was (I, he, she, it) e were (you, we, they).
Observe os exemplos:
1. I was at home yesterday. (Eu estava em casa ontem).
2. You were at work yesterday, (Voc estava no trabalho ontem.)
3. He was an American politician. (Ele era um poltico estadunidense.)
4. She was an English lawmaker. (Ela era uma legisladora inglesa.)
5. The car (it) was here two hours ago. (O carro estava aqui h duas horas atrs.
6. We were busy yesterday. (Ns estvamos ocupados(as) ontem.)
7. They were sold in Japan in 2010. (Eles foram vendidos no Japo em 2010.)
48

Formas interrogativas
Para obter as formas negativas de passado do verbo to be, deve-se inverter a posio do verbo com o sujeito da
orao. Observe alguns exemplos.

Exemplos:
1. Were you at work yesterday? (Voc estava no trabalho ontem?)
2. Was it here two hours ago? (Aquilo/isso estava aqui h duas horas?)
3. Were they sold in Japan in 2010? (Eles foram vendidos no Japo em 2010?)

Formas negativas
Para se obter as formas negativas do verbo to be no passado, deve-se introduzir a partcula not aps o verbo.
Observe as contraes possveis.
1. I was not (wasnt) at home yesterday. (Eu no estava em casa ontem).
2. You were not (werent)at work yesterday, (Vocs no estavam no trabalho ontem.)
3. He wasnt an American politician. (Ele no foi um poltico estadunidense.)

There to be
Assim como no presente, existem duas formas de passado para o verbo there to be: there was (para passado singular) e there were (para passado plural). Observe os exemplos a seguir.
1. There was a school near here. (Havia/existia uma escola perto daqui.)
2. There were many schools here. (Haviam/existiam muitas escolas perto daqui.)

Formas negativas
Para obter as formas negativas de passado do there to be, deve-se acrescentar a partcula not s formas afirmativas. Veja os exemplos:
1. There was not (wasnt) any school near here. (No Havia/existia uma escola perto daqui.)
2. There were not (werent) many schools here. (No Haviam/existiam muitas escolas perto daqui.)

Formas interrogativas
Ao inverter a posio da forma verbal was ou were com o pronome there, obtm-se a forma interrogativa do verbo

there to be. Observe os exemplos:


1. Was there a person waiting here? (Havia/existia uma pessoa esperando aqui?)
2. Were there some people waiting here? (Haviam/existiam algumas pessoas esperando aqui?)
49

Can
Os modal verbs sero tratados separadamente em um unidade especfica, mas para que voc j tenha uma referncia para sua leitura, seria importante que soubesse que o verbo can tem todas as suas formas de passado (includas
as de subjuntivo) concentradas na palavra could, independente qual seja o sujeito envolvido.
1. Sophia could play the piano very well when she was a child. (Sophia podia tocar piano muito bem quando
ela era criana.)
2. Two years ago we could buy a house, but today this is impossible. (H dois anos atrs ns podamos comprar
uma casa, mas hoje isto impossvel.)
3. I would go home if I could. (Eu iria para casa se eu pudesse.)

Used to
Ao lidar com textos em lngua inglesa, o estudante sempre se depara com um problema tpico de quem tem portugus como lngua materna: as formas de passado do ingls so correspondentes aos nossos verbos do pretrito
perfeito ou do pretrito imperfeito? Como devo entender uma frase como I played very well? Eu joguei muito
bem ou Eu jogava muito bem?
As frases grafadas em simple past em ingls geralmente correspondem ao que ns nomeamos como pretrito perfeito (ex: joguei, comi, bebi). Para se obter o mesmo significado das nossas sentenas no pretrito imperfeito
(ex: jogava, comia, bebia), a lngua inglesa faz uso da estrutura used to. De forma geral, ela utilizada para expressar uma ao que era verdadeira no passado, mas que no mais. Observe os exemplos:
1. I used to play football very well. (Eu costumava jogar/jogava futebol muito bem)
2. She used to be a great dancer. (Ela era/costumava ser uma grande danarina)
3. Did you use to travel to the country when you lived abroad? (Voc costumava viajar/viajava para o interior
quando voc morou no exterior?)
4. I didnt use to like it, but now I do. (Eu no costumava gostar disso, mas agora eu gosto)

Past continuous
Muito similar ao caso previamente estudado do present continuous, o past continuous da lngua inglesa um
tempo verbal utilizado para expressar uma ao que estava acontecendo simultaneamente outra, no passado
e formado por uma forma de passado do verbo to be (was ou were) mais um outro verbo de ao acrescido do
sufixo ING (gerund). Estude os exemplos:
1. When you called, I was taking a shower. (Quando voc ligou eu estava tomando banho)
2. Were you sleeping when I called you yesterday? (Voc estava dormindo quando eu te liguei ontem?
3. She wasnt paying attention when the teacher assigned her homework. (Ela no estava prestando ateno
quando o professor passou a lio de casa.)
Apndice #1: formas interrogativa-negativas
As vrias formas verbais estudadas nesta unidade (simple past, to be, there to be e can) apresentam, alm das
formas apresentadas (afirmativa, negativa e interrogativa) uma quarta forma: a interrogativa-negativa. As formas
interrogativa-negativas so obtidas a partir das formas negativas, cada um dos verbos seguindo suas prprias
regras. Estude os exemplos abaixo:
50

1. Didnt they like their new responsibilities? (Eles(as) no gostaram de suas novas responsabilidades?)
2. Didnt you talk to him? (Voc no conversou com eles?)
3. Werent you at work yesterday? (Voc no estava no trabalho ontem?)
4. Wasnt there a person waiting here? (No havia uma pessoa esperando aqui?)
5. Couldnt you ride bicycles when you were 6? (Voc no conseguia andar de bicicleta quando voc tinha 6
anos de idade?)
Apndice #2: Uso enftico do auxiliar did
Alm das formas interrogativas e negativas, existe outro uso para o auxiliar did: o enftico. Estude os exemplos a
seguir.
1. I liked the movie (Eu gostei do filme) I did like the movie (Eu gostei muito do filme)
2. She told you not to be late (Ela te pediu que no se atrasasse) She did tell you not to be late (Ela realmente
te pediu/foi muito clara ao te pedir que no se atrasasse.)
Apndice #3: Tabela dos 150 verbos mais comuns da lngua inglesa (regulares e Irregulares). A 2.a coluna apresenta as formas de passado desses verbos, e a 3.a coluna as formas de particpio passado (past participle).
infinitive

simple past

past participle

traduo do infinitivo

1. be

Was/were

been

ser, estar

2. have

Had

had

ter

3. do

Did

done

fazer

4. say

Said

said

dizer, falar

5. get

Got

got, gotten

conseguir, obter

6. make

Made

made

fazer

7. go

Went

gone

ir

8. see

Saw

seen

ver

9. know

Knew

known

saber, conhecer

10. take

Took

taken

tomar, pegar

11. think

Thought

thought

pensar, achar

12. come

Came

come

vir

13. give

Gave

given

dar

14. look

Looked

looked

olhar

15. use

Used

used

usar, utilizar

16. find

Found

found

encontrar, descobrir

17. want

Wanted

wanted

querer, procurar

18. tell

Told

told

dizer

19. put

Put

put

pr, colocar

20. mean

Meant

meant

significar, querer dizer

21. become

Became

become

tornar-se

22. leave

Left

left

partir, sair

23. work

Worked

worked

trabalhar

24. need

Needed

needed

precisar, necessitar

25. feel

Felt

felt

sentir

26. seem

Seemed

seemed

parecer

27. ask

Asked

asked

perguntar, pedir

28. show

Showed

showed, shown

mostrar, demonstrar

29. try

Tried

tried

tentar

30. call

Called

called

chamar, ligar

31. keep

Kept

kept

manter, continuar

51

infinitive
32. provide

52

simple past
Provided

past participle
provided

traduo do infinitivo
fornecer, prover

33. hold

Held

held

segurar, conter

34. turn

Turned

turned

virar, mudar

35. follow

Followed

followed

seguir

36. begin

Began

begun

iniciar, comear

37. bring

Brought

brought

trazer

38. like

Liked

liked

gostar

39. forget

Forgot

forgotten

esquecer

40. help

Helped

helped

ajudar

41. start

Started

started

comear

42. run

Ran

run

correr, administrar

43. write

Wrote

written

escrever

44. set

Set

set

preparar, arrumar

45. move

Moved

moved

mover, mudar

46. play

Played

played

jogar, brincar, tocar, executar

47. pay

Paid

paid

pagar

48. hear

Heard

heard

ouvir, escutar

49. include

Included

included

incluir

50. believe

Believed

believed

acreditar

51. allow

Allowed

allowed

permitir

52. meet

Met

met

encontrar, conhecer

53. lead

Led

led

levar, conduzir, liderar

54. live

Lived

lived

viver, morar

55. stand

Stood

stood

levantar

56. happen

Happened

happened

acontecer, ocorrer

57. carry

Carried

carried

carregar

58. talk

Talked

talked

conversar

59. appear

Appeared

appeared

aparecer

60. produce

Produced

produced

produzir

61. sit

Sat

sat

sentar

62. offer

Offered

offered

oferecer

63. consider

Considered

considered

considerar

64. expect

Expected

expected

esperar, ter expectativa

65. suggest

Suggested

suggested

sugerir

66. let

Let

let

deixar, permitir

67. read

Read

read

ler

68. require

Required

required

requerer

69. continue

Continued

continued

continuar

70. lose

Lost

lost

perder

71. add

Added

added

adicionar

72. change

Changed

changed

mudar

73. fall

Fell

fallen

cair

74. remain

Remained

remained

permanecer, remanecer

75. remember

Remembered

remembered

lembrar

76. buy

Bought

bought

comprar

77. speak

Spoke

spoken

falar, conversar

78. stop

Stopped

stopped

parar

79. send

Sent

sent

enviar, mandar

infinitive
80. receive

simple past
Received

past participle
received

traduo do infinitivo
receber

81. decide

Decided

decided

decidir

82. win

Won

won

ganhar, vencer

83. understand

Understood

understood

entender, compreender

84. describe

Described

described

descrever

85. develop

Developed

developed

desenvolver

86. agree

Agreed

agreed

concordar

87. open

Opened

opened

abrir

88. reach

Reached

reached

alcanar

89. build

Built

built

construir

90. involve

Involved

involved

envolver

91. spend

Spent

spent

gastar, passar o tempo

92. return

Returned

returned

retornar

93. draw

Drew

drawn

desenhar, retirar, empatar

94. die

Died

died

morrer

95. hope

Hoped

hoped

esperar, ter esperana

96. create

Created

created

criar

97. walk

Walked

walked

caminhar, andar

98. sell

Sold

sold

vender

99. wait

Waited

waited

esperar

100. cause

Caused

caused

causar

101. pass

Passed

passed

passar

102. lie

Lied

lied

mentir

103. accept

Accepted

accepted

aceitar

104. watch

Watched

watched

assistir

105. raise

Raised

raised

elevar, aumentar

106. base

Based

based

basear

107. apply

Applied

applied

aplicar

108. break

Broke

broken

quebrar, interromper

109. explain

Explained

explained

explicar

110. learn

Learned/learnt

learned, learnt

aprender

111. increase

Increased

increased

aumentar, incrementar

112. cover

Covered

covered

cobrir

113. grow

Grew

grown

crescer, desenvolver

114. claim

Claimed

claimed

alegar, afirmar, clamar

115. report

Reported

reported

reportar, descrever

116. support

Supported

supported

apoiar, dar suporte

117. cut

Cut

cut

cortar

118. form

Formed

formed

formar

119. stay

Stayed

stayed

ficar, permanecer

120. contain

Contained

contained

conter

121. reduce

Reduced

reduced

reduzir

122. establish

Established

established

estabelecer

123. join

Joined

joined

juntar, juntar-se

124. wish

Wished

wished

desejar

125. achieve

Achieved

achieved

alcanar, conquistar

126. seek

Sought

sought

procurar

127. choose

Chose

chosen

escolher

53

infinitive
128. deal

54

simple past
Dealt

past participle
dealt

traduo do infinitivo
lidar, negociar

129. face

Faced

faced

encarar, enfrentar

130. fail

Failed

failed

falhar

131. serve

Served

served

servir

132. end

Ended

ended

terminar

133. kill

Killed

killed

matar

134. occur

Occurred

occurred

ocorrer

135. drive

Drove

driven

dirigir, conduzir

136. represent

Represented

represented

representar

137. rise

Rose

risen

levantar, aumentar

138. discuss

Discussed

discussed

discutir, dialogar

139. love

Loved

loved

amar, adorar

140. pick

Picked

picked

pegar, escolher

141. place

Placed

placed

colocar, pr

142. argue

Arqued

argued

discutir

143. prove

Proved

proved

provar

144. wear

Wore

worn

vestir

145. catch

Caught

caught

pegar, apanhar

146. enjoy

Enjoyed

enjoyed

usufruir, curtir

147. eat

Ate

eaten

comer

148. introduce

Introduced

introduced

introduzir, apresentar

149. enter

Entered

entered

entrar

150. present

Presented

presented

apresentar

E.O. Teste I
1. (Unesp) Assinale a alternativa correta:
_____ he do the work last night?
a) Do
b) Does
c) Have
d) Has
e) Did
2. (Unesp) Assinale a alternativa que preenche
corretamente a lacuna da frase a seguir:
Can you tell me where __________ my bike
yesterday?
a) did you leave
b) you left
c) you have left
d) you leaved
e) did you left
3. (Unesp) Assinale a alternativa que preenche
a lacuna da frase a seguir corretamente.
The mayor __________ it difficult to refuse.
a) find
b) finding
c) founded
d) found
e) to find
4. Assinale a alternativa que preenche corretamente a lacuna da frase a seguir:
Hey, take it easy! I __________ to fool you.
a) werent trying
b) wasnt trying
c) wasnt try
d) wasnt tried
e) didnt trying
5. (Uel) When I asked Jim if he liked his job he
replied that he _____.
a) did
b) does
c) do
d) doing
e) has done
6. Assinale a alternativa que preenche corretamente a lacuna da frase a seguir:
Tell me, why _____ you so angry?
a) was
b) wasnt
c) were
d) did be
e) didnt

7. A forma negativa da frase He tore the page


of the book :
a) He tore not the page of the book.
b) He doesnt tear the page of the book.
c) He didnt tore the page of the book.
d) He didnt tear the page of the book.
e) He did tore not the page of the book.
8. A frase She put it on that big shelf pode
ser uma resposta pergunta:
a) Where does she put her cook book?
b) Where did she puts her cook book?
c) Where does she puts her cook book?
d) Where did she put her cook book?
e) Where didnt she put her cook book?
9. A forma interrogativa negativa da frase
Otto was a great student :
a) Was it Otto a great student?
b) Wasnt Otto a great student?
c) Were Otto a great student?
d) Did Otto is a great student?
e) Did Otto be a great student?
10. Valria __________ that her husbands friends
__________ for dinner.
a) doesnt know were coming
b) didnt know was coming
c) didnt know wasnt coming
d) didnt knew were coming
e) didnt know were coming

E.O. Teste II
TEXTO PARA AS PRXIMAS 6 QUESTES.
In 1960s America there was a white flight
to the suburbs, which provoked a deterioration of city centers. In the 70s and 80s the
death of heavy industry emptied once proud
cities like Manchester and Glasgow. Social
and economic change has been wreaking
chaos with cities for a long time, but each
instance was usually thought of as an isolated event or at least a regional disease.
Thats no longer true. As birthrates in more
and more countries decline, shrinking-city
syndrome is becoming a worldwide crisis.
Aging countries are getting hit the worst. In
Russia a combination of very low birthrates,
decreased life expectancy and the collapse of
the communist era is affecting the country badly. Seven major Russian cities were
shrinking in 1990; by 2000 the number had
soared to 93. In Japan, hundreds of small and
midsize cities are thinning out. Even in China,
the low birthrate means that coastal megacities like Shanghai are growing at the expense of dozens of less successful metropolises.
55

Today, while hundreds of millions of Asians


and Africans are just starting to move to cities, one quarter of the worlds centers are
declining in population twice the number a
decade ago.
Wouldnt less-crowded cities be a good thing?
Definitely not, according to Shrinking Cities, a new exhibit in Berlin that compares
city shrinkage across the world. In places like
Detroit and Liverpool, shuttered stores and
abandoned houses have led to increased violence. A 50 percent drop in the birthrate has
killed entire sectors of the economy in cities
that used to be located in East Germany.
(Adaptado de Newsweek, September 27, 2004.)

1. (UFSCar) Nas dcadas de 1970 e 1980:


a) duas cidades britnicas pagaram com o esvaziamento urbano pela perda de suas indstrias de base.
b) orgulhosas cidades da Gr-Bretanha viram
sua populao aumentar devido instalao
de indstrias pesadas.
c) a instalao de indstrias pesadas em cidades da Gr-Bretanha provocou um esvaziamento da populao.
d) Manchester e Glasgow exibiam, com orgulho, uma taxa de crescimento diretamente
ligado a um surto da indstria de base.
e) duas cidades britnicas orgulhavam-se por
ter conseguido se livrar de suas indstrias
de base.
2. (UFSCar) A Rssia:
a) tida como um pas em rejuvenescimento,
fato provocado, entre outras coisas, pela
queda do comunismo.
b) est conhecendo um envelhecimento populacional graas, entre outros fatores, ao colapso por que passa a indstria comunista.
c) combina diferentes fatores para evitar que
caia a taxa de expectativa de vida de sua
populao.
d) conheceu, em sua era comunista, taxas de
natalidade muito baixas, o que s agora est
sendo contornado.
e) um pas idoso, em que, entre outras coisas,
se combina uma taxa de natalidade baixa e
uma diminuio da expectativa de vida.
3. (UFSCar) Segundo a mostra Shrinking Cities:
a) o esvaziamento urbano, tal como o de Detroit e
Liverpool, levou a um crescimento da violncia.
b) cidades como Detroit e Liverpool, devido a
um aumento da violncia, conheceram uma
degradao urbana.
c) fica comprovado que cidades menos populosas constituem algo positivo, apesar da violncia urbana.
56

d) Berlim uma cidade que, como Detroit e


Liverpool, exibe casas abandonadas e lojas
com vidraas quebradas.
e) cidades com menos populao, como as que se
localizavam na ex-Alemanha Oriental, conheceram um decrscimo da violncia urbana.
4. (UFSCar) Na dcada de 1960:
a) as cidades americanas encolheram porque os
brancos deixaram de viver em seus subrbios.
b) houve um acentuado crescimento da populao nas cidades americanas, o que provocou
a deteriorao de seus centros.
c) a populao branca passou a procurar os subrbios das cidades americanas para ali residir.
d) os subrbios de cidades americanas comearam a se deteriorar devido procura da
populao branca por essas reas.
e) os centros e os subrbios das cidades americanas comearam a se deteriorar.
5. (UFSCar) Segundo o texto:
a) hoje no mais se acredita que as mudanas
econmicas e sociais sejam responsveis pelo
caos urbano.
b) os problemas urbanos provocados por mudanas na economia e na sociedade eram
vistos como um fenmeno mundial.
c) o isolamento regional levava as cidades a
conhecerem o caos provocado por mudanas
econmicas e sociais.
d) hoje no mais tido como verdadeiro que
o caos social de origem econmica e social
constitua algo isolado.
e) o caos urbano ainda tido como uma doena regional que, em cada caso, provoca um
isolamento econmico e social.
6. (UFSCar) Na China:
a) o crescimento de megacidades como Xangai
explica-se por sua localizao junto costa.
b) Xangai, uma das megacidades do pas, incentivou o crescimento de dezenas de outras
metrpoles localizadas junto costa.
c) dezenas de cidades tiveram menos sucesso
que Xangai em termos econmicos, embora
continuem a aumentar suas populaes.
d) Xangai, uma das doze megacidades do pas,
apresenta uma das maiores taxas de crescimento bem-sucedido.
e) a expanso populacional de grandes cidades
costeiras como Xangai se deu custa do esvaziamento de outras metrpoles.
TEXTO PARA AS PRXIMAS 4 QUESTES
BATTLING THE BLOODLINES
Its the small things the Brazilians do that annoy some Japanese in Toyota City. The immigrants dont throw their garbage where they
are supposed to. They gather outside and play

loud music at night. They play a strange card


game that involves yelling Truco! at the top
of their lungs. To Japanese in one densely populated public housing complex, it feels as if the
foreigners are closing in on them, the smoke
from the barbecues suffocating them, the Latin
music drowning out an imagined tranquility.
Ten years ago there were 200 Brazilians in the
complex. Today there are 3,500. The sidewalks
are getting narrower, said a Japanese woman
as she maneuvered a grocery cart through a gathering of Brazilian families. Theres no room
for us anymore, said her friend.
Foreigners of any stripe can be upsetting in
Japan, where conformity is a national creed
and wa, the concept of harmony, is integral to
maintaining stability and peace in a country
of 126 million people crowded onto four islands. I dont think its a good idea to concentrate Brazilians in one place, said Masae Matsui.
Two years ago, a residents association Matsui
headed proposed restricting the number of
foreigners in his public housing complex; in
April, he was elected to the city assembly.
The dark side of wa, the part that excludes
outsiders, erupted into violence earlier this
summer in Toyota City, home to thousands
of workers of the carmaker Toyota, its subsidiaries and suppliers. After a dispute with
a noodle vendor got out of hand, about 100
supporters of a rightwing nationalist group
paraded around the public housing complex
where 3,500 Brazilians live. They shouted through a loud-speaker, Foreigners go
home, taunted the Brazilians to come out
and fight and waved metal pipes in the air.
Time, August 9, 1999, p. 19.

7. (Fei) Segundo o texto, qual foi o fato concreto que deu origem s cenas de violncia em
Toyota City?
a) Greve de milhares de trabalhadores, subsidirias e fornecedores de Toyota.
b) Incidente com um vendedor de macarro.
c) Limite de 200 moradores brasileiros por con
junto habitacional.
d) Uma briga de brasileiros com um grupo nacionalista japons.
e) O fato de os brasileiros no compreenderem o wa,
conceito de harmonia fundamental no Japo.
8. (Fei) O texto revela:
a) grande estima que os japoneses tm pelos
brasileiros.
b) que os japoneses apreciam msica latina.
c) dificuldades de relacionamento entre imigrantes brasileiros e residentes japoneses.
d) que as caladas das cidades de Toyota so
muito estreitas.
e) que a populao do Japo aumentou consideravelmente nos ltimos dez anos.

9. (Fei) Theres no room for us anymore


(1o pargrafo). Qual a melhor traduo?
a) No h mais espao para ns.
b) Perdemos o rumo para sempre.
c) No h mais quartos disponveis.
d) No h companheiros de quartos para ns.
e) No temos mais direitos.
10. (Fei) Na expresso The sidewalks are GETTING narrower (1o pargrafo), voc poderia
substituir GETTING por:
a) closing.
b) having.
c) beginning.
d) coming.
e) becoming.

E.O. Teste III


Instruo: Leia o texto Status of same-sex
marriage para responder s questes de nmeros 1 a 5.
STATUS OF SAME-SEX MARRIAGE IN SOUTH
AMERICA
Argentina
The Autonomous City of Buenos Aires (a federal district and capital city of the republic) allows same-sex civil unions.
The province of Rio Negro allows same-sex
civil unions, too. Legislation to enact samesex marriage across all of Argentina was approved on July 15, 2010.
Brazil
A law that would allow same-sex civil unions
throughout the nation has been debated.
Until the end of the first semester of 2010
the Supremo Tribunal Federal had not decided about it.
Colombia
The Colombian Constitutional Court ruled in
February 2007 that same-sex couples are entitled to the same inheritance rights as heterosexuals in common-law marriages. This
ruling made Colombia the first South American nation to legally recognize gay couples.
Furthermore, in January 2009, the Court ruled that same-sex couples must be extended
all of the rights offered to cohabitating heterosexual couples.
Ecuador
The Ecuadorian new constitution has made
Ecuador stand out in the region. Ecuador has
become the first country in South America
where same sex civil union couples are legally recognized as a family and share the
same rights of married heterosexual couples.
57

Uruguay
Uruguay became the first country in South
America to allow civil unions (for both opposite-sex and same-sexcouples) in a national
platform on January 1, 2008. Children can
be adopted by same-sex couples since 2009.
(http://en.wikipedia.org/. Adaptado.)

1. Assinale a alternativa correta.


a) Segundo o texto, os pases nos quais os direitos de casais heterossexuais e de casais
homossexuais so os mesmos so o Equador
e a Colmbia.
b) De acordo com as informaes do texto, entende-se que unies civis entre pessoas do
mesmo sexo so legais em todos os pases da
Amrica do Sul.
c) De acordo com o texto, entende-se que, dentre os pases da Amrica do Sul, somente no
Brasil ainda no se permitem unies civis
entre pessoas do mesmo sexo.
d) O pas da Amrica do Sul onde as unies civis entre pessoas do mesmo sexo demoraram
mais para ser legalizadas o Uruguai.
e) As unies civis entre pessoas do mesmo sexo, vlidas em todo o territrio brasileiro, foram aprovadas em 2010 pelo Supremo Tribunal Federal.
2. Assinale a alternativa correta.

a) A Colmbia foi a ltima nao sul-americana a aprovar a unio civil de casais htero
ou homossexuais.
b) A Argentina foi a segunda nao sul-americana a reconhecer os direitos dos casais do
mesmo sexo.
c) O Equador foi o pas sul-americano que menos se empenhou para reconhecer os direitos dos homossexuais.
d) O Uruguai foi o primeiro pas sul-americano
a aprovar unies civis de casais htero e
homossexuais.
e) O Brasil no tem demonstrado nenhum interesse no reconhecimento dos direitos dos
casais homossexuais.
3. Assinale a alternativa na qual todos os termos se caracterizam como vocabulrio especfico da rea legislativa.
a) Across, approved, platform, ruling.
b) Constitution, legislation, share, throughout.
c) Across, allow, platform, rights.
d) Constitution, entitled, rights, ruling.
e) Across, extended, recognize, share.
4. Assinale a alternativa na qual todas as palavras so formas verbais relativas ao passado.
a) Adopted, become, decided, recognized, ruled.
b) Adopted, allow, become, recognized, ruled.
c) Approved, became, been, decided, ruled.
d) Allow, approved, became, decided, may.
e) Can, debated, entitled, made, offered.
58

5. Com base nas informaes do texto, o que


podemos inferir a respeito da situao atual
dos casais do mesmo sexo na Argentina?
a) As unies civis entre pessoas do mesmo sexo
so vlidas somente em Buenos Aires e na
provncia de Rio Negro.
b) Os casais do mesmo sexo provavelmente ainda no tm todos os direitos dos casais heterossexuais.
c) A provncia de Rio Negro foi a regio onde
unies civis entre pessoas do mesmo sexo
foram aprovadas mais recentemente.
d) Em Buenos Aires as leis para unies civis entre pessoas do mesmo sexo so diferentes do
restante do pas.
e) Os casais homossexuais podero ter exatamente os mesmos deveres dos casais heterossexuais.
TEXTO PARA AS PRXIMAS 3 QUESTES
PHYSICIAN, HEAL THYSELF
When the topic of medical errors comes up,
people usually think of the most outrageous
mistakes: the Florida doctor, for example,
who amputated the wrong leg of his diabetic
patient or the Colorado boy who died during
ear surgery because his anesthesiologist allegedly fell asleep. Though much publicized,
these egregious errors are relatively rare.
Far more common are mental lapses or simple slip-ups that sometimes lead to disaster.
For instance, a harried doctor misdiagnoses
a patient because he cannot spend more
than five minutes examining her. Or a pharmacist dispenses the wrong drug because he
misreads the doctors handwriting on the
prescription. Last fall the national Academy
of Sciences Institute of Medicine released
a report entitled To Err is Human, which
claimed that between 44,000 and 98,000
Americans die every year as a result of medical errors. Even the lower estimate would
make errors the eighth leading cause of death, striking down more people than motor
vehicle accidents or breast cancer. The report
outlined a series of recommendations aimed
at reducing medical errors by 50 percent
over the next five years. It advocated an approach similar to that used by the aviation
industry, with the focus on collecting information on errors and using this knowledge
to devise safer systems and procedures. President Bill Clinton has already endorsed the
report, and Congress may act on several of
its recommendations this year.
(Scientific American, May 2000.)

6. (UFPR) According to the text, it is correct to say:


01) Health professionals become outraged when
the issue of medical errors emerges.
02) Surgeons make more mistakes than anesthesiologists.
04) The difficulty in reading the physicians handwriting on a prescription may lead a pharmacist to hand out the incorrect medication.
08) Anesthesiologists are responsible for most
medical errors.
16) Doctors may make mistakes when they examine their patients in a hurry.

even making someones life a little better.


www.resourcelink.hp.comPossibilities made fresh daily.
e-services solutions from hp. HEWLETT PACKARD

7. (UFPR) The text contains:


01) examples of medical errors that have received a lot of attention in the media.
02) data related to the number of lawsuits brought
about by medical errors.
04) data related to the number of deaths caused
by medical errors.
08) examples of problems that may result from minor errors on the part of health professionals.
16) data related to the social class of the victims
of medical errors.
8. (UFPR) According to the text, it is correct to say:
01) The report To Err is Human is based on
data from the last five years.
02) The report entitled To Err is Human contains recommendations on how to reduce
the incidence of medical errors.
04) Due to the adoption of the report recommendations, the number of medical errors
has dropped 50%.
08) On the basis of the report, the American
Congress has already passed new legislation
intended to reduce the number of deaths resulting from medical errors.
16) The number of deaths due to medical errors
is higher than the number of deaths resulting from motor vehicle accidents.
TEXTO PARA AS PRXIMAS 2 QUESTES
One hundred billion pounds of food go to
waste every year in America, while 35 million
people remain hungry. The problem is not
lack of food, but lack of communication how
to get food where its needed most. Resource Link.org is an hp-designed and operated
e-service portal which connects food manufacturers with Americas Second Harvest, the
countrys largest food relief agency. The system not only automatically locates surplus
food, but dynamically interacts with shipping
companies to get that food to the right place. A process which used to take weeks now
takes minutes. This is the power of e-services
connecting companies and their services
in inventive ways and, in some cases, maybe

9. (UFPR) O objetivo do servio prestado pelo


portal ResourceLink.org :
01) Reduzir o desperdcio de alimentos.
02) Agilizar a entrega de alimentos para pessoas
necessitadas.
04) Melhorar os sistemas de armazenamento de
alimentos.
08) Evitar as perdas de produtos perecveis durante o transporte.
16) Facilitar a compra de alimentos via internet.
10. (UFPR) According to the text, it is correct to say:
01) ResourceLink.org is the name of an organization that operates food manufacturing
companies.
02) The ad attempts to show how easy it is to
buy food on-line.
04) The ad stresses the fact that the best shipping
companies use HP software in all their operations.
08) The ad attempts to show that HP is active in
dealing with social problems.
16) The ad shows some of the possibilities of
e-services solutions from HP.

59

E.O. Dissertativo
1. Write the past form (simple past) of the verbs in the chart below.
infinitive

60

simple past

past participle

traduo do infinitivo

1. be

been

ser, estar

2. have

had

ter

3. do

done

fazer

4. say

said

dizer, falar

5. get

got, gotten

conseguir, obter

6. make

made

fazer

7. go

gone

ir

8. see

seen

ver

9. know

known

saber, conhecer

10. take

taken

tomar, pegar

11. think

thought

pensar, achar

12. come

come

vir

13. give

given

dar

14. look

looked

olhar

15. use

used

usar, utilizar

16. find

found

encontrar, descobrir

17. want

wanted

querer, procurar

18. tell

told

dizer

19. put

put

pr, colocar

20. mean

meant

significar, querer dizer

21. become

become

tornar-se

22. leave

left

partir, sair

23. work

worked

trabalhar

24. need

needed

precisar, necessitar

25. feel

felt

sentir

26. seem

seemed

parecer

27. ask

asked

perguntar, pedir

28. show

showed, shown

mostrar, demonstrar

29. try

tried

tentar

30. call

called

chamar, ligar

31. keep

kept

manter, continuar

32. provide

provided

fornecer, prover

33. hold

held

segurar, conter

34. turn

turned

virar, mudar

35. follow

followed

seguir

36. begin

begun

iniciar, comear

37. bring

brought

trazer

38. like

liked

gostar

39. forget

forgotten

esquecer

40. help

helped

ajudar

41. start

started

comear

42. run

run

correr, administrar

43. write

written

escrever

44. set

set

preparar, arrumar

45. move

moved

mover, mudar

46. play

played

jogar, brincar, tocar, executar

infinitive

simple past

past participle

traduo do infinitivo

47. pay

paid

pagar

48. hear

heard

ouvir, escutar

49. include

included

incluir

50. believe

believed

acreditar

51. allow

allowed

permitir

52. meet

met

encontrar, conhecer

53. lead

led

levar, conduzir, liderar

54. live

lived

viver, morar

55. stand

stood

levantar

56. happen

happened

acontecer, ocorrer

57. carry

carried

carregar

58. talk

talked

conversar

59. appear

appeared

aparecer

60. produce

produced

produzir

61. sit

sat

sentar

62. offer

offered

oferecer

63. consider

considered

considerar

64. expect

expected

esperar, ter expectativa

65. suggest

suggested

sugerir

66. let

let

deixar, permitir

67. read

read

ler

68. require

required

requerer

69. continue

continued

continuar

70. lose

lost

perder

71. add

added

adicionar

72. change

changed

mudar

73. fall

fallen

cair

74. remain

remained

permanecer, remanecer

75. remember

remembered

lembrar

76. buy

bought

comprar

77. speak

spoken

falar, conversar

78. stop

stopped

parar

79. send

sent

enviar, mandar

80. receive

received

receber

81. decide

decided

decidir

82. win

won

ganhar, vencer

83. understand

understood

entender, compreender

84. describe

described

descrever

85. develop

developed

desenvolver

86. agree

agreed

concordar

87. open

opened

abrir

88. reach

reached

alcanar

89. build

built

construir

90. involve

involved

envolver

91. spend

spent

gastar, passar o tempo

92. return

returned

retornar

93. draw

drawn

desenhar, retirar, empatar

94. die

died

morrer

95. hope

hoped

esperar, ter esperana

61

infinitive

62

simple past

past participle

traduo do infinitivo

96. create

created

criar

97. walk

walked

caminhar, andar

98. sell

sold

vender

99. wait

waited

esperar

100. cause

caused

causar

101. pass

passed

passar

102. lie

lied

mentir

103. accept

accepted

aceitar

104. watch

watched

assistir

105. raise

raised

elevar, aumentar

106. base

based

basear

107. apply

applied

aplicar

108. break

broken

quebrar, interromper

109. explain

explained

explicar

110. learn

learned, learnt

aprender

111. increase

increased

aumentar, incrementar

112. cover

covered

cobrir

113. grow

grown

crescer, desenvolver

114. claim

claimed

alegar, afirmar, clamar

115. report

reported

reportar, descrever

116. support

supported

apoiar, dar suporte

117. cut

cut

cortar

118. form

formed

formar

119. stay

stayed

ficar, permanecer

120. contain

contained

conter

121. reduce

reduced

reduzir

122. establish

established

estabelecer

123. join

joined

juntar, juntar-se

124. wish

wished

desejar

125. achieve

achieved

alcanar, conquistar

126. seek

sought

procurar

127. choose

chosen

escolher

128. deal

dealt

lidar, negociar

129. face

faced

encarar, enfrentar

130. fail

failed

falhar

131. serve

served

servir

132. end

ended

terminar

133. kill

killed

matar

134. occur

occurred

ocorrer

135. drive

driven

dirigir, conduzir

136. represent

represented

representar

137. rise

risen

levantar, aumentar

138. discuss

discussed

discutir, dialogar

139. love

loved

amar, adorar

140. pick

picked

pegar, escolher

141. place

placed

colocar, pr

142. argue

argued

discutir

143. prove

proved

provar

144. wear

worn

vestir

infinitive

simple past

past participle

traduo do infinitivo

145. catch

caught

pegar, apanhar

146. enjoy

enjoyed

usufruir, curtir

147. eat

eaten

comer

148. introduce

introduced

introduzir, apresentar

149. enter

entered

entrar

150. present

presented

apresentar

2. Turn the following sentences into simple past. The first item has been done for you.
Love could mend your heart.
a) Love can mend your heart.
_______________________________________________
b) Can love mend your heart?
c) There is a clear message in a bottle here. _______________________________________________
d) There are some things love can do for you. _______________________________________________
e) Does commerce rule the world?
_______________________________________________
f) Do commerce and finance rule the world? _______________________________________________
_______________________________________________
g) Commerce rules the world.
h) Is terrorism a shadow over everyones head? _______________________________________________
i) Terrorism and fear are a shadow over the USA. _______________________________________________
j) Am I the only one here?
_______________________________________________
k) They do not come on Sundays.
_______________________________________________
l) They dont like it at all.
_______________________________________________
_______________________________________________
m) She doesnt see it that way.
O TEXTO ABAIXO REFERE-SE S QUESTES 3 A 5.
(UFES adaptado) Punctuation hero branded a vandal for painting apostrophes on street signs
After enduring sloppy punctuation
on the street sign outside his home
for more than a year, Stefan Gatward
could stand it no longer. The 62-year-old former soldier _(1)_ to launch a
one-man crusade against dumbed
down Britain, and _(2)_ up a paintbrush to insert a missing apostrophe.
This _(3)_ the incorrect St Johns
Close into the correct St Johns Close. But he was immediately _(4)_ of
being a vandal by one neighbour, and
his amendments have been _(5)_ off
by others who apparently prefer the
wrong version.
The 62-year-olds defence of the apostrophe comes after Birmingham council announced it would
scrap the punctuation from council signs for the sake of simplicity. Mr Gatward _(6)_ into his flat
in Tunbridge Wells, Kent, 14 months ago. He said today: As we are off St Johns Road and opposite
St Johns Church, both with the apostrophe, St Johns Close should have one too. But when Mr
Gatward decided to correct the crime against the language by painting in the missing punctuation
mark, he was jeered by a neighbour. He _(7)_ me I was wrong. He _(8)_ me a vandal and a graffiti
artist, Mr Gatward said. He tried to tell me that the Post Office would not deliver to the street if
you put in an apostrophe. Mr Gatward, who _(9)_ for four years in the Gordon Highlanders in the
1960s, is not just a campaigner for the apostrophe.
He will not join the five items or less queue at the supermarket, in protest that the sign should
read five items or fewer. He also gets annoyed when people-neglect the Royal in Royal Tunbridge Wells, and was vexed when he _(10)_ a major chain store advertising sales with signs saying
until stocks last rather than while stocks last.
I fought for the preservation of our heritage and our language but some people seem happy to let
that go. Im not, he said.
(FERNANDEZ, Colin. Punctuation hero branded a vandal for painting apostrophes
on street signs. Disponvel em: <http://www.dailymail.co.uk)

63

3. Fill in the blanks in the text below by using


words from the box. Write your answers in
the blanks found after the text.
accused called
decided moved
picked saw
scratched served
told turned
1) ____________________________________
2) ____________________________________
3) ____________________________________
4) ____________________________________
5) ____________________________________
6) ____________________________________
7) ____________________________________
8) ____________________________________
9) ____________________________________
10) ____________________________________
Responda s perguntas seguintes em portugus
4. O ex-soldado Stefan Gatward iniciou uma
cruzada pessoal contra algo que julgou errado em sua vizinhana. No que consiste essa
cruzada? Qual foi a reao iniciativa de
Stefan Gatward de alguns dos moradores da
rua dele?
5. A iniciativa mencionada no item anterior a
nica na cruzada pessoal de Stefan Gatward?
Justifique sua resposta com as informaes
do texto.

Gabarito
E.O. Teste I
1. E

2. B

3. D

4. B

5. A

6. C

7. D

8. D

9. B

10. E

E.O. Teste II
1. A

2. E

3. A

4. C

5. D

6. E

7. B

8. C

9. A

10. E

4. C

5. B

E.O. Teste III


1. A

2. D

3. D

6. 04 + 16 = 20 7. 01 + 04 + 08 = 13
8. 02 + 16 = 18 9. 01 + 02 = 03
10. 08 + 16 = 24
64

E.O. Dissertativo

1.
Veja a tabela no fim deste gabarito.
2.
a) Love could mend your heart.
b) Could love mend your heart?
c) There was a clear message in a bottle here.
d) There were some things love could do
for you.
e) Did commerce rule the world?
f) Did commerce and finance rule the world?
g) Commerce ruled the world.
h) Was terrorism a shadow over everyones
head?
i) Terrorism and fear were a shadow over
the USA.
j) Was I the only one here?
k) They did not come on Sundays.
l) They didnt like it at all.
m) She didnt see it that way.
3.
1) Decided
2) Picked
3) Turned
4) Accused
5) Scratched
6) Moved
7) Told
8) Called
9) Served
10) Saw
4.
De acordo com as informaes do texto, Stefan Gatward, inconformado com a falta de
apstrofes nas placas da rua onde mora, tomou a iniciativa de ele mesmo fazer as correes necessrias. Alguns dos vizinhos de
Stefan mostraram-se desfavorveis iniciatica dele.
5.
Pintar as placas de sua rua no foi a nica iniciativa de Stefan Gatward. Ele tambm
se recusa a entrar em filas de supermercado
cuja sinalizao esteja errada, alm de protestar contra erros gramaticais em anncios
de lojas.

infinitive

simple past

past participle

traduo do infinitivo

1. be

was, were

been

ser, estar

2. have

had

had

ter

3. do

did

done

fazer

4. say

said

said

dizer, falar

5. get

got

got, gotten

conseguir, obter

6. make

made

made

fazer

7. go

went

gone

ir

8. see

saw

seen

ver

9. know

knew

known

saber, conhecer

10. take

took

taken

tomar, pegar

11. think

thought

thought

pensar, achar, supor

12. come

came

come

vir

13. give

gave

given

dar, distribuir

14. look

looked

looked

olhar

15. use

used

used

usar, utilizar

16. find

found

found

encontrar, descobrir

17. want

wanted

wanted

querer, procurar

18. tell

told

told

dizer

19. put

put

put

pr, colocar

20. mean

meant

meant

significar, querer dizer

21. become

became

become

tornar-se

22. leave

left

left

partir, sair, deixar

23. work

worked

worked

trabalhar

24. need

needed

needed

precisar, necessitar

25. feel

felt

felt

sentir

26. seem

seemed

seemed

parecer

27. ask

asked

asked

perguntar, pedir

28. show

showed

showed, shown

mostrar, demonstrar

29. try

tried

tried

tentar

30. call

called

called

chamar, ligar

31. keep

kept

kept

manter, continuar

32. provide

provided

provided

fornecer, prover

33. hold

held

held

segurar, conter

34. turn

turned

turned

virar, mudar

35. follow

followed

followed

seguir

36. begin

began

begun

iniciar, comear

37. bring

brouhgt

brought

trazer

38. like

liked

liked

gostar

39. forget

forgot

forgotten

esquecer

40. help

helped

helped

ajudar

41. start

started

started

comear

42. run

ran

run

correr, administrar

43. write

wrote

written

escrever

44. set

set

set

preparar, arrumar, configurar

45. move

moved

moved

mover, mudar

46. play

played

played

jogar, brincar, tocar, executar

47. pay

paid

paid

pagar

48. hear

heard

heard

ouvir, escutar

49. include

included

included

incluir

65

infinitive

66

simple past

past participle

traduo do infinitivo

50. believe

believed

believed

acreditar

51. allow

allowed

allowed

permitir

52. meet

met

met

encontrar, conhecer

53. lead

led

led

levar, conduzir, liderar

54. live

lived

lived

viver, morar

55. stand

stood

stood

levantar

56. happen

happened

happened

acontecer, ocorrer

57. carry

carried

carried

carregar

58. talk

talked

talked

conversar

59. appear

appeared

appeared

aparecer

60. produce

produced

produced

produzir

61. sit

sat

sat

sentar

62. offer

offered

offered

oferecer

63. consider

considered

considered

considerar

64. expect

expected

expected

esperar, ter expectativa

65. suggest

suggested

suggested

sugerir

66. let

let

let

deixar, permitir

67. read

read

read

ler

68. require

required

required

requerer

69. continue

continued

continued

continuar

70. lose

lost

lost

perder

71. add

added

added

adicionar

72. change

changed

changed

mudar

73. fall

fell

fallen

cair

74. remain

remained

remained

permanecer, remanecer

75. remember

remembered

remembered

lembrar

76. buy

bought

bought

comprar

77. speak

spoke

spoken

falar, conversar

78. stop

stopped

stopped

parar

79. send

sent

sent

enviar, mandar

80. receive

received

received

receber

81. decide

decided

decided

decidir

82. win

won

won

ganhar, vencer

83. understand

understood

understood

entender, compreender

84. describe

described

described

descrever

85. develop

developed

developed

desenvolver

86. agree

agreed

agreed

concordar

87. open

opened

opened

abrir

88. reach

reached

reached

alcanar

89. build

built

built

construir

90. involve

involved

involved

envolver

91. spend

spent

spent

gastar, passar o tempo

92. return

returned

returned

retornar

93. draw

drew

drawn

desenhar, retirar, empatar

94. die

died

died

morrer

95. hope

hoped

hoped

esperar, ter esperana

96. create

created

created

criar

97. walk

walked

walked

caminhar, andar

98. sell

sold

sold

vender

infinitive

simple past

past participle

traduo do infinitivo

99. wait

waited

waited

esperar

100. cause

caused

caused

causar

101. pass

passed

passed

passar

102. lie

lay

lied

mentir

103. accept

accepted

accepted

aceitar

104. watch

watched

watched

assistir

105. raise

raised

raised

elevar, aumentar

106. base

based

based

basear

107. apply

applied

applied

aplicar

108. break

broke

broken

quebrar, interromper

109. explain

explained

explained

explicar

110. learn

learned, learnt

learned, learnt

aprender

111. increase

increased

increased

aumentar, incrementar

112. cover

covered

covered

cobrir

113. grow

grew

grown

crescer, desenvolver

114. claim

claimed

claimed

alegar, afirmar, clamar

115. report

reported

reported

reportar, descrever

116. support

supported

supported

apoiar, dar suporte

117. cut

cut

cut

cortar

118. form

formed

formed

formar

119. stay

stayed

stayed

ficar, permanecer

120. contain

contained

contained

conter

121. reduce

reduced

reduced

reduzir

122. establish

established

established

estabelecer

123. join

joined

joined

juntar, juntar-se

124. wish

wished

wished

desejar

125. achieve

achieved

achieved

alcanar, conquistar

126. seek

sought

sought

procurar

127. choose

chose

chosen

escolher

128. deal

dealt

dealt

lidar, negociar

129. face

faced

faced

encarar, enfrentar

130. fail

failed

failed

falhar

131. serve

served

served

servir

132. end

ended

ended

terminar

133. kill

killed

killed

matar

134. occur

occurred

occurred

ocorrer

135. drive

drove

driven

dirigir, conduzir

136. represent

represented

represented

representar

137. rise

rose

risen

levantar, aumentar

138. discuss

discussed

discussed

discutir, dialogar

139. love

loved

loved

amar, adorar

140. pick

picked

picked

pegar, escolher

141. place

placed

placed

colocar, pr

142. argue

argued

argued

discutir

143. prove

proved

proved

provar

144. wear

wore

worn

vestir

145. catch

caught

caught

pegar, apanhar

146. enjoy

enjoyed

enjoyed

usufruir, curtir

147. eat

ate

eaten

comer

67

infinitive

68

simple past

past participle

traduo do infinitivo

148. introduce

introduced

introduced

introduzir, apresentar

149. enter

entered

entered

entrar

150. present

presented

presented

apresentar

Class 5

Looking forward!

wavebreakmedia/Shutterstock

H duas estruturas verbais bsicas para se expressar aes futuras em lngua inglesa: will e going to.

Will
a forma mais genrica. Serve para expressar um futuro mais incerto, sem data definida ou que acontecer em um
futuro mais distante.
Afirmativa Sujeito + WILL + verb (base form)
Exemplos:
1. I will pass the exam (Eu passarei/vou passar no exame)
2. She will someday accept the truth. (Ela um dia aceitar/vai aceitar a verdade)
3. Those two countries will reach an agreement. (Aqueles dois pases chegaro/vo chegar a um acordo.
Negativa Sujeito + WILL NOT (WONT) + verb (base form)
Exemplos:
1. I will not (wont) pass the exam. (Eu no vou passar/passarei no exame)
2. She will never accept the truth. (Ela nunca aceitar a verdade)
3. Those two countries will not (wont) reach an agreement. (Aqueles dois pases no chegaro a um acordo)
Interrogativa WILL + Sujeito + verb (base form)
Exemplos:
1. Will you pass the exam? (Voc passar/vai passar no exame?)
2. Will she ever accept the truth? (Ela algum dia aceitar/vai aceitar a verdade?)
3. Will those two countries reach an agreement? (Aqueles dois pases alcanaro um acordo?)

Observaes:
NUNCA utilize to antes ou depois do verbo auxiliar WILL.
I will to help you. (incorrect)
I will help you. (incorrect)
NUNCA acrescente s no verbo auxiliar WILL para as terceiras pessoas do singular she, he, it.
She wills help you. (incorrect)
She will help you. (correct)

Going To
A estrutura verbal GOING TO utilizada para expressar um futuro mais prximo, com data ou preparao j definidos.
Afirmativa Sujeito + to be (present) + going to + verb
Exemplos:
1. I am going to pass the exam. (Eu vou passar no exame.)
2. George is going to take your explanation into account. (George vai levar em conta sua explicao.)
3. Brazil is going to host the 2016 Olympic Games. (Brazil vai sediar os jogos olmpicos de 2016)
4. Those two countries are certainly going to reach an agreement. (Aqueles dois pases certamente vo chegar
a um acordo.)
71

Negativa Sujeito + to be (present) + NOT + going to + verb


Exemplos:
1. You are not going to do it! (Voc no vai fazer isso!)
2. People are going to refuse any changes. (As pessoas vo recusar quaisquer mudanas)
3. This is going to be produced in Malaysia. (Isto vai ser produzido na Malsia)
Interrogativa to be (present) + sujeito + going to + verb
Exemplos:
1. Are we going to accept his apologies? (Ns vamos aceitar suas desculpas?)
2. Am I going to follow you? (Eu vou seguir vocs?)
3. Is Laura going to study abroad? (Laura vai estudar no exterior?)

Observaes:
Assim como a lngua portuguesa desaconselha o uso de estruturas como Eu vou ir ao dentista amanh
(dois usos do verbo ir), a lngua inglesa faz o mesmo com estruturas como I am going to go to the dentist tomorrow. aconselhvel que se redija ou se fale I am going to the dentist tomorrow

Apndice #1
Embora muito menos comuns, existem duas outras estruturas verbais que se prestam a expressar aes futuras.
Estude os exemplos abaixo:
Present Continuous
Exemplos:
1. I cant go to the movies with you because I am seeing my ophthalmologist tomorrow. (Eu no posso ir
ao cinema com voc porque eu estou vendo meu oftalmologista amanh)
2. They are answering the letter very soon. (Eles estaro respondendo carta muito em breve)
Simple Present
Exemplos:
1. Governments are likey to face obstacles in a near future. (Governos esto propensos a ter obstculos em
um futuro prximo)
2. His schoolmates play vooleyball every Wednesday since 1998. (Os colegas de escola dele jogam voleibol
toda quarta-feira desde 1998)
Apndice #2
O verbo auxiliar SHALL tem a mesma funo de WILL, embora seja muito mais formal e raro. SHALL muito mais
comumente encontrado associado aos sujeitos I e WE. Estude os exemplos a seguir:
1. I shall/will talk to your superiors about that. (Eu conversarei com seus superiores sobre isso)
2. We shall/will not pass beyond this point. (Ns no iremos alm desse ponto)

72

E.O. Teste I
1. (FEI) I dont think. Coloque na forma afirmativa e no tempo futuro.
a) I dont think
b) I am thinking
c) I think
d) I wont think
e) Ill think
2. (Unesp) Assinale a alternativa que preenche
corretamente a lacuna da frase a seguir:
He will _____ almost everything you ask him.
a) do
b) to do
c) doing
d) does
e) did
3. (Unesp) Ill __________ soccer this afternoon.
a) playing
b) played
c) to play
d) play
e) plays
4. (Mackenzie) A prize __________ to whoever
solves this equation.
a) has given
b) should give
c) is giving
d) will be given
e) must have given
5. (Unesp) Assinale a alternativa correta:
I expect that she _____ arrive at about
midnight.
a) is
b) will
c) going to
d) must
e) goes
6. (UEL) A forma verbal Youll em Youll find
some monster savings on books at amazon.
co.uk indica:
a) hbito.
b) futuro.
c) necessidade.
d) permisso.
e) vontade.
7. (UEL) Assinale a letra correspondente a alternativa que preenche corretamente a lacuna do texto apresentado.
Imagine travelling directly to London and Paris with no connections to run for, no buses
to board, no taxis to hail. In fact, the only
thing you have to change is the tongue you

speak upon arrival. Thats exactly what you


_____ experience aboard the high speed Eurostar passenger train.
a) are
b) had
c) does
d) go
e) will
8. Assinale a alternativa que NO expressa uma
ao no futuro.
a) Brazil will certainly become a global player.
b) The new laws will be discussed by the parliament.
c) World leaders are meeting next week.
d) World leaders meet every November in Davos, Switzerland.
e) Representatives are now discussing the new
legislation.
9. Choose the alternative that best completes
the gaps:
Smartphones __________ the simpler cell
phones a long time ago, and nobody knows
what __________ them in future.
a) replaced will replace
b) will replace will replace
c) replaces replaced
d) replaced are going to
e) is going to will replace
10. Assinale a alternativa que corretamente preenche a sentena abaixo:
Education __________ adopt new technologies in order to remain useful to society.
a) are going to
b) will to
c) is going to
d) arent going to
e) wont be

E.O. Teste II
TEXTO PARA AS PRXIMAS 6 QUESTES
STRONGER, FASTER, SMARTER
Exercise does more than build muscles and help
prevent heart disease. New science shows that
it also boosts brainpower - and may offer
hope in the battle against Alzheimers.
BY MARY CARMICHAEL

The stereotype of the dumb jock has never sounded right to Charles Hillman. A
jock himself, he plays hockey four times a
week, but when he isnt body-checking his
opponents on the ice, hes giving his mind a
comparable workout in his neuroscience and
kinesiology lab at the University of Illinois.
73

Nearly every semester in his classroom, he


says, students on the womens cross-country
team set the curve on his exams. So recently
he started wondering if there was a vital and
overlooked link between brawn and brains
- if long hours at the gym could somehow
build up not just muscles, but minds. With
colleagues, he rounded up 259 Illinois third
and fifth graders, measured their body mass
index and put them through classic PE routines: the sit-and-reach, a brisk run and timed push-ups and sit-ups. Then he checked
THEIR physical abilities against their math
and reading scores on a statewide standardized test. Sure enough, on the whole, the
kids with the fittest bodies were THE ONES
with the fittest brains, EVEN WHEN factors
such as socioeconomic status were taken
into account. Sports, Hillman concluded,
might indeed be boosting the students intellect - and also, as long as he didnt take the
puck to the head, his own (...)
(Newsweek)

1. (UFSCar) A pesquisa apresentada no texto


foi desenvolvida por:
a) Hillman e 259 cidados de Illinois.
b) colegas de Hillman e 259 cidados de Illinois.
c) Hillman e outros colegas.
d) participantes do time de hockey de Illinois e
mulheres do time de corrida.
e) colegas de Hillman e participantes do time
de hockey de Illinois.
2. (UFSCar) A ideia expressa pelo marcador
textual even when, destacado no texto, a
de que:
a) embora no se considerem os fatores socioeconmicos, os resultados so confiveis.
b) mesmo quando considerados os fatores socioeconmicos, os resultados so confiveis.
c) se considerssemos os fatores socioeconmicos, os resultados seriam confiveis.
d) porque consideramos os fatores socioeconmicos, os resultados so confiveis.
e) somente considerando os fatores socioeconmicos, os resultados so confiveis.
3. (UFSCar) Os resultados da pesquisa indicam que:
a) no h uma relao significativa entre a prtica
de atividade fsica e o desempenho do crebro.
b) fazer parte do time de hockey fundamental para alunos de 3as e 5as sries.
c) as mulheres tm melhor desempenho nas
provas de neurocincia do que os homens.
d) estudantes com ndice de massa corporal alto
no devem ser submetidos a exerccios fsicos.
e) h uma forte relao entre a prtica de atividade fsica e o desempenho do crebro.
74

4. (UFSCar) Charles Hillman :


a) professor e pesquisador da Universidade de
Illinois.
b) mdico neurocirurgio na Universidade de
Illinois.
c) estudante da Universidade de Illinois.
d) atleta da Universidade de Illinois.
e) tcnico do laboratrio de neurocincia da
Universidade de Illinois.
5. (UFSCar) Charles Hillman tinha um questionamento sobre uma possvel relao entre:
a) as mulheres atletas e o time de corrida.
b) a prtica de exerccios fsicos e a otimizao
do desempenho do crebro.
c) o uso do crebro e o desenvolvimento do
cncer de mama.
d) a prtica de exerccios fsicos e o desenvolvimento de msculos nas mulheres.
e) as habilidades fsicas e o aumento do crebro.
6. (UFSCar) Os referentes their e the ones
destacados no texto se referem respectivamente a:
a) mulheres e crianas.
b) habilidades fsicas e crianas.
c) testes padronizados e mulheres.
d) alunos de 3as e 5as sries e crianas.
e) corpos e crebros.
7. (UFSC) Select the propositions which indicate Future Tense.
01) Paul and Mary are going to Africa in July.
02) The children wont be hungry anymore.
04) David is going to study in a good school.
08) Gregory is working in Africa now.
16) David will be eleven years old next year.
8. (PUC-PR) Im sorry, but I __________ able to
meet you for lunch tomorrow.
a) havent been
b) cant be
c) dont be
d) wont be
e) wasnt
TEXTO PARA AS PRXIMAS 2 QUESTES.
Leia o texto: A wave of anger is sweeping the
cities of the world
The protests have many different origins. In
Brazil people rose up against bus fares, in
Turkey against a building project. Indonesians have rejected higher fuel prices. In the
euro zone they march against austerity, and
the Arab spring has become a perma-protest
against pretty much everything.
Yet just as in 1848, 1968 and 1989, when
people also found a collective voice, the

demonstrators have much in common. In


one country after another, protesters have
risen up with bewildering speed. They
tend to be ordinary, middle-class people,
not lobbies with lists of demands. Their
mix of revelry and rage condemns the corruption, inefficiency and arrogance of the
folk in charge.
Nobody can know how 2013 will change the
world if at all. In 1989 the Soviet empire
teetered and fell. But Marxs belief that 1848
was the first wave of a proletarian revolution
was confounded by decades of flourishing
capitalism and 1968 did more to change sex
than politics. Even now, though, the inchoate significance of 2013 is discernible. And
for politicians who want to peddle the same
old stuff, news is not good.
The Economist, June 29, 2013. Adaptado.

9. (Fuvest) Ao comparar os protestos de 2013


com movimentos polticos passados, afirmase, no texto, que:
a) nem sempre esses movimentos expressam
anseios coletivos.
b) as crenas de Marx se confirmaram, mesmo
aps 1848.
c) as revoltas de 1968 causaram grandes mudanas polticas.
d) no se sabe se os protestos de 2013 mudaro
o mundo.
e) mudanas de costumes foram as principais
consequncias de movimentos passados.
10. (Fuvest) Segundo o texto, os protestos de
2013, em diversos lugares do mundo,
a) vm perdendo fora por falhas de organizao.
b) questionam a atuao dos lobbies nas reivindicaes das diversas classes sociais.
c) condenam a corrupo e outros comportamentos inadequados da classe poltica.
d) resultam de motivaes econmicas precisas.
e) tm poucos aspectos em comum.

E.O. Teste III


TEXTO PARA AS PRXIMAS 4 QUESTES
NEUROSCIENCE NO MORE...PLEASE!
A chocolate orgy for sciences sake.
How does your brain know when youve
had too much of a good thing? Dana Small,
a cognitive neuroscientist at Northwestern
Universitys School of Medicine in Chicago,
decided to look into it by feeding people
chocolate. She stood in the hallway of her
department and recruited nine self-proclaimed
cocoa-bean lovers, hooked them up to MRI
machines, gave them chocolate, and mix
of revelry and rage condemns the corthen
measured blood flow in their brains while
they ate. Between bites, they were asked
two questions: Do you want another piece? and On a scale of minus-10 to plus10,
how much did you like the last piece? Of
course, in the time the volunteers euphoria
over tasting the confection turned to satiation, and, as it did, Small saw changes in
their brain activity. When the subjects ate
chocolate and enjoyed it, a region associated
with mood the medial part of the orbital
frontal cortex was active. But as they began to feel satiated, blood stopped flowing
to that area and increased in another region
probably responsible for the decision to stop
eating the adjacent lateral orbital frontal cortex. Such images will help scientists
better understand how addiction causes
the brains normal response to a good thing
to go awry. But the research also yielded a
curious, unexpected result: Women ate twice as much chocolate as men before getting
their fill. And while men stopped eating because they began to find the chocolate unpleasant, women stopped only when their
bellies were stuffed. During the course of
the study, Small also discovered another interesting fact: People of Asian descent tended to be satisfied after eating only one or
two pieces of chocolate. Non-Asians, on the
other hand, practically gorged themselves.
SINHA, G. NO MORE...PLEASE! Popular Science.
New York, v. 259, n. 7, Jan. 2002, p. 42.
Vocabulrio:
to go awry: sair do curso normal; gorged
themselves: empanturraram-se

1. (Ufg) Da leitura do texto, percebe-se que a


pergunta inicial:
a) expressa o interesse de um grupo de obesos.
b) respondida pelos participantes da pesquisa.
c) cria expectativa para a apresentao do assunto.
d) evidencia a complexidade do fenmeno.
e) revela uma preocupao dos participantes.
75

2. (UFG) Em relao tipologia textual, o texto :


a) argumentativo, porque trata dos malefcios
e benefcios de se comer chocolate.
b) cientfico, porque apresenta uma tese e procura fundament-la com evidncias.
c) apelativo, por tentar convencer as pessoas a
comerem mais chocolate.
d) didtico, porque requer o conhecimento do
significado exato dos termos com que opera.
e) informativo, porque relata os resultados de
uma experincia na rea de neurocincia.
3. (UFG) Da leitura do texto, conclui-se que:
a) os participantes da pesquisa, de ambos os
sexos, eram apreciadores de chocolate.
b) o experimento foi realizado por meio de exames laboratoriais.
c) a sensao de saciedade surge dez minutos
aps a ingesto incessante de chocolate.
d) a carncia afetiva provoca, nas mulheres, um
impulso para um maior consumo de chocolate.
e) descendentes asiticos satisfazem-se quando comem grande quantidade de chocolate.
4. (UFG) A sentena (...) they began to feel
satiated, blood stopped flowing to that area
and increased in another region est corretamente escrita no simple future em:
a) they will began to feel satiated, blood will
stopped flowing to that area and will increased in another region
b) they are going to feel satiated, blood is
going to stop flowing to that area and will
have increased in another region
c) they will to begin to feel satiated, blood will
stop flowing to that area and will be increased in another region
d) they will begin to feel satiated, blood will
stop flowing to that area and will increase
in another region
e) they have began to feel satiated, blood has
stopped flowing to that area and has increased in another region
TEXTO PARA AS PRXIMAS 4 QUESTES
Book reading experience and the ultimate
technology

76

The computer is just a feeble attempt of humankind to replicate the workings of a unique invention, while science is an attempt
to understand the craftsmanship of the
greatest masterpiece Nature.
As we all know, scientists can only replicate
carbon-based life forms. They cannot literally make from out of thin air any genetic
material. They rely on raw materials from
resources provided by nature. It is just like
trying to simulate the functions of the human brain by inventing the computer. The
brain is the fastest and most powerful multimedia computer that exists in the present.
It is organic, self upgrading and has high articulation. The computer, on the other hand,
cannot help itself without the intervention
of humans. A computer hooked to the internet provides access to all the knowledge
of humanity. However, there is a downside
about this convenience. It deprives the human brain the necessary thinking skills and
training for data hunting and information
processing. A student today only needs to
access the internet to do research homework.
However, the learning experience is defeated since it is not the student who researches the data needed for the school homework but the web browser. 1So where is
the selective learning process?
If carrying books is so tiresome, mobile devices like smart phones will do the trick.
All a student has to do is search in a web
browser for the Intext free eBook reader, for
example, and then hit enter. While there are
lots of commercially available eBooks being
offered out there, education must not be so
costly. Thats why there are millions of electronic books downloadable all over the world
for free.
Reading books, either printed or the electronic version, provides the learner the essential mental training for data hunting, logical
reasoning for information processing and
the uncanny ability to differentiate truth from
lies, right from wrong, facts from fallacy,
real from a hoax and the appropriate from
the inappropriate.
In the process of reading a book, a learner
elevates the capacity of the human brain
for stimulation of the senses by indexing
information for instant recall and further
developing comprehension through mental
processing of data.
Nature is the Ultimate Technology and
the human brain is one of its most powerful inventions. We are zillions of years more
advanced as compared to our technology.
However, digital media such as e-texts can

surely serve as invaluable and easy to access


reading material. Thus, lets take advantage
of this technology to the fullest by engaging
in reading extensively in order to enhance
our natural skills and talents.
JOEFEL CAGAMPANG http://www.goarticles.com

5. (UERJ) The general theme of the text is the


impact of the digital era on the process of
knowledge acquisition.
In order to express his point of view, the
author bases his argument on the following
factor:
a) improvement of brain activities.
b) advancement of genetic research.
c) superiority of human intelligence.
d) development of concentration skills.
6. (UERJ) Paragraph 5 lists positive results
from reading books.
The exemplification offered by the author
emphasizes the capacity of:
a) synthesis.
b) distinction.
c) identification.
d) memorization.
7. (UERJ) The use of computers for learning
purposes is discussed in the text.
According to the author, the use of computers without any guidance constitutes an
obstacle to:
a) data collecting.
b) logical thinking.
c) knowledge sharing.
d) quantitative browsing.
8. (UERJ) The strategy of asking questions is
often used for rhetorical purposes.
So where is the selective learning process?
(ref. 1)
The question above is intended to:
a) express a doubt.
b) start a discussion.
c) confirm a viewpoint.
d) anticipate an objection.
TEXTO PARA AS PRXIMAS 2 QUESTES
Audio books: all pros, no cons

What is an audio book? An audio book is


a recording of the contents of a book read
aloud, created and distributed on CDs, cassette tapes or other digital formats. They
have become quite popular since first being
introduced roughly 20 years ago with the
CD format in the lead for sales.
One of the pros for audio books is the fact
that they support multi-tasking, and can
also be stored into small devices for easy listening, like an iPod or MP3 Player. For example, you may listen to one and retrieve the
information you would from a regular book
while exercising, cleaning around the house,
or just while in the car driving. You would
not be able to do any of those activities while reading a regular book. But while listening to an audio book this becomes possible.
Another pro supporting audio books is the
effect it has on childrens motivation and
learning. Teaching them to read has never
been any easier or enjoyable in their favor.
By using different types of toys from favorite cartoon characters to interact with audio
books, children are able to learn a usually
difficult subject quite easily while also having
a good time.
My final pro supporting an audio book is
the fact that people who are unable to read
regular books, due to certain problems they
cannot help such as blindness, are certainly
able to listen to audio books to gather the
contents they may need from a regular book.
This is a major pro that is a big help to a lot
of people around the world, with no con to
argue against it. So why not go and check
out some audio books today and find out for
yourself how useful and how fun they are?
There are loads of free digital audio books as
well. 1Therefore, you can test the water for
free as well.
http://bookstove.com

9. (UERJ) Therefore, you can test the water for


free as well. (ref. 1)
In the fragment, people are encouraged to
read audio books to test their efficiency. The
major motivation for this course of action
consists of:
a) courtesy offer.
b) speedy ordering.
c) refundable option.
d) costless download.

77

1
0. (UERJ) In the text, the title and subtitle
clearly reflect the authors position regarding audio books.

TEXTO PARA AS PRXIMAS 2 QUESTES


Future Hot Careers: Space Tourism to Genetic Counseling

The idea conveyed by the title and subtitle


is best expressed in the following statement:
a) Their use is considered as a beneficial practice.
b) Their efficiency is confirmed in sports activities.
c) Their quality is determined by the narrators
voice.
d) Their advantage is related to mental concentration.

E.O. Dissertativo
1. Escreva as frases a seguir usando as duas
formas de future: will e going to. O primeiro
item j foi feito para voc.
a) You (to earn) a lot of money.
(will): You will earn.
(going to): You are going to earn.
b) You (to travel) around the world.
(will): _______________________________
(going to): ___________________________
c) You (to meet) lots of interesting people.
(will): _______________________________
(going to): ___________________________
d) Everybody (to adore) you.
(will): _______________________________
(going to): ___________________________
e) You (not/to have) any problems.
(will): _______________________________
(going to): ___________________________
f) Many people (to serve) you.
(will): _______________________________
(going to): ___________________________
g) They (to anticipate) your wishes.
(will): _______________________________
(going to): ___________________________
h) There (not/to be) anything left to wish for.
(will): _______________________________
(going to): ___________________________
i) Everything (to be) perfect.
(will): _______________________________
(going to): ___________________________
j) But all these things (to happen/only) if you
marry me.
(will): _______________________________
(going to): ___________________________
78

In our information-rich society there is an


ever increasing demand for workers in the
fields of computers, health care, science and
space technology much of it driven by the
demands of the retiring baby boomers. If
you like to plan ahead, here is sampling of
some of the jobs that will be hot in the next
several years and beyond.
Genetic Counseling
Doctors will be able to test for dozens of
genetic markers and predict when a person
will likely experience a genetically based
condition. With more tests and treatments
available, genetic counselors will be needed
to help individuals and families make decisions about genetic technologies as it applies to science and personal beliefs. Today,
about 2,000 counselors are recognized by
the American Board of Genetic Counseling.
Space Tourism
While this one may sound far-fetched, the
entire industry of space tourism is poised
to take off. There are already 200 reservations for space flights. Space Adventures plans on hiring about 10 space tour
guides to start, said spokeswoman Stacey
Tearne. The worlds first space hotel is also
set to open, which could be the beginning
of a whole new sector of jobs which will
require the merging of space smarts with
great hospitality.
Roboticists
In a way, robots have already taken over
the world. The components, processors and
sensors for robots are getting cheaper every
quarter, said Paul Saffo, a technology forecaster. Hundreds of new applications for
robots are already being developed. Robots
already work in research laboratories, factories, hospitals, daycares and housekeeping,
and the trend is only expected to grow as the
field progresses.
Disponvel em: <http://www.dailygalaxy.com/
my_weblog/2008/08/future-top-10-h.html>.
Acesso em: 31 de maio de 2012. [Adaptado].

Responda s perguntas a seguir em portugus.


2. (UFRM) Em relao aos avanos da gentica
apresentados no texto:
a) o que os mdicos podero fazer no futuro?
b) qual ser o papel dos consultores genticos?
3. (UFRM) De acordo com o texto, a indstria
do turismo espacial est pronta para decolar.
Mencione os trs fatos que permitem ao autor fazer essa afirmao.
TEXTO PARA A PRXIMA QUESTO

of processed foods, and todays children will


be the first generation ever to live shorter
lives than their parents. Its time for change.
Its time we switch to fresh food and home
cooking. Its time for a Food Revolution.
(Adaptado de http://www.jamieoliver.com/ foundation/
jamies-food-revolution/. Acessado em 28/09/2011.)

a) Quais so, segundo o texto, as consequncias da atual dieta das crianas norte-americanas?
b) No que consiste a revoluo alimentar proposta por James Oliver?

Gabarito
E.O. Teste I

Due to budget cuts, our school is not distributing report card. They are available online
for download. Luckily, Im the only one in
my family that is computer literate, not that
Ill make changes or anything...
Disponvel em: <http://www.cartoonstock.com/directory/c/
computer_literacy.asp&h>. Acesso em: 2 set. 2011.

4. (UEG) Como o estudante do quadrinho acima pode conseguir informaes sobre suas
notas na escola?
5. (Unicamp)
WHY WE NEED A FOOD REVOLUTION

By James Oliver

Were losing the war against obesity in the


US. Its sad, but true. Our kids are growing
up overweight and malnourished from a diet

1. E

2. A

3. D

4. D

5. B

6. B

7. E

8. E

9. A

10. C

4. A

5. B

E.O. Teste II
1. C

2. B

3. E

6. D

7. 1 + 2 + 4 + 16 = 23

8. D

9. D

10. C

E.O. Teste III


1. C

2. E

3. A

4. D

5. C

6. B

7. B

8. C

9. D

10. A

E.O. Dissertativo

1.
a)
b) you will travel / you are going to travel
c) you will meet / you are going to meet
d) Everybody will adore you. / Everybody is
going to adore you.
e) You will not have any / You are not going
to have
f) Many people will serve you. / Many people are going to serve you.
g) They will anticipate your / They are going
to anticipate
h) There will not be anything - There is
not going to be
i) Everything will be perfect. / Everything
is going to be
j) But all these things will only happen / But
all these things are only going to happen
79

2.
a) Mdicos s conseguiro testar marcadores genticos para prever a possibilidade
das pessoas sofrerem de uma doena gentica no futuro.
b) Consultores genticos sero responsveis
por ajudar indivduos e famlias a chegarem a concluses a respeito de tecnologias genticas.
3.
De acordo com o texto, j foram feitas 200
reservas para voos espaciais; j existem planos para a contratao de guias tursticos
espaciais, e o primeiro hotel espacial poder
ser inaugurado em breve.
4.
O estudante pode simplesmente baix-los da
internet. (He can download them.)
5.
a) Obesidade, m nutrio e expectativa de
vida reduzida, em comparao aos pais.
b) A volta do consumo de alimentos frescos
e de comida caseira.

80

Kzenon/Shutterstock

Class 6
Do you understand?
Text interpretation

De forma geral, a maneira que o vestibulando aborda as provas no o mais adequado. D-se ateno especial
leitura do texto, o que sacrifica no a nota final, mas sim o tempo gasto para se alcanar uma boa pontuao.
Ao enfatizar a leitura (e o que erroneamente chamado pelo vestibulando de traduo) do texto, o vestibulando
pode at alcanar a nota desejada, mas a consequncia costuma ser um desperdcio de tempo muito grande, o que
costuma ser desastroso.
Para evitar essa catstrofe sugere-se aqui uma abordagem mais adequada e objetiva. Os passos que so
sugeridos a seguir fazem parte do ensino de linguagem que normalmente chamada de leitura instrumental.
1st step: Verifique o(s) ttulo(s), fonte e imagens associadas ao texto. Ao levantar estas informaes voc
estar se preparando para a leitura do mesmo.
2nd step: Leia atentamente os enunciados e alternativas (quando houver) relacionadas ao texto, dessa
forma voc estar ao mesmo tempo coletando informaes sobre ele e se preparando para procurar apenas o que
lhe for perguntado.
3rd step: Faa uma leitura superficial do texto (skimming), no se preocupando com as palavras desconhecidas e com informaes muito especficas. Essa primeira leitura destina-se apenas a tomar contato com o sentido
mais geral do texto.
4th step: a partir da informao requisitada pelo enunciado das perguntas, possvel identificar a qual
trecho do texto aquela questo se refere.
5th step: ao ler qualquer trecho de um texto, tente trabalhar apenas com as palavras que voc conhece ou
com aquelas que voc seja capaz de deduzir o significado. verdade que ao ignorar as palavras desconhecidas,
possvel que sua compreenso de algumas informaes do texto fique comprometida, mas com certeza uma considervel quantidade delas seja apreendida.
Ao seguir as instrues propostas antes desse texto, poderamos deduzir que se trata de um texto informative/jornalstico, por ter sido retirado do website da reviste Time. A ilustrao sugere que se trata de uma pessoa
desvairada, compulsiva pela leitura de mensagens, ideia reforada pelo ttulo (12 dicas para viciados em email) .
A concepo inicial seria reforada pelas informaes apresentadas pelos enunciados e alternativas (mesmo
que saibamos que h mais alternativas erradas do que corretas). Na sequncia, o vestibulando poderia se concentrar na questo nmero 2, por ser necessria apenas a informao contida na prpria questo.
A questo 1 baseia-se no primeiro trecho do texto. Sabemos disso porque o escritor abre o texto em primeira pessoa (I used to think I). J a questo 3 respondida pelo trecho no qual a amiga do escritor apresentada
(I have a friend)
Em uma resoluo mais tradicional, o vestibulando teria fatalmente lido o texto algumas vezes, o que
significaria perda de tempo e poderia gerar confuso de raciocnio porque h muitas informaes que no so essenciais para a resoluo das questes propostas. Por outro lado, se voc se concentrar em responder as perguntas
propostas, os resultados costumam aparecer mais rpida e claramente.

83

Vamos praticar com o texto a seguir.


(FUVEST Adaptado)

12 Steps for E-Mail Addicts


I used to think I could quit checking my e-mail any time I wanted to, but I stopped kidding myself years ago. My e-mail program is up and running 24 hours a day, and once I submit to its siren call, whole hours can go missing. I have a friend who recently found herself stuck on a cruise ship near Panama that didnt offer e-mail, so she chartered a helicopter to take her to the nearest Internet cafe.
There was nothing in her queue but junk mail and other spam, but she thought the trip was worth it.
I know how she felt. You never know when youre going to get that note from Uncle Eric about your inheritance. Or
that White House dinner invitation with a time-sensitive R.S.V.P.
www.time.com

1. A passagem nos conta que o escritor


a) acredita que j passou da hora para ele parar de pensar que pode, a qualquer momento, interromper seu
hbito de ler e-mails.
b) est totalmente ciente de que um verificador compulsivo de e-mail.
c) costumava pensar que apenas garotos desperdiavam horas inteiras verificando suas mensagens.
d) no achava que seriam necessrios anos para se livrar do seu vcio de ler e-mails.
e) acredita que se conseguir ficar 24 horas sem ler qualquer e-mail, ele vai definitavamente se livrar desse
hbito.
2. Escolha a correta traduo para ...whole hours can go missing
a) no sinto falta das horas perdidas.
b) vale a pena desperdiar vrias horas.
c) sou capaz de perder horas inteiras.
d) posso perder totalmente a noo das horas.
e) no me importo em ficar at altas horas.
3. De acordo com a informao do texto, o que a amiga do escritor descobriu quando ela finalmente conseguiu
checar seus e-mails?
a) Mensagens desimportantes
b) Uma mensagem do prprio escritor.
c) Um convite para o jantar
d) Nenhuma mensagem
e) Uma mensagem de seu tio.

84

E.O. Teste I
TEXTO PARA AS PRXIMAS 3 QUESTES
Beams of Money

DAISUKE TANAKAs daily commute has


gotten a lot simpler in recent weeks now
that he can pay for all his train tickets
with his mobile phone. To travel Tokyos
trains, subways and even some taxis, all
he needs to do is to waive his mobile
near the now standard card reader. He
has also downloaded software for several prepaid and credit cards, turning his
mobile phone into a replacement for his
wallet. I havent touched my wallet at
all today, he said one recent evening.
The mobile wallet was announced by NTT
Docomo with great fanfare back in 2004,
when the technology first became available in Japan, but so far its been a dud
fewer than one in five owners of a mobile wallet handset has ever used it to pay
for anything. Mobile wallets are accepted
at too few places, say experts. Thats now
changing quickly, however. New services
are rolling out so fast, 2007 is emerging
as the year of the mobile wallet in Japan.
The biggest reason is the rapid expansion of e-money services accessible by
mobile: 7-Eleven, the countrys dominant convenience chain, has just launched its own brand of e-money, and leading chain AEON will follow suit later
this year. The mobile Suica service provides one of the biggest incentives for
users to go mobile. The Suica fare card,
introduced in 2001, now has more than
20 million users.
Newsweek. July 9, 2007

1. (Fatec) De acordo com o texto, Daisuke Tanaka:


a) trabalha para a companhia de metr.
b) tem desconto nos bilhetes de trem.
c) no utiliza frequentemente trem, metr e txi.
d) trocou uma carteira por um celular.
e) pode pagar seus bilhetes de trem atravs de
seu celular.
2. (Fatec) Segundo o texto, a mobile wallet:
a) est fazendo muito sucesso desde 2004.
b) tambm um telefone celular.
c) aceita em diversos estabelecimentos no Japo.
d) foi lanada pela rede 7-Eleven.
e) tem sido utilizada por um grande nmero de
japoneses.

3. (Fatec) De acordo com o texto, 7-Eleven, Suica e AEON:


a) So empresas japonesas que fornecem suprimentos para o metr de Tkio.
b) So empresas que j oferecem ou oferecero
os seus cartes de dinheiro eletrnico.
c) So empresas para quem Daisuke Tanaka trabalha.
d) So nomes de lojas de convenincia no Japo.
e) So empresas que colaboraro com a NTT Docomo do Japo.
TEXTO PARA AS PRXIMAS 3 QUESTES
(Unesp) I started to run because I felt desperately unfit. But the biggest pay-off for
me was and still is the deep relaxation
that I achieve by taking exercise. It tires me
out but I find that it does calm me down.
When I started running seven years ago, I
could manage only 400 meters before I had
to stop. Breathless and aching, I walked the
next quarter of a mile, alternating these two
activities for a couple of kilometers. When I
started to jog I never dreamt of running in
a marathon, but a few years later I realized
that if I trained for it, the London Marathon,
one of the biggest British sporting events,
would be within my reach. My story shows
that an unfit 39-year old, as I was when I
started running, who had taken no serious
exercise for twenty years, can do the marathon and that this is a sport in which women can beat men. But is it crazy to do it?
Does it make sense to run in the expectation
of becoming healthier?
My advice is: if you are under forty, healthy
and feel well, you can begin as I did by jogging gently until you are out of breath, then
walking, and alternating the two for about
three kilometers. Build up the jogging in
stages until you can do the whole distance
comfortably.
Headway Intermediate Students Book.
Oxford University Press. Adaptado.

4. (Unesp) Assinale a alternativa correta:


a) A autora do texto se considera uma pessoa
relaxada e indiferente em relao prtica
de esportes.
b) O texto apresenta o depoimento de uma
corredora que iniciou sua prtica nesse
esporte porque se sentia fora de forma.
c) A autora do texto se exercitou seriamente
durante vinte anos para poder participar da
maratona de Londres.
d) O texto apresenta argumentos contrrios
prtica da corrida por pessoas na faixa
etria acima dos quarenta anos.
e) De acordo com o texto, a prtica de exerccios
por vinte anos causa, especialmente nas
mulheres, dores crnicas e falta de ar.
85

5. (Unesp) De acordo com o texto:


a) a autora comeou a praticar corrida aos
sete anos de idade e nessa poca conseguia
percorrer 400 metros.
b) pessoas na faixa etria de quarenta anos
esto geralmente fora de forma para a
prtica da corrida.
c) a autora comeou a praticar corrida porque
sempre sonhou em correr na maratona de
Londres.
d) pessoas abaixo de quarenta anos conseguem
iniciar a prtica da corrida alternando etapas
de corrida e de caminhada.
e) corredores na faixa etria de quarenta anos
geralmente sentem falta de ar nos primeiros
estgios da corrida.
6. (Unesp) No texto, as expresses pay-off (1
pargrafo), a couple of (2 pargrafo), my
reach (2 pargrafo) e becoming healthier
(2 pargrafo) significam, respectivamente:
a) sem pagamento, alguns, minha riqueza, e
tornar-se saudvel.
b) pagamento, uma dupla de, meu objetivo e
tornar-se saudvel.
c) corte no pagamento, uma dupla de, minha
riqueza e ficar doente.
d) desafio, um casal de, meu objetivo e tornarse saudvel.
e) recompensa, alguns, meu alcance e tornarse saudvel.
TEXTO PARA AS PRXIMAS 4 QUESTES
UPDATA: BAD BLOOD
It doesnt look like something youd
want dripping into your veins, wrote Wil
McCarthy in the August 2002 issue of Wired.
At the time, he had no way of knowing
just how right he was about Hemopure, the
artificial blood that seemed so promising. It
was universally compatible and had a threeyear shelf life (unrefrigerated). But a recent
meta - analysis of trials on several
substitutes - including Hemopure - contains
some 1gory results. Turns out, the fake
bloods scavenge nitric oxide, causing
vasoconstriction; patients who get them
are 2.7 times more likely to have a heart
attack and 30 percent more likely to die. A
Journal of the American Medical Association
editorial has called for a halt to trials.
KATHARINE GAMMON Wired. August, 2008

7. (Fatec) Os pacientes que utilizam Hemopure podem apresentar:


a) Ausncia de problemas cardacos.
b) Uma recuperao demorada.
c) Sintomas de vaso contrio.
d) Baixos ndices da substncia xido ntrico.
e) Uma recuperao mais rpida.
86

8. (Fatec) Hemopure um tipo de sangue artificial que:


a) Tem vida til de trs anos, quando devidamente refrigerado.
b) Pode ser utilizado em qualquer pessoa, sem
restries de idade.
c) Tem vida til indefinida, quando devidamente refrigerado.
d) Est sendo utilizado por hospitais, com algumas reservas.
e) Tem vida til de trs anos e atende a todos
os tipos sanguneos.
9. (Fatec) A palavra gory (ref. 1) pode ser
substituda por:
a) Scary.
b) Efficient.
c) Terrific.
d) Promising.
e) Interesting.
10. A palavra likely destacada no texto expressa:
a) certeza.
b) probabilidade.
c) dvida.
d) necessidade.
e) urgncia.

E.O. Teste II
Baseado nas informaes do texto abaixo,
marque (C) ou errado (E) para as afirmaes
de 1 a 5.
TEXTO PARA AS PRXIMAS QUESTES
THE WORST OF BOTH WORLDS?
In the global-warming debate, theres a big
gap between public rhetoric (which verges on
hysteria) and public behavior (which indicates indifference). People say theyre worried
but dont act that way. Greenhouse emissions
continue to rise despite many earnest pledges to control them. Just last week, the United Nations reported that of the 41 countries
it monitors (not including most developing
nations), 34 had increased greenhouse emissions from 2000 to 2004. These include most
countries committed to reducing emissions
under the Kyoto Protocol.
Why is this? Here are three reasons. First:
With todays technologies, we dont know
how to cut greenhouse gases in politically
and economically acceptable ways. Second:
In rich democracies, policies that might curb
greenhouse gases require politicians and the
public to act in exceptionally enlightened
(read: unrealistic) ways. Third: Even if
rich countries cut emissions, it wont make
much difference unless poor countries do likewise - and so far, theyve refused because

that might jeopardize their economic growth and poverty-reduction efforts.


Unless we develop cost-effective technologies that break the link between carbondioxide emissions and energy use, we cant do
much. Anyone serious about global warming
must focus on technology - and not just assume it. Otherwise, our practical choices are
all bad: costly mandates and controls that
harm the economy; or costly mandates and
controls that barely affect greenhouse gases.
Or, possibly, both.
Adapted from: The Worst of Both Worlds?.
Newsweek November 13, 2006, page 45.

(UFPE) In accordance to the text, in the


glwarming debate, it is noticed that:
1. ( ) there have been no agreements to control greenhouse emissions at all.
2. ( ) public rhetoric and public behavior diverge.
3. ( ) most countries that are committed to reducing emissions dont do so.
4. ( ) countries that are monitored by the United
Nations follow the Kyoto Protocol strictly.
5. ( ) terrible consequences are foreseen if global warming isnt controlled.
Leia o texto a seguir, avalie as afirmaes
abaixo, some o numeral daquelas que considerar corretas e escreva o resultado no espao abaixo provido.
TEXTO PARA AS PRXIMAS QUESTES

out of the hospital and back in school,


she is determined to keep fighting for
equality. God has given me this new
life, she said in February, her first public
statement since the shooting. I want to
serve the people. I want every girl, every
child, to be educated. Worldwide, there
are 66 million girls out of school, according to UNESCO many more than boys,
who dont have to face the same discrimination and obstacles that girls do in some
countries.
Malala was critically injured in the attack, but she suffered no permanent brain
injuries. She underwent several successful surgeries in Pakistan and the United
Kingdom, where she now lives after her
father was given a job with the Pakistani Consulate. In March, she went back to
school for the first time since the attack, attending an all-girls high school in
Birmingham, England. And while she
recovers from her injuries, she is continuing to raise awareness and money for
education. Last month, she announced a
$45,000 grant to a fund that was set up
in her name and the first to benefit will
be girls from the Swat Valley.
By Kyle Almond, CNN June 17, 2013 Updated 1346
GMT (2146 HKT) Adapted from: <http://edition.cnn.
com/2013/04/30/ world/malala-girls-education/
index.html> Accessed on: July 18th, 2013.

(UFSC) Select the CORRECT proposition(s).


Text gives information about:

6. ( ) the number of girls who are out of school


all over the world.

7. ( ) Malalas current age and health state.

8. ( ) the amount of money Malalas father receives from the government.


Six months ago, Malala Yousafzai was lying
in a hospital bed, recovering from a Taliban
attack in which she was shot in the head and
neck. The shooting was intended to si-lence
the Pakistani teenager who had defied the
Talibans ban against girls in school. But it
had the opposite effect: Instead of silencing
the 15-year old, the attack only made her
voice more powerful.
Malalas story has raised global awareness
of girls education. And now that shes

9. ( ) the reason why Pakistani girls cannot attend school.

10. ( ) Malalas plans for the future.

87

E.O. Teste III


TEXTO PARA AS PRXIMAS 10 QUESTES
The New York Times on the web
The Rush to Enhancement: Medicine Isnt
Just for the Sick Anymore
By Sherwin B. Nuland

Until the mid-1960s, medical research


was primarily driven by the desire to solve the problems of sick people. Although Aristotle was what might be termed
today a pure laboratory investigator,
with no thought of the clinical usefulness of his findings, the vast majority of
those physicians later influenced by his
contributions to biology were trying to
solve the mysteries of human anatomy
and physiology for the distinct purpose
of combating sickness. The discovery of
blood circulation in the 17th century, the
elucidation of the anatomical effects of
disease in the 18th, the introduction of
antisepsis and anesthesia in the 19th,
the development of antibiotics and cardiac and transplant surgery in the 20thall of these were the direct results of physicians and others having recognized a
specific group of challenges that stood in
the way of making sick people better. Armed with knowledge of the disease processes, they entered their laboratories to
address specific clinical issues. Their goal
was improving the lot of actual patients,
often their own.
The rise of molecular biology since the
late 1950s has had the gradual and quite unforeseen effect of turning the eyes
of medical scientists increasingly toward
the basic mechanisms of life, rather than
disease and death. Of course, this has
always been the orientation of all non-medical biologists, studying growth, reproduction, nutrition or any of the other
characteristics shared by all living things.
But now the boundaries have become
blurred, between research that will alter
the approach to disease and research that
will alter the approach to life itself. While until very recently the bedside usually
determined what was done in the medical
research laboratory, the findings coming
out of the laboratory nowadays are just
as likely to tell the clinician what he can
do at the bedside. The tail often wags the
dog. In fact, the tail is becoming the dog.
Adaptado. Encontra-se na ntegra em <http://nytimes.
com/library/review/051098medicine-review.html>

88

1. (Unifesp) A palavra although na frase


do primeiro pargrafo, Although Aristotle
was what might be termed today a pure
laboratory investigator..., indica uma ideia
de:
a) alternncia.
b) exemplificao.
c) oposio.
d) condio.
e) enumerao
2. (Unifesp) Na frase do segundo pargrafo, ...
turning the eyes of medical scientists increasingly toward the basic mechanisms of life,
rather than disease and death., a expresso
rather than pode ser substituda, sem
mudar o sentido, por:
a) even if.
b) in order to.
c) moreover.
d) furthermore
e) instead of.
3. (Unifesp) One of the recent medical developments of last century was:
a) transplant surgery.
b) cardiac blood circulation control.
c) antisepsis.
d) anesthesia.
e) human molecule anatomy.
4. (Unifesp) Atualmente, os dois tipos de pesquisa mdica:
a) determinam as doenas que sero pesquisadas mais intensamente.
b) realizam testes em animais de laboratrio,
principalmente em ces.
c) interferem no diagnstico clnico de pacientes em leitos de hospitais.
d) fazem testes laboratoriais com seres humanos acamados.
e) no apresentam uma diferenciao clara.
5. (Unifesp) O texto, como um todo:
a) apresenta uma evoluo cronologicamente
inversa das descobertas mdicas.
b) exemplifica historicamente a ideia de que o
interesse da medicina oscila pendularmente
a cada sculo.
c) defende o avano da tecnologia, como no
caso dos transplantes.
d) compara a viso das pesquisas mdicas do
passado com a do presente.
e) afirma que as descobertas atuais so
inconsistentes, na medida em que no se
concentram na cura de doenas.

6. (Unifesp) The focus of medical research until


the mid 60s was to:
a) investigate purely the causes of illnesses
in laboratories.
b) discover the mysteries of the human body.
c) solve the problems of sick people.
d) discover physiological enigmas.
e) elucidate the anatomical effects of blood
circulation.
7. (Unifesp) From the late 1950s on, medical
scientists started increasingly to focus on:
a) disease and death.
b) basic mechanisms of life.
c) all living things.
d) molecules and chemistry.
e) reproduction and genetics.
8. (Unifesp) H pouco tempo, as pesquisas mdicas:
a) eram determinadas pela prtica clnica.
b) tendiam a definir os procedimentos clnicos
e cirrgicos adotados.
c) alteravam a viso mdica da vida em si.
d) tratavam da doena como um processo natural do mecanismo da vida.
e) percebiam a vida, a doena e a morte como
campo de estudo da biologia.
9. (Unifesp) Na frase do segundo pargrafo, Of
course, this has always been the orientation
of all non-medical biologists..., a palavra
this refere-se a:
a) research in molecular biology.
b) gradual and unforeseen effect.
c) medical scientists.
d) study of basic mechanisms of life.
e) study of disease and death.
10. (Unifesp) Na ltima frase do primeiro pargrafo, Their goal was improving the lot of
actual patients, often their own., a palavra
their, que ocorre duas vezes, refere-se:
a) aos mdicos e outros.
b) s pessoas doentes.
c) aos fsicos e bilogos.
d) aos prprios pacientes.
e) ao conhecimento dos processos da doena.

E.O. Dissertativo
Getting Enough Sleep?
By Emily Sohn

One of the greatest things about growing older is that you get to stay up later. And its
not just your parents who make that decision. Your body gives you permission to enjoy the darker hours too. Recent research has
shown that brain changes during teen years
make it easier for kids to stay up late. But
just because you can stay up late, it doesnt
mean you should, scientists say. Young people who dont get enough sleep are often late
for school, or they miss it completely, says a
recent study by the National Sleep Foundation (NSF). Sleepy kids also tend to be bad
tempered and unhappy. And their grades
suffer. Our sleep-wake schedules may seem
to be ruled by the need to get to school or
work on time, but theyre really under the
control of our bodys internal clock. Every
mammal has a master clock in its brain
that tells its body what time it is and when
it needs to sleep. Scientists recently discovered cells in the brain that collect information about light directly from the eyes.
When light comes in, our internal clock
thinks that its daytime. Then, as darkness
arrives, the body secretes a chemical called
melatonin, which tells the clockthat its
nighttime. As we approach our teen years,
melatonin secretion comes at a later time.
Thats why children who usually go to bed at
8:30 or 9 p.m. all of a sudden start having
trouble falling asleep.
Adaptado de Science News for Kids.

1. (Unicamp) O que acontece, segundo o texto,


com jovens que costumam dormir menos do
que o necessrio?
2. (Unicamp) Para o que serve o nosso relgio
interno?
3. (Unicamp) Nosso corpo produz uma substncia qumica denominada melatonina.
Qual a funo dessa substncia e como ela
nos afeta quando nos aproximamos da adolescncia?
89

TEXTO PARA AS PRXIMAS 2 QUESTES


Leia o seguinte texto e responda, em portugus, ao que se pede.
Yahoo! wants to reinvent the postage stamp
to cut spam. Researchers are testing a scheme where users pay a cent to charity for
each email they send so clearing their inbox and conscience simultaneously. Yahoo!
Researchs CentMail resurrects an old idea:
that levying a charge on every e-mail sent
would instantly make spamming uneconomic. But because the cent paid for an accredited stamp to appear on each email goes
to charity, CentMails inventors think it will
be more successful than previous approaches
to make e-mail cost. They think the cost to
users is offset by the good feeling of giving
to charity.
<http://www.newscientist.com/article/dn17577>.
Acessado em 14/08/2009. Adaptado.

4. (Fuvest) O texto apresenta uma proposta


feita pela empresa Yahoo! para diminuir a
quantidade de mensagens eletrnicas indesejadas ou spams. Qual a proposta?
5. (Fuvest) Por que os inventores do CentMail
acreditam que sua proposta ser mais bem
sucedida que as anteriores?

Gabarito
E.O. Teste I
1. E

2. B

3. B

4. B

5. D

6. E

7. C

8. E

9. A

10. B

E.O. Teste II
1. F

2. V

3. V

4. F

5. V

6. V

7. V

8. F

9. V

10. V

E.O. Teste III


1. C

2. E

3. A

4. E

5. D

6. C

7. B

8. A

9. D

10. A

E.O. Dissertativo

1.
Segundo o texto, jovens que dormem menos
do que o necessrio frequentemente se atrasam para as aulas ou as perdem por completo. Alm disso, os jovens tm a tendncia a
apresentarem mal-humor, serem infelizes e
a no obterem boas notas escolares.
90

2.
Segundo o texto, nosso relgio interno informa ao corpo que horas so e tambm quando
precisamos dormir.
3.
A melatonina uma substncia que regula
nosso relgio interno, avisando-o quando o
perodo noturno chega. Quando nos aproximamos da adolescncia, a secreo da melatonina acontece mais tarde do que o habitual, o que faz com que os adolescentes
tenham problemas para adormecer.
4.
A proposta do Yahoo! sugere que se cobre de
seus usurios um centavo para cada mensagem enviada.
5.
Segundo o texto, o sucesso dessa iniciativa
ocorreria porque todo o dinheiro arrecadado
seria direcionado para instituies de caridade.

91

Class 7

Should we continue?

wavebreakmedia/Shutterstock

O termo Modal Verbs ou simplesmente Modals serve para designar um tipo especial de verbos auxiliares que tm
a capacidade de alterar o sentido de outro verbo que os segue. Eles no seguem as regras e padres gramaticais
seguindo, portanto um modo prprio (da o termo modal, em ingls). Os verbos modais so: can, could, may,
might, should, must, ought to, will, shall e would.
Observe as regras gramaticais dos verbos modais:
1. No acrescente s terceira pessoa do singular
He can understand Spanish very well. (correct)
He cans understand Spanish very well. (incorrect)
2. No so usados verbos auxiliares para frases negativas ou interrogativas.
I should not be afraid of anything. (correct)
I dont should be afraid of anything (incorrect)
Should I carry on studying? (correct)
Do you should be afraid? (incorrect)
3. NUNCA acrescente a partcula to nem antes, nem aps um verbo modal (com exceo de have to e ought to)
He can understand Russian very well (correct)
He can to understand Russian very well (incorrect)
De modo a facilitar sua compreenso, vamos separar os verbos modais em categorias

Can, could, may/might


Expressam capacidade, habilidade, possibilidade, probabilidade, para pedir e dar permisso e pedir auxlio. Todas as
leituras possveis incluem, no portugus, o verbo poder, sendo que o verbo CAN est sempre no presente. COULD,
alm de ser a forma de passado de CAN, tambm pode ser entendido como futuro do pretrito do verbo CAN
(poderia, poderamos). MAY e MIGHT tambm so normalmente compreendidos como poderia, poderamos etc).

Estude os exemplos:
CAN X COULD
1. In order to pass, you can study on the weekends. (Para passar, voc pode estudar aos finais de semana)
Mais provvel.
2. In order to pass, you could study on the weekends. (Para passar, voc poderia estudar aos finais de semana.)
Menos provvel, hipottico.
CAN X MAY/MIGHT
1. Can you help me, please? (mais informal) X May you help me, please? (Mais formal, respeitoso)
2. It can rain today. (muito provvel) X It may rain today (menos provvel, talvez chova)
COULD
1. We could have pizza tonight. (Ns poderamos comer pizza hoje noite) Possibilidade futura, situao
hipottica.
2. When I was a kid I could play the guitar, but Ive completely forgotten how to play it. (Quando eu era garoto
eu podia/tinha a habilidade de tocar violo, mas eu esqueci completamente). Passado
3. If you were here, you could talk to us. (Se voc estivesse aqui, voc poderia conversar conosco) Hipottico
(subjuntivo).
93

Should, must, ought to, have to


Nesta categoria, encontramos verbos que normalmente expressam sugestes, conselhos, ordens, proibies e obrigaes. Eles so normalmente entendidos em portugus como os verbos dever (ria) ou ter que. Estude os exemplos
abaixo.
SHOULD X MUST
1. I think you should go out a little. (Eu acho que voc deveria sair um pouco) Uma simples sugesto.
2. You must go now. (Voc deve/ tem que ir agora) Mais imperativo, quase uma ordem.
3. You should watch that movie (Voc deve/deveria assistir aquele filme) Uma simples sugesto; o filme bom.
4. You must watch that movie (Voc deve/deveria assistir aquele filme) quase obrigatrio que se assista ao
filme; ele timo.
SHOULD x OUGHT TO
1. You should study more (Vocs deveriam estudar mais) Mais informal
2. You ought to study more (Vocs deveriam estudar mais) Mais formal, mais enftico.

Observaes 1:
Quando utilizado em sua forma negativa, MUST expressa proibio.
Exemplos:
1. You must not (mustnt) cross the yellow line. (Voc no pode/ no deve/ est proibido de cruzar a linha amarela)
2. She mustnt smoke here. We are in a hospital. (Ela no deve fumar aqui. Ns estamos em um hospital).

Observaes 2:
A locuo verbal have to (+ verb) obedece s mesmas regras gramaticais aplicveis ao verbo to have, mas
altera a funo do verbo que o segue,deixando de expressar posse e passando a expressar obrigao.
Exemplos:
1. You have to protect the citizens. Youre a policeman. (Voc tem que proteger os cidados. Voc um policial.)
2. Everybody has to wear a helmet from that point on. Thats what the regulations say. (Todos tm que usar um
capacete a partir deste ponto. o que diz o regulamento.)

WILL, SHALL e WOULD


Todos os verbos desse grupo so verbos auxiliares, ou seja, no tm sentido sozinhos, mas somente associados ou referindo-se a outros verbos.
De forma geral, WILL e SHALL fazem com que o verbo imediatamente aps eles esteja no simple future.
SHALL, alm de muito mais formal, tambm normalmente utilizado quando com os sujeitos I e WE.
WOULD um verbo auxiliar muito importante, pois equivale (com poucas excees) a todo o futuro do
pretrito do portugus.

Estude os exemplos a seguir:


He will talk to you later. (Ele conversar/vai conversar com voc mais tarde)
We shall talk to you later. (Ns conversaremos/vamos conversar com voc mais tarde)
They would talk to you if they had time. (Elas conversariam com voc se tivessem tempo.)
94

WILL X WOULD
muito importante perceber claramente as diferenas entre estes dois verbos.
1. I will like to meet them. (Eu vou gostar de conhec-las)
I would like to meet them. (Eu gostaria de conhec-las)
2. Money will be a problem. (Dinheiro ser um problema.)
Money would be a problem (Dinheiro seria um problema.)
SHALL
Os usos mais comuns do modal SHALL esto presentes naquelas perguntas de confirmao, os QUESTION
TAGS. Observe os exemplos a seguir:
1. Lets start, shall we? (Vamos comear?)
2. Shall we dance? (Vamos danar?)
Alm disso, possvel observar a ocorrncia de SHALL em textos jurdicos ou religiosos:
1. Education shall be free (A educao ser/dever ser gratuita)
2. You shall not murder. (No matars)

Formas negativas
As formas negativas dos modal verbs so obtidas atravs da adio da palavra NOT.
Cannot (cant); could not (couldnt); may not, might not; should not (shouldnt); must not (mustnt); ought
not to; will not (wont); shall not (shant); would not (wouldnt).

Formas interrogativas
As formas interrogativas dos modal verbs so obtidas atravs da inverso do sujeito da orao e do verbo modal.
Can I...?; could I...?; may I...?; might you...? should you...?; must I...? will you...? would you...? Shall we...?

Apndice #1
As formas de passado dos modal verbs so obtidos da seguinte forma:
modal + have + past participle
Exemplos:
1. You should have arrived earlier. (Voc deveria ter chegado mais cedo)
2. France could have colonized Brazil. (A Frana poderia ter colonizado o Brasil)
3. I wouldnt have done it this way. (Eu no teria feito isto desta maneira)
Observaes:
A lista dos verbos irregulares contendo seus respectivos past participles est anexada na aula 5 desta
apostila.

Apndice #2
Existem algumas outras estruturas que eventualmente so colocadas entre os verbos modais, principalmente por compartilharem algumas de suas caractersticas. So elas:
Neednt o mesmo de dont need to
Exemplo: You neednt feed the dog/ You dont need to feed the dog (Voc no precisa alimentar o cachorro)
95

Had better Similar a should.


Exemplo: You had better/should leave now. (Voc deveria sair agora)
Would rather (d rather) Similar a would prefer.
Exemplo: I would rather/would prefer to watch a movie tonight. (Eu preferiria assistir a um filme hoje noite)

Apndice #3
Estude alguns dos casos particulares a seguir:
O futuro de CAN dado pela locuo verbal be able to.
Exemplo: Students will be able to solve that kind of drill. (Os estudantes sero capazes de/conseguiro
resolver aquele tipo de atividade.)
O passado de may dado pelo verbo might.
Exemplo:
They may help you out. (presente Eles poderiam te ajudar)
They might have helped you (passado Eles poderiam ter te ajudado)
O verbo there to be (haver/existir) pode ser combinado com os verbos modais.
Exemplos:
There can be a chance. (Pode haver uma chance)
There should be someone waiting for them. (Deveria haver algum esperando por eles)

96

E.O. Teste I

6. (PUC-RJ)

1. (FCC-BA) Yes dear, you _____ come home late


tonight.
a) must to
b) ought
c) can to
d) may
e) have
2. Assinale a alternativa correta.
Im sorry the train was late and I _____ arrive earlier.
a) couldnt
b) ought not
c) dont
d) mustnt
e) wouldnt
3. (UFSC) Assinale a alternativa correta.
You __________ smoke here.
a) might to
b) have permission
c) can to
d) may
e) ought
4. (FCMSC-SP) You mustnt take that medicine
expressa:
a) uma ordem.
b) uma advertncia.
c) uma obrigao.
d) uma proibio.
e) um conselho.
5. (Mackenzie)

In you should start each day with a song...


in your soul, should expresses an idea of:
a) certainty.
b) obligation.
c) impossibility.
d) probability.
e) advice.
7. (Mackenzie) In which of the statements
does MUST express necessity?
a) Theres somebody in the other office. It must
be my boss!
b) You mustnt smoke here!
c) She must be a very good student. She always
gets As.
d) I must go right now!
e) Who must he be? There are lots of people
around him!
8. (Mackenzie) In the sentence, You may be
wrong, but you may be right. MAY means:
a) possibility
b) permission
c) ability
d) deduction
e) obligation

The man in the cartoon:


a) is planning his future.
b) is currently attending college.
c) has become a doctor.
d) never misses an opportunity.
e) regrets things in his past.

TEXTO PARA A PRXIMA QUESTO


Stuck for cash? Here are some moneymaking
ideas!
Need some extra cash? Why not consider
working from home? Setting up your own
business may sound complicated and expensive, but there are many ideas that dont
need a lot of money to follow through. Plus,
of course, theres the added advantage that
you can completely tailor your working day
to suit your lifestyle.
But dont forget to tell the tax man or you
could end up with an unexpected bill!!!
97

9. (UFPEL) A expresso business may sound


complicated and expensive..., traz consigo
a ideia de:
a) permisso.
b) possibilidade.
c) certeza.
d) habilidade.
e) obrigao.

1. O verbo MAY, em depression and hostility


may increase the risk for cardiovascular disease, diabetes and high blood pressure indica:
a) habilidade.
b) capacidade.
c) certeza.
d) obrigao.
e) possibilidade.

10. (Mackenzie) Indicate the alternative that


best completes the following sentence.
Look how wet the ground is. it __________
last night.
a) might be sunny
b) must have rained
c) should have been warm
d) may be snowing
e) ought to dry

2. O estudo mencionado no texto:


a) avaliou a influncia de nveis de hostilidade
em indivduos deprimidos.
b) baseou-se em diferentes tipos de exames
mdicos e respostas a questionrios.
c) foi importante para controlar os nveis da
protena C3 no sangue de soldados com presso arterial elevada.
d) foi desenvolvido para testar a relao entre
presso arterial e diabetes.
e) concentrou-se na anlise de manifestaes
depressivas em veteranos do Vietn.

E.O. Teste II

3. Os resultados do estudo mencionado no texto sugerem que:


a) os nveis da protena C3 diminuem em homens que passaram por experincias traumticas de guerra.
b) nveis altos de hostilidade so potencial
ameaa sade mental.
c) doenas cardiovasculares podem estar relacionadas aos nveis de hostilidade, raiva
e depresso.
d) baixa ingesto de protenas pode causar raiva e depresso, estimulando comportamentos hostis.
e) os veteranos do Vietn analisados estavam
menos deprimidos que o esperado.

(Fuvest-adaptado)

Researchers studying 313 healthy Vietnam


veterans have found that anger, depression and hostility may increase the risk for
cardiovascular disease, diabetes and high
blood pressure.
Over a period of ten years, the men had regular physical examinations involving a wide
variety of medicaltests. They also underwent
psychological examinations using well-established questionnaires to determine their
levels of hostility, anger and depression. The
researchers measured blood levels of a protein called C3, a marker for the inflammation that is a risk factor for cardiovascular
illnesses. After controlling for other variables, the scientists found that those in the
highest one-quarter in hostility, anger and
depression showed a steady and significant
increase in C3 levels, while those in the lowest one-quarter had no increase.
www.nytimes.com. Adaptado.

98

TEXTO PARA AS PRXIMAS 6 QUESTES


TEEN DEPRESSION
Depression is defined as an illness when
the feelings of sadness, hopelessness, and
despair persist and interfere with a child or
adolescents ability to function.
Though the term depression can describe a
normal human emotion, it also can refer to a
mental health illness. Depressive illness in children and teens is defined when the feelings of
depression persist and interfere with a child or
adolescents ability to function. Depression is
common in teens and younger children. About
5 percent of children and adolescents in the
general population suffer from depression at
any given point in time. Children under stress,
who experience loss, or who have attentional,
learning, conduct or anxiety disorders are at a
higher risk for depression. Teenager girls are
at especially high risk, as are minority youth.
Depressed youth often have problems at home.
In many cases, the parents are depressed, as

depression tends to run in families. Over the


past 50 years, depression rises, so does the
teen suicide rate. It is important to remember that the behavior of depressed children
and teenagers may differ from the behavior
of depressed adults. The characteristics vary,
with most children and teens having additional psychiatric disorders, such as behavior
disorders or substance abuse problems. Mental health professionals advise parents to be
aware of signs of depression in their children.
Some of these signs may be: frequent sadness,
tearfulness, crying; hopelessness; decreased
interest in activities or inability to enjoy previously favorite activities; persistent boredom;
low energy; social isolation; poor communication; poor concentration; extreme sensitivity
to rejection or failure, and increased irritability, anger, or hostility; among others.

c) Depresso um termo usado para indicar


uma emoo normal ou uma doena mental
que afeta 5% das crianas pequenas. Pode
ser diagnosticada na adolescncia quando
sentimentos como tristeza, desesperana e
desencanto interferem no comportamento
do indivduo.
d) Depresso um termo usado para indicar
uma emoo humana normal que, quando
persiste, gera uma doena que tambm pode
afetar crianas e adolescentes. Caracterizase pela tristeza, desesperana e desencanto
manifestadas na adolescncia.
e) Depresso um termo usado para indicar
uma emoo ou uma doena que apenas afeta
as mulheres, na adolescncia. Pode ser diagnosticada quando sentimentos como tristeza,
desesperana e desencanto persistem e interferem no comportamento do indivduo.

(Extrado de: www.focusas.com/Depression.html)

4. (Unesp) Escolha a alternativa correta, de


acordo com o texto.
a) O comportamento de um adulto e o de uma
criana ou adolescente com depresso podem ser diferenciados, pois o adulto sempre
est consciente de que sofre de depresso.
b) O comportamento de um adulto e de uma criana ou adolescente com depresso nunca so diferenciados, ainda que a criana ou adolescente
tenha distrbios psiquitricos adicionais.
c) Nos ltimos 50 anos, os ndices de depresso
entre adultos aumentaram consideravelmente e, como consequncia, o ndice de suicdios de adultos tambm aumentou.
d) Crianas que vivem sob presso, que vivenciam perdas e que sofrem de ansiedade correm menos riscos de depresso do que adolescentes nas mesmas condies.
e) Crianas que vivem sob presso, que vivenciam perdas e que sofrem de ansiedade,
entre outros fatores, apresentam maior tendncia a sofrer de depresso.
5. (Unesp) Escolha a alternativa correta, de
acordo com o texto.
a) Depresso um termo usado para indicar
uma emoo humana normal ou uma doena
mental que afeta a maior parte das crianas
e adolescentes. Manifesta-se como doena
quando sentimentos como tristeza, desesperana e desencanto persistem e interferem
no comportamento do indivduo.
b) Depresso um termo usado para indicar
uma emoo humana normal ou uma doena
que tambm pode afetar crianas e adolescentes. Pode ser diagnosticada como doena
quando sentimentos como tristeza, desesperana e desencanto persistem e interferem
no comportamento do indivduo.

6. (Unesp) De acordo com o texto, indique a


alternativa que preenche corretamente a lacuna da sentena:
When one or more signs of depression persist,
parents __________ professional help.
a) would have to look for
b) are looking for
c) have to look for
d) would have had to look for
e) looked for
7. (Unesp) De acordo com o texto, indique a
alternativa que expressa o mesmo sentido da
expresso em destaque na sentena:
Parents and caregivers have to try to help
teenagers when they FEEL DEPRESSED.
a) feel down
b) feel confused
c) feel disappointed
d) feel excited
e) feel any mental disorders
8. (Unesp) De acordo com o texto, indique a alternativa que expressa o mesmo significado de:
Adults expect teens to act moody.
a) Teens expected adults to act moody.
b) Adults are expected by teens to act moody.
c) Adults and teens are expected to act moody.
d) Teens are expected to act moody.
e) Teens always act moody, although it is never
expected by adults.
9. (Unesp) Indique a sentena que expressa
um conselho.
a) When depressed, teens always ask for adult
guidance.
b) Teens see more of what life has to offer and
then they become depressed.
c) Adolescents who never make new friends
become depressed.
99

d) Adolescents dont try to make new friends


when they feel depressed.
e) When teens become depressed, they should
try to ask an adult for help.
TEXTO PARA A PRXIMA QUESTO
A RACIAL GAP
Blacks undergo lifesaving lung-cancer surgery at a lower rate than whites. What can
be done? Doctors have long known that lung
cancers, which kills 160.000 Americans each
year, takes a heavier toll among black Americans, particularly black men, than among
whites. In part thats because 34% of black
men in the U.S. smoke cigarettes, compared
with 28% of white men. (Black women tend
to smoke less than white women). It also has
to do with differences in income and access
to medical care. But there has always been
a lingering suspicion that some of the gap
might be due to either overt or subconscious
discrimination. A study in New England
Journal of Medicine appears to bolster that
disturbing conclusion. Unlike other cancers,
lung cancer is extremely hard to detect in
its earliest, most treatable stages. Even so,
about 20% of lung-cancer patients are found
to have a tumor whose biological characteristics and small size give them a good chance of being cured if the malignant growth is
surgically removed.
(Time, October 25th, 1999)

1
0. (UFRRJ) Might may be replaced by
a) must.
b) can.
c) have to.
d) should.
e) could.

E.O. Teste III


TEXTO PARA AS PRXIMAS 8 QUESTES
ARE YOU A DIGITAL NATIVE OR A DIGITAL
IMMIGRANT?
We all know that we are living in an increasingly technologically driven world. Living
here in the heart of Silicon Valley I certainly feel it every day. In fact, I dont think I
know a single couple in my neighborhood,
other than my wife and I, who dont work
in the technology field in some capacity. Our
local companies are Facebook, Apple, Google,
Yahoo, and so many venture capital firms that
I cant keep them straight. But you dont
have to live in Silicon Valley to feel that the
world is getting more and more technology
centered, focused, and driven. We can debate the pros and cons of this reality but we
100

cant deny that the world has changed very


quickly in head spinning ways. Two recent
comments led me to finally enter the
21st century by getting a smart phone this
week, kicking and screaming.
First, I mentioned to one of my undergraduate
classes at Santa Clara University that I didnt
have a smart phone, but rather I had a dumb
phone. My phone can make and receive phone calls and thats about it. No email, internet, and so forth. So one of my students
looked at me in an odd and curious way,
like she was talking to someone from another planet, and stated in a matter of fact
manner, Professor Plante, even 2nd graders
have smart phones. Ouch!
Second, I was talking with a producer at
the PBS NewsHour who wanted me to do a
live interview within a few hours of his call
regarding some late breaking news about
clergy sexual abuse, which is my specialty.
I was out of the office and driving my car
when he called and in a matter of fact manner he said that he wanted to send me some
important information to my smart phone
to best prepare me for the upcoming nterview. When I told him that I couldnt receive
anything since I had a dumb phone and not
a smart phone, there was a long silence. He
then said hed have to just read it to me over
the phone as a Plan B. He wasnt happy ...
neither was I. In case you havent noticed,
the 21st century is really upon us and to live
in it one really does need to be connected in
my view. Although I often consider myself
a 19th or 20th century guy trapped in the
21st century we really do need to adapt. For
most of us we are just living in a new world
that really demands comfort with and access
to technology. This notion of digital native
vs. digital immigrant makes a great deal
of sense to me. Young people in our society are digital natives. They seem to be very
comfortable with everything from iPhones
to TV remotes. Digital immigrants, like me,
just never feel that comfortable with these
technologies. Sure we may learn to adapt
by using email, mobile phones, smart ones
or dumb ones, Facebook, and so forth but
it just doesnt and perhaps will never be
very natural for us. It is like learning a second language ... you can communicate but
with some struggle. This has perhaps always
been true. I remember when I was in graduate school in the 1980s trying to convince
my grandparents that buying a telephone
answering machine as well as a clothes dryer
would be a good idea. They looked at me like
I was talking in another language or that I
was from another planet.

Perhaps we have a critical period in our lives


for technology just like we do for language.
When we are young we soak up language so
quickly but find it so much harder to learn a
new language when we are older. The same
seems to be true for technology.
So, this week I bought my first smart phone
and am just learning to use it. When questions arise, I turn to my very patient teenage
son for answers. And when hes not around,
I just look to the youngest person around for
help. So, what about you? Are you a digital
native or a digital immigrant and how does
it impact your life?
Adapted from Digital Native vs Digital Immigrant?
Which are you? Published on July 24, 2012 by
Thomas G. Plante, Ph.D., ABPP in Do the Right
Thing http://www.psychologytoday.com/blog/
do-the-rightthing/201207/digital-native-vs-digitalimmigrantwhich-are-you retrieved on July 28, 2012

1. (PUC-RJ adaptada) The text suggests that


nowadays the world is divided into two
groups of people.
a) those who work in the technology field and
those who are against it.
b) the ones who live in Silicon Valley and those
who live in the fields.
c) the smart phone users and the wireless phone addicts.
d) those who work for Apple and those who
work for Facebook.
e) the technological natives and the technological foreigners.
2. (PUC-RJ adaptada) The main purpose of the
text is.
a) to compare the new smart phones to old
conventional devices.
b) to argue that people should adopt simple
dumb phones for their daily activities.
c) to highlight that young people are usually
technologically driven and centered.
d) to analyze the characteristics and the advantages of smart phones.
e) to prove that old people cannot learn how to
use electronic instruments.
3. (PUC-RJ adaptada) In the sentence, He
then said hed have to just read it to me over
the phone as a Plan B., the underlined pronoun refers to:
a) the authors dumb phone.
b) the information needed for the interview.
c) the authors smart phone.
d) the upcoming interview.
e) the conversation the author had with the
TV producer.

4. (PUC-RJ adaptada) Mark the CORRECT statement concerning the meanings of the words
extracted from the text.
a) kicking and screaming in ... by getting a
smart phone this week kicking and screaming. means revolutionary
b) odd in So one of my students looked at
me in an odd and curious way, ... means
respectful.
c) late breaking news in I was talking with
a producer (...) some late breaking news
means tragic news.
d) a critical period in Perhaps we have a critical period in our life for technology. means a threatening moment.
e) soak up in When we are young we soak up
language so quickly (...) means absorb.
5. (PUC-RJ adaptada) Paraphrasing the sentence In case you havent noticed, the
21st century is really upon us and to live it
one really does need to be connected in my
view, we can say that.
a) the future is here and we must be connected
to the world.
b) the present century has come to make things
more difficult for people.
c) everybody understands that technology is
necessary to survive on Earth.
d) people should try to escape the new
centurys negative effects.
e) digital natives have not noticed that they
need to be connected.
6. (PUC-RJ adaptada) The author explains the
expression dumb phone as:
a) a phone used by those who are digital natives.
b) a phone which does not have internet access.
c) a phone that can communicate with people
from another planet.
d) a phone specially designed for second graders.
e) a phone designed for those who have hearing
problems.
7. (PUC-RJ adaptada) We cant deny in ...we
cant deny that the world has changed very
quickly... and My phone can make in My
phone can make and receive phone calls...
express the ideas of, respectively:
a) probability duty
b) condition ability
c) obligation assumption
d) possibility obligation
e) impossibility ability
101

8. (PUC-RJ adaptada) Mark the INCORRECT option concerning the statements.


a) The authors resistance to using a smart
phone is comparable to his grandparents
resistance to using a clothes dryer.
b) In the authors opinion we cant avoid dealing
with technology in the 21st century.
c) Teenagers are much more familiar to the digital world than adults are.
d) When he bought a smart phone, the author
immediately got adapted to using it.
e) The author has recently faced some problems
for not using a smart phone.
9. (Uel) World Class Pure Texas bottled water just
hit supermarket shelves in Houston. The firm also
bottles private-label water and is selling Hakeen
Olajuwon H2O at Houstons Summit Arena, home
to the NBA champion Rockets. Giant food distributor Sysco MAY take the brand national.
O verbo MAY, no texto, indica:
a) um dever.
b) uma necessidade.
c) uma certeza.
d) obrigao.
e) possibilidade.
10. (PUC-SP) ...learning about nonhuman organisms DNA sequences can lead to an understanding of their natural capabilities that
can be applied toward solving challenges in
health care,...
...DNA sequences can lead to an understanding
of their natural capabilitites..., a palavra
CAN indica a ideia de:
a) conhecimento.
b) permisso.
c) habilidade.
d) confirmao.
e) probabilidade.

E.O. Dissertativo
1. Todas as frases a seguir apresentam um erro
gramatical. Reescreva-as corretamente:
a) How do can book publishers beat Amazon?
b) Who can to speak for the hospital patience?
c) Imagine what a face cans tell us.
d) Does a face can tell us something meaningful?
e) The presidents of the USA and Russia to can
actually destroy the world.
TEXTO PARA AS QUESTES 2 E 3
CAN WE FEED THE WORLD AND SUSTAIN
THE PLANET?
A five-step global plan could double food
production by 2050 while greatly reducing
environmental damage
By Jonathan A. Foley

102

The world must solve three food problems


simultaneously: end hunger, double food
production by 2050, and do both while drastically reducing agricultures damage to the
environment. Five solutions, pursued together, can achieve these goals: stop agriculture from consuming more tropical land, boost
the productivity of farms that have the lowest
yields, raise the efficiency of water and
fertilizer use worldwide, reduce per capita
meat consumption and reduce waste in food
production and distribution.
A system for certifying foods based on how well
each one delivers nutrition and food security
and limits environmental and social costs would
help the public choose products that push agriculture in a more sustainable direction.
(www.scientificamerican.com. Adaptado.)

Responda s perguntas a seguir em portugus


2. (Unesp) Dentre as cinco solues apresentadas no segundo pargrafo do texto, quais
as que se destinam a aumentar a produo
de alimentos?
3. (Unesp) Qual a proposta apresentada no
texto para que as pessoas possam escolher
produtos mais sustentveis?
TEXTO PARA AS QUESTES 4 E 5
SO LONG, FACEBOOK!
Andrea Bennett, 18 August 2012.

Ive deleted my Facebook: I was tired of having


to check my profile all the time and I just
felt overexposed and ready to trade my computer for sunshine.
I dont need Facebook, and Facebook probably doesnt need me. Right? Right But
those of us without Facebook run the risk of
being considered abnormal or eccentric, or
not being hired by potential employers who
distrust people who dont use the site.
(Adaptado de http://www.adbusters.org/ blogs/adbustersblog/so-long-facebook. html. Acessado em 12/09/2012.)

Responda s perguntas a seguir em Portugus


4. (Unicamp) Como a autora do texto acima estava se sentindo antes de fechar sua conta
no Facebook?
5. (Unicamp) Quais so, segundo o texto, os
riscos de no ter uma conta no Facebook?

Gabarito
E.O. Teste I
1. D

2. A

3. D

4. D

5. E

6. E

7. D

8. A

9. B

10. B

E.O. Teste II
1. E

2. B

3. C

4. E

5. B

6. C

7. A

8. D

9. E

10. E

E.O. Teste III


1. E

2. C

3. B

4. E

5. A

6. B

7. E

8. D

9. E

10. E

E.O. Dissertativo

1.
a) How can book publishers beat Amazon?
b) Who can speak for the hospital patience?
c) Imagine what a face can tell us.
d) Can a face tell us something meaningful?
e) The presidents of the USA and Russia can
actually destroy the world.
2.
Duas solues apresentadas destinam-se
produo de alimentos. Uma delas cita o aumento da produtividade, a outra o aumento, de forma global, na eficincia no uso da
gua e fertilizantes.
3.
Para possibilitar o acesso das pessoas e produtos sustentveis, o texto prope a criao
de sistemas de certificao de alimentos que
demonstre como so produzidos os alimentos, aspectos nutricionais e segurana alimentar e ambiental e o custo social deles.
4.
A autora do texto sentia-se cansada de checar a todo instante seu perfil do Facebook
ela achava que estava superexposta e pronta
para trocar a luz do dia por seu computador.
5.
Os riscos de no ter uma conta no Facebook
concentram-se em duas reas. A primeira
social: essas pessoas podem ser consideradas
anormais ou excntricas. A outra profissional: no estar conectado pode dificultar
novos contratos ou causar desconfiana das
pessoas que tm contas em redes sociais.

103

Class 8

Perfection

Syda Productions/Shutterstock

Similarmente ao portugus, a lngua inglesa tambm apresenta tempos perfeitos. Todos os tempos perfeitos apresentam uma combinao do verbo to have (ter) mais um particpio passado (as formas de particpio dos verbos
irregulares esto apresentadas na tabela de verbos na unidade 5). Os tempos perfeitos da lngua inglesa so:
Present Perfect, present Perfect Continuous, Past Perfect, Past Perfect Continuous, Future Perfect e Future Perfect
Continuous.

Present Perfect
HAVE/HAS + PAST PARTICIPLE
O Present Perfect um tempo verbal que causa um certo desconforto ao estudante brasileiro. Tal sentimento decorrente das limitaes de mtodos de leitura que se baseiam em tradues (como a abordagem principal deste
material didtico). Para melhorar a compreenso do Present Perfect faz-se necessria a compreenso dos usos
associados a ele, assim como tambm faz-se necessria cuidadosa negociao dos significados por ele introduzidos
nos vrios de seus usos. So eles:
Importante: Ele utilizado para expressar uma ao que comeou no passado e cujas consequncias se
estendem (e so relevantes) at o momento da fala.
Exemplos:
ary and John have worked together since 2010. (Mary e John tm trabalhado juntos desde 2010)
1. M
2. I have lost my passport. (Eu perdi meu passaporte). Neste caso, meu passaporte continua perdido.
3. S he has left the building (Ela saiu do prdio). Ela saiu do prdio e ainda no retornou.
4. B e patient, please! I havent finished it yet. (Seja paciente, por favor! Eu ainda no terminei.) Ou seja, eu
comecei algo e ainda estou executando esta tarefa.
Ele utilizado para expressar aes que tm se repetido nos ltimos tempos.
Exemplos:
1. She has studied a lot recently. (Ela tem estudado muito recentemente)
2. I have slept very well since I started my treatment. (Eu tenho dormido muito bem desde que eu comecei meu
tratamento.)
3. We have been responsible for him for 5 years. (Ns somos/temos sido os responsveis por ele nos ltimos 5
anos.) Ns comeamos e ainda somos responsveis por ele.
Ele utilizado por aes que acabaram de acontecer ou esto quase acontecendo.
Exemplos:
1. I have just sent you my report. (Eu acabei de te mandar meu relatrio)
2. Rush! The train has arrived. (Apresse-se! O trem est chegando)
Com as palavras ever (ou never) normalmente expressam-se experincias de uma vida inteira.
Exemplos:
1. Have you ever seen snow? (Voc j viu a neve?) Ou seja, desde o nascimento at o momento da fala.
2. No, I have never seen snow. (No, eu nunca vi a neve) Ou seja, desde o nascimento at o momento da fala.
3. She has never been abroad (Ela nunca esteve no exterior)
CUIDADO!!!
No se deve usar o present perfect com aes j terminadas no passado ou com tempo especfico (tambm
no passado).
Exemplos:
1. I have arrived here last week (Incorrect)
I arrived here last week (Correct)
105

2. Joseph has lived there in 2010 (Incorect)


Joseph lived there in 2010 (Correct)

Present Perfect Continuous


HAVE/HAS + BEEN + VERB(ING)
Trata-se de uma variao do Present Perfect. Este utilizado quando se quer enfatizar a continuidade uma ao
que comeou no passado e continua no presente.
Exemplos:
1. She has been reading that book since she arrived. (Ela est lendo aquele livro desde que ela chegou) Ou
seja, ela no interrompeu a leitura em nenhum momento.
2. They have been living in the same house for 35 years. (Elas esto morando na mesma casa h 35 anos.) Ou
seja, elas nunca mudaram de casa.

Past perfect
HAD + PAST PARTICIPLE
O Past Perfect corresponde ao registro mais bsico do tempo verbal que a lngua portuguesa chama de Pretrito
mais que Perfeito. A definio e o uso so praticamente os mesmos. Eles servem para expressar uma ao que
est no passado do passado. Ou seja, se for necessrio expressar duas aes no simultneas no passado, a ao
mais antiga (que aconteceu primeiro) deve ser expressa no Past Perfect, enquanto a mais recente (que aconteceu
depois), deve ser expressa no Simple Past. Estude os exemplos a seguir.
Exemplos:
1. Dont be mad! When you arrived home, we had already eaten the whole pizza (No fique bravo! Quando
voc chegou em casa, ns j tnhamos comido a pizza inteira)
2. Portugal and Spain had already signed the Tordesilhas treat when Pedro lvares Cabral arrived in the Brazilian Coast in 1500. (Portugal e Espanha j tinham/haviam assinado o tratado de Tordesilhas quando Pedro
lvares Cabral chegou na costa brasileira em 1500)

Past Perfect Continuous


HAD + BEEN + VERB (ING)
O Past Perfect tambm pode ser usado em sua forma contnua, e assim como o Present Perfect Continuous, a nfase est na durao e continuidade da ao.
Exemplos:
1. I was short of breath on the phone because I had been running in the park. (Eu estava ofegante ao telefone
porque eu tinha corrido/ estava correndo no parque.)
2. She had been waiting for a long time when you arrived. (Ela havia esperado/ esteve esperando por um longo
tempo quando voc chegou.)
106

Future perfect continuous


Embora muito mais raros, possvel que voc encontre os dois tempos acima em textos mais elaborados, portanto,
vamos aprender um pouco sobre eles. Ambos so usados para expressar aes no futuro que se iniciaram em algum
ponto do passado.

Estude os exemplos a seguir:


In a near future, Brazil will have hosted the two greatest events in the world. (Em um futuro prximo, o Brazil
ter sediado os dois maiores eventos do mundo.) Future Perfect.
By 2030, I will have been living for 59 years. (Quando 2030 chegar, eu terei vivido por 59 anos.) Future
Perfect Continuous.

Formas interrogativas
As formas negativas dos tempos perfeitos so conseguidas atravs da inverso do verbo auxiliar to have com o
sujeito da orao.
Exemplos:
1. Has she already finished her chores? (Ela j terminou as tarefas dela?)
2. Had the countries been conquered before 1800 A.D.? (Os pases j haviam/tinham sido conquistados antes
de 1800 D.C?)
3. Will you have concluded the investigation by November? (Voc ter concludo a investigao at novembro?)

Formas Negativas
As formas negativas dos tempos perfeitos so conseguidas atravs da adio do advrbio NOT junto ao verbo
auxiliar to have, ou em alguns casos atravs da adio do advrbio NEVER.
Exemplos:
1. I have not (havent) talked to anyone since I arrived. (Eu no conversei com ningum desde que cheguei)
2. The prizes had not been given away before noon. (Os prmios no tinham sido distribudos antes do meio-dia.)
3. They have never given any interview. (Eles nunca deram nenhuma entrevista)

107

E.O. Teste I
1. (Fuvest) Qual destas sentenas est correta:
a) I dont have never taken a course in Japanese.
b) I have never taken a course in Japanese.
c) I never didnt take a course in Japanese still.
d) I ever did not take a course in Japanese.
e) I took not a course in Japanese ever.
2. (Unesp) Assinale a alternativa correta.
Were still waiting for Bill. He __________ yet.
a) hasnt come
b) havent come
c) didnt come
d) doesnt come
e) hadnt come
3. (UFRGS) Choose the best alternative to complete the sentence below correctly:
Mexico __________ many difficult crises in
history, but now it __________ its own future.
a) has faced is shaping
b) faced was shaped
c) have faced shapes
d) have been facing shaped
e) faces has been shaped
TEXTO PARA A PRXIMA QUESTO
CALORIC RESTRICTION
Since 1935 researchers 4have known that
when laboratory rats and mice are fed a
very-low-calorie diet 30 to 50 percent of
2
their normal intake they live about 30
percent longer than their well-fed confreres, as long as they get sufficient nutrition.
Free radicals seem to be responsible: the less
food consumed, the fewer free radicals are
produced possibly because on a low-calorie
regimen cells power-generating machinery
operates at high efficiency, as it does during
exercise. There havent been solid studies on
how caloric restriction affects human beings,
but researchers speculate that someday drugs may 3enhance cellular efficiency without
diets. Consuming fewer calories while maintaining a healthy level of nutrients isnt
easy... 1so dont quit eating just yet.
(Newsweek, June 30, 1997. p. 59.)

4. (UFRN) A locuo verbal have known (ref. 4)


indica uma noo de temporalidade referente a:
a) dois momentos no passado.
b) passado e futuro.
c) passado, exclusivamente.
d) passado e presente.
108

5. (ITA) Since 1985 the Shop __________ a


Company limited by guarantee with charitable status; its aim is primarily to relieve
poverty in developing countries.
(Panfleto da loja ONE WORLD SHOP, em Edimburgo, Esccia)

A alternativa que melhor preenche a lacuna


do texto :
a) is.
b) was.
c) had been.
d) have been.
e) has been.
6. (Mackenzie) Indicate the alternative that
best completes the following sentence.
When the manager arrived, the problem
_______________.
a) was been solved already.
b) should to be solved yet.
c) had already been solved.
d) has still been solved.
e) had already solved.
7. (PUC-PR) Choose the RIGHT alternative to
complete the passage:
Everything __________ ready for the party. The room __________, the furniture
__________. There __________ bottles of wine
and food on the table. A jazz record __________
and the atmosphere was just right.
a) is is clean is moved are is playing
b) was had been cleaned had been moved
were was playing
c) had been had been cleaned had been
moved were had been played
d) was had cleaned had moved was
had played
e) were was cleaned was moved were
was playing
8. (UECE) In the sentences Mr. Barbosa took
on the entire legal system, he is overseeing
the precedent-setting trial and Mr. Barbosa has at times been exasperated, the verbs
are, respectively, in the:
a) simple present, present perfect and present
continuous.
b) past perfect, simple present and present
perfect.
c) simple past, present continuous and present
perfect.
d) simple past, present perfect and present
continuous.

9. (UFSJ) Young Nina and her grandmother are having a conversation:


Grandma, how long have you and Granpa been married? asked Nina.
1
Weve been married for fifty years, Grandma replied. (...)
In the joke, the sentence Weve been married for fifty years means that Ninas grandparents:
a) lived together for fifty years.
b) were married for fifty years.
c) got married fifty years ago.
d) were married for a long time.
1
0. When Brazil __________ by Cabral in 1500, the Tordesilhas Treaty __________ by Portugal and Spain.
a) is discovered was signed
b) was discovered had already been sign
c) discovered signed
d) was discovered had already been signed
e) has discovered had signed

E.O. Teste II
1. (Enem)

A tira, definida como um segmento de histria em quadrinhos, pode transmitir uma mensagem
com efeito de humor. A presena desse efeito no dilogo entre Jon e Garfield acontece porque:
a) Jon pensa que sua ex-namorada maluca e que Garfield no sabia disso.
b) Jodell a nica namorada maluca que Jon teve, e Garfield acha isso estranho.
c) Garfield tem certeza de que a ex-namorada de Jon sensata, o maluco o amigo.
d) Garfield conhece as ex-namoradas de Jon e considera mais de uma como maluca.
e) Jon caracteriza a ex-namorada como maluca e no entende a cara de Garfield.
2. (Enem)

My report is about how important it is to save


paper, electricity, and other resources. Ill send
it to you telepathically.
GLASBERGEN, R. Todays cartoon. Disponvel em: http://
www.glasbergen.com. Acesso em: 23 jul. 2010.

Na fase escolar, prtica comum que os professores passem atividades extraclasse e marquem
uma data para que as mesmas sejam entregues
para correo. No caso da cena da charge, a professora ouve uma estudante apresentando argumentos para:
a) discutir sobre o contedo do seu trabalho j entregue.
b) elogiar o tema proposto para o relatrio solicitado.
c) sugerir temas para novas pesquisas e relatrios.
d) reclamar do curto prazo para entrega do trabalho.
e) convencer de que fez o relatrio solicitado.
109

3. (Enem) WAR
Until the philosophy which holds one race
superior
And another inferior
Is finally and permanently discredited and
abandoned,
Everywhere is war Me say war.
That until there is no longer
First class and second class citizens of
any nation,
Until the color of a mans skin
Is of no more significance than the color of
his eyes
Me say war.
[...]
And until the ignoble and unhappy regimes
that hold our brothers in Angola, in Mozambique.
South Africa, sub-human bondage have been
toppled,
Utterly destroyed
WeII, everywhere is war Me say war.
War in the east, war in the west,
War up north, war down south
War war Rumors of war.
And until that day, the African continent
wiII not know peace.
We. Africans, will fight we find it necessary And we know we shall win As we are
confident in the victory.
MARLEY. B. Disponvel em: http://www.sing365.
com. Acesso em: 30 jun. 2011 (fragmento).

Bob Marley foi um artista popular e atraiu


muitos fs com suas canes. Ciente de sua
influncia social, na msica War, o cantor se
utiliza de sua arte para alertar sobre:
a) a inrcia do continente africano diante das
injustias sociais.
b) a persistncia da guerra enquanto houver
diferenas raciais e sociais.
c) as acentuadas diferenas culturais entre os
pases africanos.
d) as discrepncias sociais entre moambicanos
e angolanos como causa de conflitos.
e) a fragilidade das diferenas raciais e sociais
como justificativas para o incio de uma guerra.
4. (Enem) Hows your mood?

110

For an interesting attempt to measure cause and


effect try Mappiness, a project run by the
London School of Economics, which offers a
phone app that prompts you to record your
mood and situation.
The Mappiness website says: Were particularly interested in how peoples happiness
is affected by their local environment air
pollution, noise, green spaces, and so on
which the data from Mappiness will be absolutely great for investigating. Will it work?
With enough people, it might. But there are
other problems. Weve been using happiness
and well-being interchangeably. Is that ok?
The difference comes out in a sentiment
like: We were happier during the war. But
was our well-being also greater then?
Disponvel em: http:www.bbc.co.uk.
Acesso n: 27 jun. 2011 (adaptado).

O projeto Mappiness, idealizado pela London School of Economics, ocupa-se do tema


relacionado:
a) ao nvel de felicidade das pessoas em tempos de guerra.
b) dificuldade de medir o nvel de felicidade
das pessoas a partir de seu humor.
c) ao nvel de felicidade das pessoas enquanto
falam ao celular com seus familiares.
d) relao entre o nvel de felicidade das pessoas e o ambiente no qual se encontram.
e) influncia das imagens grafitadas pelas
ruas no aumento do nvel de felicidade
das pessoas.
5. (Enem) Going to university seems to reduce
the risk of dying from coronary heart disease.
An American study that involved 10000 patients from around the world has found that
people who leave school before the age of 16
are five times more likely to suffer a heart
attack and die than university graduates.
Word Repat News. Magazine Speak Up. Ano
XIV, n 170. Editora Cameot, 2001.

Em relao s pesquisas, a utilizao da expresso university graduates evidencia a inteno de informar que:
a) as doenas do corao atacam dez mil pacientes.
b) as doenas do corao ocorrem na faixa dos
dezesseis anos.
c) as pesquisas sobre doenas so divulgadas
no meio acadmico.
d) jovens americanos so alertados dos riscos
de doenas do corao.
e) maior nvel de estudo reduz riscos de ataques do corao.

TEXTO PARA A PRXIMA QUESTO

TEXTO PARA A PRXIMA QUESTO

Sex Tourism and Male Prostitution: European


Women Travel to Africa In Search Of Young
African Men For Sex

Brazil struggles to curb sex tourism

Reuters has a very interesting article that


caught my attention today about a new
trend in the sex tourism industry. The piece
is about old rich European women most likely from the United Kingdom traveling to
Africa in search of pretty young black men
for sex.
Notice the double standards in the mainstream
media in relation to sex tourism. Sex tourism is not just specific to just one gender yet the Occidental media has always
framed this as a male only domain. We all
have watched the documentaries about European or North American men traveling
to Asia, Africa, or South America in search
of young women for sex. Female sex tourism has gone on for decades yet now the
mainstream media is finally focusing on the
other side of sex tourism from the rich western womans perspective.
The rich Occidental women have egos as well
they want to feel attractive and sexy. The
European women feel compelled to travel to
Africa in search of young black men because
the power dynamics are in their favour.

The UN estimates that two million young people under the age of 18 are involved in prostitution. Traditionally the trade has been associated with Asia. But in recent years, Brazil
has become an increasingly popular destination. Theres a paradise quality to Recife on
the countrys north-east coast. Not the paradise of desert islands and solitude but a more
earthly variety, with vibrant beaches and beautiful people. The latter is the attraction
for a growing number of foreign tourists who
come to Brazil looking for sex, and in many
cases they are willing to pay for it. There is
a growing demand, mostly from Germans, Italians and other Europeans. They come here
not for the culture and beaches, but for sex,
often with minors. The age of consent in Brazil is 18, but many of those at work here are
much younger. Recifes secretary for tourism,
Romeo Batista, says the long-term antidote to
the sex trade lies in better social policies so
that Brazilian girls have less need for foreign
men and money. But on the question of shortterm solutions, he was somewhat defensive:
Prostitutes exist everywhere - look at Paris
for example. Here they just happen to work in
highly visible areas, and its not just a question
of getting them off the streets. You have to detain their clients, which is why weve installed
cameras which also help reduce violence.

Disponvel em: <http://orvillelloyddouglas.


wordpress. com/2007/11/27/sex-tourism-older-whitewomentravelling-to-africa-seeking-young-blackmen>. Acesso em: 17 mar. 2010. (Adaptado).

6. (UEG) De acordo com as afirmaes do texto,


CORRETO afirmar que:
a) a mdia ocidental ignora o turismo sexual
praticado por mulheres, dando enfoque apenas para os casos masculinos.
b) a prostituio masculina j supera, em nmeros, a prostituio feminina em pases da
frica, da sia e da Amrica do Sul.
c) o turismo sexual na sia, frica e Amrica do
Sul praticado tanto por homens quanto por
mulheres de origem europeia e norteamericana.
d) alm de aumentar a autoestima, o turismo
sexual praticado por mulheres europeias na
frica estimulado pelas relaes de poder
em favor desse gnero.

Disponvel em: <http://news.bbc.co.uk/2/


hi/ americas/4061325.stm>. Acesso
em: 17 mar. 2010. (Adaptado).

7. (UEG) De acordo com o contedo do texto,


CORRETO afirmar que o turismo sexual:
a) praticado por turistas asiticos e europeus
em visita ao Brasil.
b) a principal causa de prostituio infantil
em pases da sia e no Brasil.
c) deve ser combatido com medidas que tero
efeito somente a longo prazo.
d) na opinio do Secretrio de Turismo do Recife, poderia ser combatido pela adoo de
polticas sociais.
111

8. (Fuvest) We live in a highly technological society. It _(1)_ for mankind some of the most
intricate and beautifully discriminating machines ever seen as well as some of the most
powerful. What modern medicine has been
able _(2)_ to cope with certain diseases _
(3)_ to earlier generations truly miraculous.

(Dialogue 2, 1992).

Qual a sequncia que preenche corretamente


os espaos numerados?
a) (1) had devised, (2) to do, (3) will be seem.
b) (1) has devised, (2) to do, (3) will be seem.
c) (1) has been devised, (2) to be doing, (3)
would have seemed.
d) (1) has devised, (2) to do, (3) would have
seemed.
e) (1) was devising, (2) to do, (3) will have seem.
9. (Epcar) When football _____ professional in
South Africa in 1959, 12 clubs broke from the
amateur ranks. However, in the strict days of
Apartheid, these pioneers _____ whites-only
organizations and _____ today, all but a few,
defunct. One of the survivors is Arcadia from
Tshwane/ Pretoria, an outfit that today competes in the amateur ranks and concentrates
on junior football.
Mark the alternative which completes the
gaps from the text correctly.
a) had gone have been were
b) went were are
c) have been have been would be
d) was had been will be
TEXTO PARA A PRXIMA QUESTO

You have to motivate yourself with challenges. Thats how you know youre still alive.
Once you start doing only what you _(I)_ you
can do, you _(II)_ on the road to death.
Jerry Seinfeld

1
0. (Mackenzie) The verb forms that correctly
complete the blanks I and II in the quotation are:
a) should have proved, will be
b) had proven, would be
c) have proven, are
d) may have proven, can be
e) will be proving, will have been
112

E.O. Teste III


TEXTO PARA AS PRXIMAS 6 QUESTES
DO PEOPLE ONLY USE 10 PERCENT OF
THEIR BRAINS?
By Robynne Boyd

The human brain is complex. Along with performing millions of mundane acts, it composes concertos, issues manifestos and comes
up with elegant solutions to equations. Its
the wellspring of all human feelings, behaviors, experiences as well as the repository
of memory and self-awareness. So its no
surprise that the brain remains a mystery
unto itself.
Adding to that mystery is the contention
that humans only employ 10 percent of
their brain. If only regular folk could tap
that other 90 percent, they too could become
savants who remember to the twenty thousandth decimal place or perhaps even have
telekinetic powers.
Though an alluring idea, the 10 percent
myth is so wrong it is almost laughable,
says neurologist Barry Gordon at Johns Hopkins School of Medicine in Baltimore. Although theres no definitive culprit to pin
the blame on for starting this legend, the
notion has been linked to the American
psychologist and author William James, who
argued in The Energies of Men that We are
making use of only a small part of our possible mental and physical resources. Its also
been associated with Albert Einstein, who
supposedly used it to explain his cosmic towering intellect.
The myths durability, Gordon says, stems
from peoples conceptions about their own
brains: they see their own shortcomings as
evidence of the existence of untapped gray
matter. This is a false assumption. What is
correct, however, is that at certain moments
in anyones life, such as when we are simply
at rest and thinking, we may be using only
10 percent of our brains.
It turns out though, that we use virtually
every part of the brain, and that most of the
brain is active almost all the time, Gordon
adds. Lets put it this way: the brain represents three percent of the bodys weight and
uses 20 percent of the bodys energy. Although its true that at any given moment
all of the brains regions are not concurrently firing, brain researchers using imaging
technology have shown that, like the bodys
muscles, most are continually active over a
24-hour period.
(www.sciam.com/article. February 7, 2008. Adaptado.)

1. (Unifesp) Segundo o neurologista Barry Gordon:


a) A relao entre o peso e a energia consumida
pelo crebro semelhante dos outros rgos.
b) O crebro um msculo que deve ser exercitado sem interrupo.
c) A energia consumida pelo crebro varia de
10% a 90%, dependendo do peso do crebro.
d) William James estava correto em associar
Einstein maior utilizao da potencialidade cerebral.
e) Grande parte do crebro fica ativa quase o
tempo todo.
2. (Unifesp) The human brain:
a) is employed about 10% of the time in its
full capacity.
b) could use 90% of its energy in future, after
additional research.
c) may be used at only 10% of its full potential
in certain situations.
d) is highly demanded by people who have an
extraordinary memory for numbers.
e) was much more active in scientists like
Albert Einstein and William James.
3. (Unifesp) The statement by William James
We are making use of only a small part of
our possible mental and physical resources.
a) agrees with Einsteins theories.
b) supports the 10 percent myth.
c) confirms Barry Gordons ideas.
d) is an argument against the complexity of
the brain.
e) demonstrates that the brain is a Gods
mystery.
4. (Unifesp) No trecho do segundo pargrafo
they too could become savants who remember to the twenty thousandth decimal
place or perhaps even have telekinetic powers. , a palavra PERHAPS significa, em
portugus:
a) exceto.
b) at.
c) portanto.
d) talvez.
e) certamente.
5. (Unifesp) No trecho do terceiro pargrafo Its also been associated with Albert Einstein, who supposedly used it to explain his
cosmic towering intellect. , a letra S em
ITS indica a forma verbal:
a) is.
b) were.
c) has.
d) was.
e) had.

6. (Unifesp) No trecho do ltimo pargrafo


like the bodys muscles , a palavra
LIKE indica:
a) semelhana.
b) exemplificao.
c) generalizao.
d) probabilidade.
e) contraste.
TEXTO PARA A PRXIMA QUESTO
METHODS OF INSTRUCTION
Curiously, 2universities 3are very anxious to
do research on every institution in society
except 1themselves. They know 5a lot about
how smart students are on arrival, but very
little about how much 4has been learned
by graduation time. Because educators do
not investigate how well they teach or how
much students learn, they do not have any
process of trial and error by which to improve methods of instruction. And so teaching
remains one of the few human activities
that does not get demonstrably better from
one 7generation to the 6next.
7. (Unb) Marque V (verdadeiro) ou F (falso)
para cada uma das afirmaes abaixo.
( ) themselves (ref.1) refers to universities
(ref.2)
( ) are very anxious (ref.3) the same as ARE
VERY EAGER
( ) has been (ref.4) is a perfect form
( ) a lot (ref.5) can be correctly substituted
by A GREAT DEAL
( ) next (ref.6) refers to generation (ref.6)
TEXTO PARA A PRXIMA QUESTO
LONG LIVE LIFE!
Children born today will be able to live to
130 because of medical advances. For the
first time, scientists __________ the extra
years that breakthroughs in human genetics, organ cloning and the biology of ageing
will add to peoples lifespans.
They believe that by 2050, scientific advances __________ 50 years to the current average of 75 for men and 79 for women, precipitating radical changes in the way people
live, plan their careers, spend their leisure
time and raise families.
(From Long Live Life!. SPEAK UP. Ano
XII, 149, October 1999, page 8.)

8. (UFPE) Fill in the blanks with the correct


choice.
a) is quantified, has been adding
b) are quantified, has added
c) have quantified, will be adding
d) has quantified, is adding
e) was quantified, would add
113

TEXTO PARA A PRXIMA QUESTO


The Fear Is Old
The Economy New
By THOMAS L. FRIEDMAN

There is something perverse about reading


the business news these days. Every month
the Labor Department comes out with a new
set of statistics about how unemployment is
down and thousands of jobs are being created. But these stories always contain the
same 2caveat, like the warning on a pack of
cigarettes, that this news is bad for the health
of the economy. The stories always go on to
say that these great employment statistics
triggered panic among Wall Street investors
and led to a sell off of stocks and bonds.
(...)
Of course there has always been a link between unemployment numbers and inflation
expectations. The more people are working,
the more they have the money to pay for
things; the more consumer demand outstrips
factory capacity, the more prices shoot up,
and the more prices shoot up the more the
value of bonds, with their fixed interest rates, erodes.
But what has been so frustrating about the
market reactions in recent months is that
despite the surging economy, inflation has
not been rising. It has remained flat, at
around 3 percent, and 1yet Wall Street, certain that the shadow it sees is the ghost of
higher inflation come to haunt the trading
floors, has been clamoring to the Federal Reserve for higher rates. (...)
The New York Times Magazine. May 22, 1994.

9. (Ita) O que determinou a utilizao do Present Perfect Tense no ltimo pargrafo do


texto foi:
a) o estilo do autor.
b) a referncia a um tempo passado no explicitado no texto.
c) a referncia a acontecimentos / sentimentos
desencadeados no passado e que continuam
no presente.
d) a atribuio de maior nfase ao que se pretende dizer.
e) a referncia a sentimentos / acontecimentos
que ocorrem no presente.
TEXTO PARA A PRXIMA QUESTO
TEXT II
Blame male trees for your allergies

Scientists think they have found what causes people to sneeze and suffer runny noses and itchy eyes. Tom Ogren, a California
114

horticulturalist, suggests that we may suffer


allergies because in cities, we plant male
trees along streets. Male trees are chosen
over females because the latter produce seed
pods and fruits that need to be cleaned up.
Male plants dont require such maintenance,
but they produce great amounts of pollen,
and the pollen is what causes the hay fever
reactions in many people.
BLAME male trees for your allergies. Speak up
magazine. So Paulo: Camelot, Feb. 2001, p. 37.

1
0. (UFRRJ) The sentence Scientists think they
have found what causes people to sneeze...,
is equivalent to:
a) what causes people to sneeze is founded by
scientists.
b) what causes people to sneeze were found by
scientists.
c) what causes people to sneeze has been
found by scientists.
d) what causes people to sneeze was found by
scientists.
e) what are the causes of sneezing by people.

E.O. Dissertativo
1. Nas frases a seguir, os verbos em parnteses
devem ser escritos em simple past ou em past
perfect. Leia-as e escolha entre as duas formas.
O primeiro item j foi resolvido para voc.
a) Jill had phoned (to phone) Dad at work before she left (to leave) for her trip.
b) Sarah __________ (to turn on) the radio after she __________ (to wash) the dishes.
c) When she __________ (to arrive) the game
__________ already __________ (to start).
d) After the woman __________ (to come)
home she __________ (to feed) the cat.
e) Before he __________ (to sing) a song he
__________ (to play) the guitar.
f) She __________ (to watch) a video after the
children __________ (to go) to bed.
g) I __________ (to be) very tired because I
__________ (to study) too much.
2. Nas frases a seguir, os verbos em parnteses devem ser escritos em present perfect ou em past
perfect. Leia-as e escolha entre as duas formas.
O primeiro item j foi resolvido para voc.
a) When I went to the car park, I found that
my car had been stolen. (to steal)
b) Joseph __________ in Paris for 4 years before he could speak French fluently. (to live)
c) I __________ German for 5 years now.
(to learn)
d) The locals were amazed because they
__________ a camel before. (never, to see).

e) They wouldnt let him in because he


__________ his membership card. (to forget)
f) They said: You cant come in because you
__________ your membership card. (to forget)
g) __________ the new film by Fellini? (you,
to see)
h) They __________ a lot of champagne by the
time the party ended. (to drink)
i) They were angry because they __________
for too long. (to wait)
3. Escolha um verbo que complete adequadamente as lacunas, e grafe-o em uma das
seguintes formas: present perfect or simple
past. O primeiro item j foi resolvido para
voc.
a) John played football yesterday.
b) They __________ the car. It looks new again.
c) Last year we __________ to Italy.
d) Bill and Susan __________ the book. Now
they can watch the film.
e) I __________ my friend three days ago.
f) We __________ another country before.
g) She __________ a new car in 2013.
h) Im sorry, but I __________ my homework.
TEXTO PARA A PRXIMA QUESTO
FASHION: BYE BYE, BARBIE
The new breed of Brazilian beauty
BY MAC MARGOLIS

Its hard to imagine, but Suyane Moreira


didnt always like what she saw in the mirror. Sure, she was pretty enough, the sort
of pretty that can stop a conversation. And
with her cinnamon skin, the curtain of raven hair and deep, black, come-hither eyes,
who wouldnt notice? But until recently Moreira mostly saw what wasnt there. I wanted to be blond and blue-eyed, she says.
Her girlhood idol was Xuxa, the wildly popular childrens TV-show host, and Brazils
answer to Barbie.
No longer. Ever since she started strutting
for Ford Models late last year, this reedy
youngster from a drowsy village in northeastern Brazil has refashioned her attitude.
Moreira, who turns 19 in September, has posed for Italian Vogue and the British fashion
bible ID and will soon debut on catwalks at
New York and London. She knows its a steep
climb to that glamorous world where Brazilian bermodel Gisele reigns. But when Moreira consults the looking glass these days,
she sees what was there all along: a striking
young woman whose burnished skin and
angular features tell of deep indigenous
roots. I am proud to be Indian, she told
NEWSWEEK recently. I like the way I look.

In Brazils complicated social taxonomy, Moreira is a cafuza the progeny of African


and Indian ancestors. Her late father, a nightclub singer, was black. Her mother is a
descendant of full-blooded native Brazilians
which native Brazilians isnt clear. The genealogy died with Moreiras great-grandmother, who - family legend has it was stolen
from the cradle by white hunters on the Serra do Cariri, a scarp named after a bygone
Indian nation.
Native Brazilians have always inflamed the
national imagination, either as menaces or
mascots. For nearly 300 years, when they
outnumbered the European colonists, they
were seen either as barbarians at the gate or
as Christians in the rough. Many massacres
later, when they were no longer a threat,
they could be safely resurrected as cultural
icons and even heroes.
Does Moreiras rise represent ethnic pride or
opportunism? Has the fashion industry struck a blow for tolerance or found a pretty new
product for the ethnic market? Theres room
for skepticism, but marketing Indianness
is itself proof of changing attitudes. Moreira
is not about to shed her heels to return to
the reservation, but she no longer fancies
becoming a Brazilian Barbie. I have a dream, she confesses. Id love to spend some
days with an Indian tribe, learning their
dances and eating their food. Which tribe? Im not sure, she says, flicking her jet
hair and flashing a camera-ready grin. There
wont be any lack of invitations.
NEWSWEEK
Glossary
strutting: desfilar
reedy: esbelta
catwalks: passarelas
bermodel: topmodel
progeny: descendncia
bygone: extinta
menaces: ameaas
in the rough: em potencial
rise: asceno
struck a blow for: ser a favor defender
to shed her heels: abandonar seus saltos altos

Read the text and answer the following


questions in English.
4. (Uff) Qual a mudana principal observada
na atitude de Suyane em relao a sua prpria aparncia?
5. (Uff) Find in the text two different uses of
the present perfect:
a) one referring to an indefinite point in the past;
b) another which includes the present time.
115

6. A tabela a seguir traz os verbos irregulares mais comuns em lngua inglesa. Tente se lembrar das
formas de past participle (particpio passado) dos verbos, assim como de seus significados em
portugus. A tabela completa est disponvel no gabarito dos exerccios desta unidade.
infinitive

116

simple past

arise

arose

be

was, were

bear

bore

beat

beat

become

became

begin

began

bend

bent

bite

bit

bleed

bled

blow

blew

break

broke

bring

brought

build

built

burn

burned, burnt

burst

burst

buy

bought

cast

cast

catch

caught

choose

chose

come

came

cost

cost

cut

cut

deal

dealt

dig

dug

dive

dove, dived

do

did

draw

drew

dream

dreamed, dreamt

drink

drank

drive

drove

eat

ate

fall

fell

feed

fed

feel

felt

fight

fought

find

found

fit

fitted, fit

flee

fled

fly

flew

forbid

forbade

forecast

forecast

foresee

foresaw

forget

forgot

forgive

forgave

freeze

froze

get

got

past participle

traduo do infinitivo

infinitive

simple past

give

gave

go

went

grow

grew

hang

hung

have

had

hear

heard

hide

hid

hit

hit

hold

held

hurt

hurt

keep

kept

know

knew

lay

laid

lead

led

lean

leaned, leant

learn

learned, learnt

leave

left

lend

lent

let

let

lie

lay

lie

lied

light

lit, lighted

lose

lost

make

made

mean

meant

meet

met

overcome

overcame

overdo

overdid

overdraw

overdrew

overdrink

overdrank

overeat

overate

overfeed

overfed

overhang

overhung

overhear

overheard

overlay

overlaid

overpay

overpaid

override

overrode

overtake

overtook

pay

paid

plead

pleaded, pled

prove

proved

put

put

quit

quit, quitted

read

read

reset

reset

rid

rid

ride

rode

ring

rang

rise

rose

past participle

traduo do infinitivo

117

infinitive

118

simple past

run

ran

saw

sawed

say

said

see

saw

seek

sought

sell

sold

send

sent

set

set

sew

sewed

shake

shook

shine

shined, shone

shoot

shot

show

showed

shrink

shrank, shrunk

shut

shut

sing

sang

sink

sank, sunk

sit

sat

sleep

slept

slide

slid

speak

spoke

spend

spent

spin

spun

split

split

spread

spread

spring

sprang, sprung

stand

stood

steal

stole

stick

stuck

strike (hit)

struck

swim

swam

take

took

teach

taught

teel

told

think

thought

throw

threw

undergo

underwent

understand

understood

wear

wore

win

won

withdraw

withdrew

write

wrote

past participle

traduo do infinitivo

GABARITO
E.O. Teste I
1. B

2. A

3. A

4. D

5. E

6. C

7. B

8. C

9. C

10. D

E.O. Teste II
1. D

2. E

3. B

4. D

5. E

6. D

7. D

8. D

9. B

10. C

4. D

5. C

E.O. Teste III


1. E

2. B

3. B

6. A

7. V V V V V

8. C

9. C

10. C

E.O. Dissertativo

1.
a) J resolvido.
b) turned on had washed
c) arrived had already started
d) had come fed

e) sang had played


f) watched had gone
g) was had studied
2.
a) J resolvido.
b) had lived
c) have learned
d) had never seen
e) had forgotten
f) forgot
g) have you seen
h) had drunk
i) had waited
3.
a) J resolvido.
b) have washed
c) went
d) have read
e) saw
f) havent visited
g) bought
h) havent done
4.
Suyane Moreira desejava ser loura e ter
olhos azuis quando era criana. Atualmente, ela se mostra orgulhosa de sua origem e
aparncia indgena.
5.
a) has posed has struck found
b) Has refashioned Have always inflamed

6.
infinitive

simple past

past participle

traduo do infinitivo

arise

arose

arisen

levantar, trazer

be

was, were

been

ser, estar

bear

bore

borne

tolerar, portar

beat

beat

beat

bater

become

became

become

tornar-se

begin

began

begun

iniciar, comear

bend

bent

bent

dobrar, deturpar

bite

bit

bitten

morder

bleed

bled

bled

sangrar

blow

blew

blown

soprar, explodir

break

broke

broken

quebrar, interromper

bring

brought

brought

trazer

build

built

built

construir

burn

burned, burnt

burned, burnt

queimar

burst

burst

burst

estourar

buy

bought

bought

comprar

cast

cast

cast

lanar, divulgar

catch

caught

caught

pegar, apanhar

choose

chose

chosen

escolher

come

came

come

vir

cost

cost

cost

custar

cut

cut

cut

cortar

deal

dealt

dealt

lidar

119

infinitive

120

simple past

past participle

traduo do infinitivo

dig

dug

dug

cavar

dive

dove, dived

dived

mergulhar

do

did

done

fazer

draw

drew

drawn

desenhar, tirar, empatar

dream

dreamed, dreamt

dreamed, dreamt

sonhar

drink

drank

drunk

beber, tomar

drive

drove

driven

dirigir, conduzir

eat

ate

eaten

comer

fall

fell

fallen

cair

feed

fed

fed

alimentar

feel

felt

felt

sentir

fight

fought

fought

lutar

find

found

found

encontrar, descobrir

fit

fitted, fit

fitted, fit

encaixar, condicionar

flee

fled

fled

fugir

fly

flew

flown

voar

forbid

forbade

forbidden

proibir

forecast

forecast

forecast

prever

foresee

foresaw

foressen

prever

forget

forgot

forgotten

esquecer

forgive

forgave

forgiven

perdoar

freeze

froze

frozen

congelar

get

got

got, gotten

conseguir, obter

give

gave

given

dar, distribuir

go

went

gone

ir

grow

grew

grown

crescer, desenvolver

hang

hung

hung

pendurar, enforcar

have

had

had

ter

hear

heard

heard

ouvir

hide

hid

hidden

esconder

hit

hit

hit

atingir

hold

held

held

segurar

hurt

hurt

hurt

ferir

keep

kept

kept

manter, continuar

know

knew

known

saber, conhecer

lay

laid

laid

pr, colocar, depositar

lead

led

led

levar, conduzir, liderar

lean

leaned, leant

leaned, leant

apoiar, confiar

learn

learned, learnt

learned, learnt

aprender

leave

left

left

partir, deixar

lend

lent

lent

emprestar

let

let

let

deixar, permitir

lie

lay

lain

colocar, pr, assentar

lie

lied

lied

mentir

light

lit, lighted

lit, lighted

acender

lose

lost

lost

perder

make

made

made

fazer

mean

meant

meant

significar, querer dizer

meet

met

met

encontrar, conhecer

infinitive

simple past

past participle

traduo do infinitivo

overcome

overcame

overcome

superar

overdo

overdid

overdone

exagerar

overdraw

overdrew

overdrawn

retirar em excesso

overdrink

overdrank

overdrunk

beber em excesso

overeat

overate

overeaten

comer em excesso

overfeed

overfed

overfed

superalimentar

overhang

overhung

overhung

suplantar

overhear

overheard

overheard

ouvir por acaso

overlay

overlaid

overlaid

sobrepor

overpay

overpaid

overpaid

pagar em excesso

override

overrode

overriden

prevalecer

overtake

overtook

overtaken

suplantar, tomar o controle

pay

paid

paid

pagar

plead

pleaded, pled

pleaded, pled

apelar, suplicar

prove

proved

proved

provar

put

put

put

pr, colocar

quit

quit, quitted

quit, quitted

interromper, quitar, parar

read

read

read

ler

reset

reset

reset

reiniciar

rid

rid

rid

livrar-se

ride

rode

ridden

andar de bicicletar ou a cavalo

ring

rang

rung

tocar

rise

rose

risen

levantar, ascender

run

ran

run

correr, administrar

saw

sawed

sawn, sawed

plantar

say

said

said

dizer

see

saw

seen

ver

seek

sought

sought

procurar

sell

sold

sold

vender

send

sent

sent

enviar, mandar

set

set

set

preparar, arrumar, configurar

sew

sewed

sewed

costurar

shake

shook

shaken

chacoalhar

shine

shined, shone

shined, shone

brilhar

shoot

shot

shot

atirar

show

showed

shown, showned

mostrar

shrink

shrank, shrunk

shrunk

encolher

shut

shut

shut

calar-se, fechar

sing

sang

sung

cantar

sink

sank, sunk

sunk

afundar

sit

sat

sit

sentar

sleep

slept

slept

dormir

slide

slid

slid, sliden

deslizar

speak

spoke

spoken

falar

spend

spent

spent

gastar, passar

spin

spun

spun

girar

split

split

split

dividir, separar

spread

spread

spread

espalhar

spring

sprang, sprung

sprung

disparar

121

infinitive

122

simple past

past participle

traduo do infinitivo

stand

stood

stood

levantar

steal

stole

stolen

roubar

stick

stuck

stuck

colar, aderir

strike (hit)

struck

stricken

atingir, atacar

swim

swam

swum

nadar

take

took

taken

tomar, pegar

teach

taught

taught

ensinar

teel

told

told

dizer

think

thought

thought

pensar, assumir, supor

throw

threw

thrown

lanar, atirar

undergo

underwent

undergone

passar por, atravessar

understand

understood

understood

entender, compreender

wear

wore

worn

vestir

win

won

won

vencer, ganhar

withdraw

withdrew

withdrawn

retirar

write

wrote

written

escrever

Você também pode gostar